You are on page 1of 282

1

REASONING-ANALOGY
INTRODUCTION
Analogy means ‘‘resemblance of one object to another in
certain aspects’’. Analogy Scots are meant to test a
candidate's overall knowledge, power of reasoning and ability
to think concisely and accurately.
In this chapter, a question consist of words related to
each other based on some logic and it is required to find the
pair of words analogous to those given in the question.
To solve these questions, following two simple steps are to
be followed :
Step I The candidate is required to identify the
relationship between the pair of words given.
Step II Find out the other pair such that the relationship
between the third and the fourth words is similar to the
relationship that exists between the first and second words.
For Example : Devotee : Temple and Patient : Hospital
is a parallel pair of words because they have similar kind of
relationship between them. As devotees go to temple and
patients go to hospital.
Some most common types of analogous relationships are
as follows :
1. Antonymous Relationship : In this type of
analogy, one word in each pair means the opposite of
another.
For Example :
Employed : Jobless [First word is opposite to the second
word]
Fat : Thin [First word is opposite to the second word]
(1)
2 Fundamentals of Reasoning

2. Synonymous Relationship : In this types of


analogy, you will need to find words.
For Example :
Huge : Big [Huge and Big means the that convey the
same meaning same thing]
Lucky : Fortunate [Lucky and Fortunate means the
same thing]
3. Cause and Effect : In this type of analogy, one word
in the pair describes a condition or action, while the
other word describes an outcome or effect to that
action or condition.
For Example :
Fire : Death [Fire causes death]
Heavy Rain : Flood [Heavy rain causes flood]
4. Worker and Product Relationship : In this type
of analogy, one word of the pair represents the
working professionals and the other word of the pair
represents and their final product.
For Example :
Poet : Poem [A poet writes a poem, poem is the final
product of poet]
Farmer : crop [crop is the final product of farmer]
5. Worker and Tool Relationship : In this type of
analogy, one word of the pair represents the working
professionals and the other word of the pair is the
tool used for their working.
For Example :
Tailor : Needle [Needle is a tool used by Tailor]
Doctor : Stethoscope [Stethoscope is a tool used by
Doctor]
6. Part and whole Relationship : In this type of
analogy, one word of the pair represents a single part
of the whole object given in other word of the pair.
Reasoning-Analogy 3

For Example :
Pen : Nib [Nib is part of Pen]
car : Engine [Engine is a part of car]
7. Tool and Action Relationship : In this type of
analogy, one word of the pair represents the tool and
the other word gives its function/action.
For Example :
Binocular : View [The function of a binocular is to view]
knife : cut [The function of a knife is to cut]
8. Category/Type Relationship : In this type of
analogy, one word is the element/member of group
that other word describes.
For Example :
Salsa : Dance (Salsa is Similarly some other similar
kinds of analogous are)
Folk : Music [Folk is a type of music]
l Intensity relationship
l Animal/Thing and Sound Relationship
l Animal and Young one Relationship
l Male and Female Relationship
l Country and capital Relationship
l State and capital Relationship
l Quantity and unit Relationship
l Sports and Players Relationship
ANALOGY : SOLVED EXAMPLES
Advertisements :
In each word of the following questions consists of pair of
words bearing a relationship among these, from amongst the
alternatives, pick up the pair that best illustrate a similar
relationship.
1. Chess : Game
(a) King : Rule (b) Book : Read
4 Fundamentals of Reasoning

(c) Punjabi : Language (d) Doctor : Stethoscope


Ans. (c)
Explanation : Chess is a game and similarly Punjabi is a language.
2. Teacher : teaching
(a) Cricketer : mone (b) Doctor : care
(c) Driver : driving (d) None of the above
Ans (c)
Explanation : Teacher's job is teaching similarly driver's job is
driving.
3. Play ground : play
(a) Swimming Pool : Swim (b) Damage : flood
(c) Leather : shoe (d) Raincoat : cold
Ans. (a)
Explanation : In swimming pool we swim hence option A is the
answer.
4. Book : Chapter
(a) Pen : Pencil (b) Computer : Laptop
(c) Mobile : Landline (d) House : Kitchen
Ans. (d)
Explanation : (d) 'Chapter' is a part of 'Book' and similarly, kitchen is
a part of 'House'.
5. Sunrise : Sunset
(a) Dawn : Twilight (b) Noon : Midnight
(c) Morning : Night (d) Energetic : Lazy
Ans. (b)
Explanation : As 'Sunrise' and 'Sunset' are opposite to each other, in
the same manner 'Energetic' and 'Lazy' are opposite to each other.
6. Nepal Kathmandu
(a) India : Mumbai (b) Italy : Rome
(c) China : Shanghai (d) France : Lyon
Ans. (b)
Explanation : (b) Country and capital pair.
7. Steel : Alloy
(a) Zinc : Halogen (b) Sulphur : Metal
(c) Tin : Nonmetal (d) Carbon : Non metal
Ans. (d)
Explanation : Steel is an alloy and carbon is a non-metal.
8. Silence : Noise
(a) Sing : Dance (b) Doctor : engineer
Reasoning-Analogy 5

(c) Up : down (d) None of the above


Ans. (c)
Explanation : As silence is opposite of noise, similarly. Down is
opposite of up.
9. Poet : Poem
(a) Summer : rain (b) Brush : painting
(c) Soldier : Gun (d) Producer : Film
Ans. (d)
Explanation : Poem is the product of poet, similarly film is final
product of producer.
In the following question, choose the pair/group of words the show the
name relationship as given at the top of every pair/group.
10. India : Pakistan
(a) USA : Canada (b) Pakistan : Afghanistan
(c) England : Afghanistan (d) None of the above
Ans. (c)
Explanation : A group of 'Fish' is called' Shoal'. Similarly, a group of
elephants is called 'Flock'.
11. Candle : Wick
(a) Bicycle : wheel (b) Light : Bulb
(c) Lawyer : curt (d) Hammer : Nail
Ans. (a)
Explanation : As wick is a part of candle similarly what is a part of
Bicycle.
12 Odisha : Bhubaneswar
(a) Nagaland : Shillong
(b) Kerala : Thiruvananthapuram
(c) Jagatsinghpur : Cuttack
(d) Maharashtra : Bhopal
Ans. (b)
Explanation : As Bhubaneswar is capital of odisha, similarly
Thiruvanthapuram is capital of Kerala.
13. Bible : Quran
(a) Brinjal : lady finger (b) Bat : ball
(c) Tree : Leaves (d) None of the above
Ans. (a)
Explanation : Bible and Quran belong to same class i. e. , religious
books and Brinjal and Ladyfinger, also belong to same class i. e. , vegetables.
In the following question, choose the pair/group of words that show the
same relationship as given at the top of every pair/group.
6 Fundamentals of Reasoning

14. Triangle : Hexagon


(a) Cone : Sphere (b) Rectangle : Octagon
(c) Pentagon : Heptagon (d) Angle : Quadrilateral
Ans. (d)
Explanation : Sides in figure second are double of sides of figure first.
15. Frankness : Blunt
(a) Substitute : Replace (b) Weep : Laugh
(c) Sickness : Death (d) Rest : Activity
Ans. (a)
Explanation : 'Frankness' and 'Blunt' are synonyms and so are
substitute Replace.
16. Premchand : Gaban
(a) Madam Curie : Radium (b) Shakespeare : Skylark
(c) Dickens : Bleak House (d) Nobel : Dynamite
Ans. (c)
Explanation : 'Gaban' is written by Premchand and Bleak House by
'Dickens'.
17. Milk : Cream
(a) College : Professors (b) Sugar : Sweet
(c) Clay : Pottery (d) Fruit : Mango
Ans. (d)
Explanation : 'Cream' is made from 'Milk'. Likewise, 'Pottery' is
made from 'Clay'.
18. Errors : Inexperience
(a) Skill : Mistake (b) Training : Economy
(c) Losses : Carelessness (d) Picture : See
Ans. (c)
Explanation : As 'Inexperience' causes 'Errors', similarly,
'Carelessness' causes 'Losses'.
19. Murk : Dark
(a) Abysmal : Superb (b) Possible : Occur
(c) Deep : Shallow
(d) Simultaneous : coincident
Ans. (b)
Explanation : (c) Both the words are synonyms of each other.
20. Marble : sculptor
(a) Canvas : painter (b) mechanic : Garage
(c) Driver : driving (d) Mechanic : Garage
Ans. (a)
Reasoning-Analogy 7

Explanation : A sculptor design on marble similarly a painter paints


on canvas.
PRACTICE SET
Directions (Q1-10) For each of the following questions
select the answer pair that expresses a relationship most
similar to that expressed in the given pair.
1. Rectangle : Pentagon
(a) Circle : Square (b) Hexagon : Octagon
(c) Triangle : Rhombus (d) Diagonal : Perimeter
Ans. (a)
2. Sorrow : Misery
(a) Happiness : Joy (b) Song : Music
(c) Chaos : Peace (d) Expand : conduce
Ans. (a)
3. Soldier : Gun
(a) Baker : Over (b) Musician : Instrument
(c) Farmer : Needle (d) Carpenter : Saw
4. Deceit : Honesty
(a) Truth : Beauty (b) Sportsmanship : Fortitude
(c) Courageous : unheroic (d) Flaw : Defect
Ans. (d)
5. Apple : lychee
(a) Music : Notes (b) Bulbul : Cuckoo
(c) Books : Preface (d) Reptiles : Crawl
Ans. (b)
6. Red Fort : Delhi
(a) Bodh Gaya : Gujarat (b) Hawa Mahal : Kolkata
(c) Char Minar : Hyderabad (d) Amarnath Cave :
Uttarakhand
Ans. (c)
7. Triangle : Hexagon
(a) Cone : Sphere (b) Rectangle : Octagon
(c) Pentagon : Heptagon (d) Angle : Quadrilateral
Ans. (a)
8. Horse : Neigh
(a) Dog : Growl (b) Lion : Bleat
(c) Snake : Creep (d) Parrot : Grunt
Ans. (a)
8 Fundamentals of Reasoning

9. Europe : Continent
(a) Yuri Gagarin : Space (b) Andes : River
(c) Engine : Train (d) Vikramaditya : ship
10. Thermometer : Temperature
(a) length : breadth (b) Cardiograph : heart rate
(c) Solar energy : Sun (d) Millimeter : Scale

ANSWER KEY
1. (c) Second figure has one more side than the first figure.
2. (a) Both are synonyms of each other.
3. (d) A soldier uses gun for doing operation, similarly
carpenter uses saw for doing his operations.
4. (d) They both are antonyms of each other, Deceit means
practice of misleading others through lies which is
opposite of honesty.
In the same way unheroic means coward which is
antonym of courageous.
5. (b) both the words belong to the same category.
6. (c) Red Fort is located in Delhi and similarly char Minar is
located in Hyderabad.
7. (a) Both are synonyms of each other.
8. (a) As neigh is sound of horse, similarly growl is sound of dog.
9. (d) As europe is a continent, similarly vikramaditya is a ship.
10. (b) As temperature is measured from a thermometer in the
same way heart rate is measured with cardiograph.

qqq
2 REASONING-ALPHA
NUMERIC SEQUENCE
INTRODUCTION
Alphanumeric sequence is an important chapter in
reasoning and the questions on this subject are asked in
almost all the competitive exams. Alphanumeric sequence is
a sequence which comprise both upper and lower case
alphabetic characters, punctuation marks, symbols such as
@, & * etc. and numbers.
For example we can make an alphanumeric sequence as
follows :
s (# ) ER9 % T5 ( b) 6 @ D 8
In the above sequence, we can see that there are
numbers, alphabets, and symbols. These types of sequences
are called alphanumeric sequence. We can make many
sequences like this and can deduce questions based on the
sequences.
Important Points to Remember : Before jumping to
the questions, Let us understand the meaning of key words
that are generally used in such questions like "following,
followed by, preceding, and preceded by".
Questions regarding this chapter can come in
this way :
l Let us take two successive alphabets : P and Q. Here
P is preceding Q and Q is preceded by P. Also, Q is
following P and P is followed by Q.
l Now take three successive alphabets : P ,Q and R.
Here P and Q are preceding R, Z , but Q is
immediately preceding R. Similarly, Q and R are
following P, but Q is immediately following P
whereas, R is not immediately following P.

(9)
10 Fundamentals of Reasoning

(a) Which element is n th from the left of the sequence?


To get the answer of the above question, we have to look at the
sequence from the left side and have to count the exact number
that is asked in the question.
(b) Which element is n th from the right of the sequence ?
To get the answer of the above question, we have to look at the
sequence from the right side and have to count the exact
number that is asked in the question.
(c) Which element is m th to the right of n th from the left of the
sequence ?
To get the answer for such type of questions, we have to find out
the n th element from the left side of the sequence and then find
out the m th element to the right of that particular element.
(d) Which element is m th to the left of n th from the left of the
sequence ?
To get the answer for such type of questions, we have to find out
the n th element from the left side of the sequence and then find
out the m th element to the left of that particular element.
(e) Which element is m th to the right of n th from the right of the
sequence ?
To get the answer for such type of questions, we have to find out
the n th element from the right side of the sequence and then
find out the m th element to the right of that particular
element.
(f) Which element is m th to the left of n th from the right of the
sequence ?
To get the answer for such type of questions, we have to find out
the n th element from the right side of the sequence and then
find out the m th element to the left of that particular element.
Formats of the questions will give a better idea about
such test.
Directions (Example No. 1-3 study the following
sequence carefully an answer the questions that fellow.)
1. Which element is 3 rd to the right of 5 th from left :
(a) 6 (b) ε (c) & (d) 2
Ans. (d) 2
Explanation :
Reasoning–Alpha Numeric Sequence 11

1 2 3 4 5 6 7 8 9 10 11 12 13 14 15 16 17
A $ % 5 G O& 2 E 6 3 N K * ! H 2
2. How many numbers are the there are immediately preceded
by a symbol and followed by a consonant :
(a) One (b) Two (c) Three (d) None
Ans. (a) One
Explanation : Only one is possible which is % 5G
3. Which element is 4 th to the left 11 th from left
(a) ! (b) & (c) % (d) 3
Ans. (b) &
Explanation :
1 2 3 4 5 6 7 8 9 10 11 12 13 14 15 16 17
A $ % 5 G O& 2 E 6 3 N K * ! H 2
4. In the following series, which element will be 5 th to the right
of 16 th element from the right end if the given series is written
in the reverse order :
7 P # V E 9 B (c) A 3 ? 5 C * Z % 6 D & Q @ 1
(a) % (b) B (c) 5 (d) ?
Ans. (d) ?
Explanation : Reversed sequence : 1 @ Q & D 6 % Z * C 5 ? 3 A
(c) B 9 E V # P 7 Hence, now the 16 th element from the eight
end is % and 5 th to the right of % is ?
5. How many letters are immediately followed by L in the given
letter series :
CPMLMMBLTLDESL
(a) 5 (b) 3 (c) 4 (d) 6
Ans. (c) 4
Explanation : C P M L M M B L T L D E S L
Clearly, M, B, T and S are immediately followed by L.
6. In the following series of numbers, find out how many time 1, 3
and 7 have appeared together, 7 being in the middle and 14
and 3 on either side of 7 :
2 9 3 1 7 3 7 7 7 1 7 3 1 7 3 8 5 7 1 3 7 1 17 3 9 0 6
(a) 2 (b) 4 (c) 3 (d) 5
Ans. (b) 4
Explanation : Clearly, the given number series is as follows :
2931737771731738571371773906
12 Fundamentals of Reasoning

1/ 3 7 1/ 3
∴ Required pattern =
Preceding Middle Following
Therefore, total number of such patterns = 4
ALPHA NUMERIC SEQUENCE-SOLVED EXAMPLES
Directions (Example No. 7 – 10) Study the following
sequence carefully and answer the questions that follow :
6RoT4A8%BF1E#W@9H*MN032V5GP7Q
7. How many such consonants are there in the above
arrangement, each of which is immediately preceded by a
number but not immediately followed by a number :
(a) Three (b) One (c) Two (d) None
Ans. (a) Three
Explanation : According to question :
Consonant-Consonant-Number is the required pattern.
6 R o T 4 A 8 % B F 1 E # W @ 9 H Z m N 32 V 5 9 P 7 Q

× × × × × × × × × × ×

∴ Required consonant = 3 times


8. How many such vowels are there in the above arrangement,
each of which is immediately preceded by a letter but
immediately not followed by a symbol :
(a) None (b) One (c) Two (d) Three
Ans. (b) One
Explanation : According to question :
6 R o T 4 A 8 % B F 1 E # W @ 9 H I M N 32 V 5 G P 7 Q

× × ×
Letter-Vowel-Letter/Number is the required pattern.
∴ Required vowel = 1 time
9. Three of the following four are alike in a certain way based on
their position in the above arrangement and so form a group.
Which of the following does not belong to that group :
(a) 4o6 (b) F%A (c) H@ (b) 523
Ans. (d) 523
Explanation : According to question :
−2 −2
4  → o  → 6
−2 −2
F  → %  → A
−2 −2
*  → H  → @
Reasoning–Alpha Numeric Sequence 13

−2 −2
5  → 2  → 3
10. If all the symbols are eliminated from the above arrangement,
than which of the following will be the 5 th to the right of 17 th
element from the right end :
(a) T (b) G (c) H (d) W
Ans. (c) H
Explanation : According to question :
After eliminating symbols New sequence :
6 R O T 4 A 8 B F 1 E W 9 H I M N 32 V G P 7 Q
5 th to the right of 17 th from the right end
= (17 − 5)
= 12 th from right end
=H
Directions (Example No. 11-13). Study the following
sequence carefully and answer the question.
M3R#AP49@KD1UH$2NWEQ6TV7*8BX
11. How many such numbers are there in the above arrangement
which are immediately preceded by a symbol :
(a) 2 (b) 1 (c) 4 (d) 3
Ans. (a) 2
Explanation :
× × × × × ×

M 3 R # A P 4 9 @ K D 1 U H 5 2 N W E Q 6 T V 7 * 8 B X

∴ Required number = 2
12. If started from M, M is interchanged with 3 and R is
interchanged with # and so on than which element will be
twelfth from left :
(a) 1 (b) $ (c) N (d) D
Ans. (d)D
Explanation : New sequences :
3M#RPA9 4 K@1DHU2$WNQET67U8*XB

From left
th
12

12 th from left end = D


13. What will come next in the following sequence : MRP, 9KU, $ N
Q. based on the above arrangement :
14 Fundamentals of Reasoning

(a) 178 (b) T*8 (c) T7B (d) QT7


Ans. (c) T7B
Explanation :
M3R#AP49@KD1UH$2NWEQ6TV7*8BX

So, following the sequence will be T7B.


14. In the given number series, how many 4's are there which are
immediately preceded by a prime number but not immediately
followed by a prime number :
41 41 54 26 41 83 49 24 83 48 28 45 48 74 64 54
(a) 3 (b) 5 (c) 4 (d) 6
Ans. (b) 5
Explanation :
× × × × × ×

4 1 4 15 4 2 6 4 1 8 3 4 9 2 4 8 3 4 8 2 8 4 5 4 87 4 6 4 5 4

∴ Required 4 = 5 times
15. Which digit comes in the middle of the given series :
24354456788956717
(a) 8 (b) 7 (c) 6 (d) 5
Ans. (b) 7
Explanation : Total number of digits = 17
17 + 1 18
∴ Middle digit = = = 9 th digit
2 2
∴ 9th digit in the given series = 7
PRACTICE SET
1. How many such 6's are there in the following number series,
each of which is immediately preceded by 1 or 5 and
immediately following by 3 or 9 :
2637564296134163915692316923165432
19671
(a) Nine (b) Five (c) Two (d) Three
Ans. (d) Three
2. How many letters are immediately followed by L in the given
letter series :
CPMLMMBLTLDESL
(a) 5 (b) 3 (c) 4 (d) 6
Reasoning–Alpha Numeric Sequence 15

Ans. (c)
3. In the following letter series, how many time B, C and N have
appeared together such that C in is the middle and B and N
are on any one side :
BCMXNCXNBXNCBNCBYBCXNBCNABONMZC
B
(a) 4 (b) 3 (c) 5 (d) 6
Ans. (b)3
4. Which of the following is the 10th to the right of the 19th from
the right end of the above arrangement :
(a) 2 (b) * (c) # (d) T
Ans. (d) T
5. How many such consonants are there in the above
arrangement, each of which is immediately preceded by a
number but not immediately followed by a number :
(a) None (b) One (c) Two (d) Three
Ans. One
6. How many such symbols are there in the above arrangement,
each of which is immediately preceded by number and
immediately followed by a letter :
(a) There (b) None (c) One (d) Two
Ans. (a) There
7. If all the symbols are dropped from the above arrangement,
which of the following will be the 14th from the left end :
(a) T (b) 3 (c) 6 (d) R
Ans. (c) 6
8. Three of the following four are alike in a certain way based on
their positions in the above arrangement and so form a group.
Which is the one that does not belong to that group :
(a) (c) 2 4 (b) 59P (c) RV @ (d) VG8
Ans. (c) RV @
9. In the following series, If all the alphabets of the series are
written in reverse order, then which of the following will be
8th to the right of the 7th from the left end of the series :
CEBACDBCDACEBEDCABACEDUBAUBDBU
(a) D (b) B (c) A (d) C
Ans. (b) B
16 Fundamentals of Reasoning

10. How many even numbers are there in the following sequence
of number each of which is immediately followed by an odd
number as well as immediately preceded by an even number :
86796832753842358526119
(a) Six (b) Three (c) Five (d) None of these
Ans. (d) None of these
qqq
3
REASONING-BLOOD RELATIONS
REASONING-BLOOD RELATIONSHIP
Introduction
Blood Relation problems mainly deal with hierarchical
structure of i. e., grandparents, parents and children etc. In
this, a chain of relationships is given in the form of
information and on the basis of this information, relation
between any two members of the chain is asked from the
candidate. Candidates are supposed to be familiar with the
knowledge of different relationships in the family.
Directions (Example No. 3-5) Read the following
information carefully and answer the questions given below :
Mohapatra family is very famous for their shopping
malls. The owner of house is Umesh Mohapatra. He has two
sons. Their names are Rakesh and Rohit Mohapatra. Rakesh
is married to Shrui. Shilpa is sister of Shilpa's mother's name
is Sujata. Deepak is nephew of Rohit. Rohit's mother is Gita.
IMPORTANT BLOOD RELATIONS
1. Mother's Husband = Father
2. Father's Wife = Mother
3. Father's or Mother's Son = Brother
4. Father's or Mother's daughter = Sister
5. Father's or Mother's brother = Uncle
6. Father's or Mother's sister = Aunt
7. Father's or Mother father = Grandfather
8. Father's or Mother = Grandfather
9. Brother's or sister's son = Nephew
10. Brother's or sister's Daughter = Niece

(17)
18 Fundamentals of Reasoning

11. Uncle's or Aunt's son or daughter = Cousin


12. Sister's Husband = Brother-in-law
13. Brother's Wife = Sister-in-law
14. Son's wife = Daughter-in-law
15. Daughter's husband = son-in-law
16. Husband's or wife's sister = sister-in-law
17. Husband's or wife's brother = Brother-in-law
18. The only son of your father = yourself
19. Your father's wife = Mother
20. Son of the only son of the father = son
Relations from one generation to next :
Generation I Gandfather, Grandmother, maternal grandfather,
maternal grandmother

Generation II Mother, Father, Uncle, Aunt, Maternal uncle,


Maternal aunt

Generation III Cousin Sister, Sister-in-law, brother, brother-in-law

Generation IV Son, daughter, nephew, niece

1. Pointing to a photograph of a boy Rahul said, "He is the son of


the only son of my mother". How is Rahul related to that boy :
(a) Brother (b) Father (c) Cousin (d) Uncle
Ans. (b) Father
Explanation : Rahul's mother's only son is Rahul himself. So,
the boy in the photo is Rahul's son, i.e., Rahul is the boy's
father.
2. Introducing a boy, a girl said, "He is the son of the daughter of
the father of my uncle". How is the boy related to the girl :
(a) Son-in-law (b) Uncle
(c) nephew (d) brother
Ans. (d) brother
Reasonsing–Blood Relation 19

Explanation : The father of the boy's uncle → the grandfather


of the by and daughter of the grandfather → sister of father.
3. How is Gita related to Umesh
(a) Mother (b) Sister (c) Aunt (d) Wife
Ans. (d) Wife
Explanation : Option D is correct because Gita is Rohit's
mother and Umesh is Rohit's father.
4. How is Sujata related to Rakesh.
(a) Mother (b) Mother-in-law
(c) Aunt (d) Sister-in-law
Ans. (b) Mother-in-law
Explanation : Shilpa is Shruti's sister, so Sujata is also
Shruti's mother.
5. What is relationship between Deepak and Rakesh :
(a) Son-father (b) Father-son
(c) Brother (d) Cousins
Ans. (a) Son-father
Explanation : As Deepak is nephew of Rohit so he must be
Rakesh's son.
6. Pointing to Punit, Aditya said, "I am the only son of one of the
sons of his father. "How is Punit related to Aditya :
(a) Father (b) Uncle
(c) Father or Uncle (d) Nephew
Ans. (c) Father or Uncle
Explanation : The only son of one of the sons of Punit's father
→ Either Punit is the father or uncle of Aditya.
7. Anupan said to a lady sitting in a car, "the only daughter of the
brother of my wife is the sister-in-law of the brother of your
sister. "How the husband of the lady is related to Anupam:
(a) Uncle (b) Nephew
(c) Maternal Uncle (d) Son-in-law
Ans. (d) Son-in-law
Explanation : Anupan's son-in-law is the brother of the lady
who was sitting in the car. Hence, the husband is also the
son-in-law of Anupam.
8. Pointing towards a girl, Arun said, "she is the only daughter of
my grandfather's son. "How is that girl related to Arun :
(a) Aunt (b) Sister (c) Cousin (d) Wife
20 Fundamentals of Reasoning

Ans. (b) Sister


Explanation : Option (b) is correct because Arun's
grandfather's only son is Arun's father. Therefore the girl is
Arun's sister.
9. Pointing towards a woman, Amit said, "She is the daughter of
my father's sister". How is the woman related to Amit.
(a) Brother (b) Uncle (c) Cousin (d) Sister
Ans. (c) Cousin
Explanation : Sister of Amit's father is Amit's aunt and
daughter of aunt is cousin. Hence woman is the cousin of Amit.
10. Introducing Jay, a woman said, " His wife is the only girl child
of my father. "How is Jay related to that woman :
(a) Uncle (b) Father-in-law
(c) Husband (d) Grandfather
Ans. (c) Husband
Explanation : The only girl child of the woman's father is she
herself. Hence, the woman and Jay has a wife-husband
relationship.
11. Introducing Darsh, Jyoti said, " he is the only son of my
mother's mother. "How is Jyoti related to Darsh :
(a) Mother (b) Aunt (c) Cousin (d) Niece
Ans. (d) Niece
Explanation : Darsh is brother of Jyoti's mother, so he is
maternal uncle of Jyoti and Jyoti is his niece.
12. Amit said to Mohan, "The boy in red shirt is younger of the two
brothers of the daughter of my father's wife." How is the boy in
red shirt related to Amit.
(a) Father (b) Uncle (c) Brother (d) Nephew
Ans. (c) Brother
Sol. : Following family diagram can be drawn from the given
information.
Married Couple
Amit's Amit' s mother
father
Daughter
Boy in
Sister Brother
Amit's Daughter of Amit's mother red shirt

Brother
Now, it is clear that the boy in red shirt is the brother of Amit.
Reasonsing–Blood Relation 21

Explanation : The Daughter of father's wife is sister of Amit.


Hence, the boy in red shirt is the brother of Amit.
13. A woman said to a man, "The daughter of your only sister is the
sister of my husband. "What is the relation of man's sister to
the woman :
(a) Mother-in-law (b) Mother
(d) Data inadequate (d) None of these
Sol. (b) The family diagram showing the relation is drawn as

Hence, the man's sister is mother-in-law of woman.


Explanation : Ravi, Govind and Ajay, all are brother's and
sakshi is wife of Ravi. Hence Ajay is Sakshi's brother-in-law.
14. Looking at a photograph, a person said, "I have no brother or
sister but that man's father is my father's son." At whose
photograph was the person looking at :
(a) His own (b) His son's
(c) his father's (d) His nephew's
Ans. (b) His son's
Explanation : The only child of person's father is the person
himself. So, the man in photograph is his own son.
15. Kamal told Vinay, "Yesterday I defeated the only brother of the
daughter of my grandmother. "Whom did Kamal defeated :
(a) Father (b) Brother
(c) Son (d) Father-in-law
Ans. (a) Father
Explanation : The daughter of grandmother is aunt of Kamal
and the only brother of Kamal's Aunt is his father.
PRACTICE SET
1. Abhiskeh has brother, Anil, Abhiskeh is the son of Rajesh.
Mukesh is Rajesh father. In terms of relationship, what is Anil
of Mukesh.
(a) Son (b) Grandson (c) Brother (d) Grandfather
Ans. (d) Grandfather
22 Fundamentals of Reasoning

2. Pointing to a lady, Aman said, 'She is the only daughter of my


father-in-law's wife. How is this lady related to Aman.
(a) Mother (b) Wife (c) Cousin (d) Sister
Ans. (b) Wife
3. Soham said to Yash. "This girl is the wife of the grandson of my
mother". How is Soham related to this girl :
(a) Father (b) Brother
(c) Son (d) Father-in-law
Ans. (a) Father
Directions : (Q. No. 4-6) Read the following information
carefully to answer the questions that follow.
I. Ruchi and Suresh are married couple having two
children. Manisha and Divya.
II. Divya is married to Amit who is the son of Gauri and
Anuj.
III. Nisha is the daughter of Amit.
IV. Karuna, who is Amit's sister, is married to Harish
and has two sons. Vijay and Parikshit.
V. Gauri is the grandmother of Parikshit.
VI. Manisha is the maternal aunt of Nisha.
4. How is Karuna related to Divya :
(a) Sister (b) Amit
(c) Sister-in-law (d) Mother
Ans. (c) Sister-in-law
5. What is the relationship between Vijay and Nisha :
(a) Cousins (b) Husband-wife
(c) Father-Daughter (d) None of these
Ans. (a) Cousins
6. Which of the following is true :
(a) Anuj is Divya's maternal uncle.
(b) Vijay is the son of Manisha
(c) Gauri is Harish's mother-in-law
(d) Nishi is the cousin of Karuna
Ans. (c) Gauri is Harish's mother-in-law
Reasonsing–Blood Relation 23

7. Looking at a woman sitting next to him, Amit said, "She is the


sister of the husband of my wife." How is the woman related to
Amit :
(a) Niece (b) Daughter (c) Sister (d) Wife
Ans. (c) Sister
8. Mudit told his friend chirag, pointing to a photograph, "Har
father is the only son of my mother. " The photograph is of
whom :
(a) Mudit's niece
(b) Mudit's sister
(c) Mudit's daughter
(d) Mudit's mother
Ans. (c) Mudit's daughter
9. Aman is the brother of Ankit. Kanika is the sister of Shubham.
Ankit is the son of Kanika. How is Aman related Kanika.
(a) Son (b) Brother (c) Nephew (d) Father
Ans. (a) son
10. A man showed a boy next to him said, "He is the son of my wife's
sister-in-law but I am the only child of my parents. "How is my
son related to him :
(a) Cousin (b) Nephew (c) Brother (d) Uncle
Ans. Cousin
qqq
4
REASONING-CLASSIFICATIONS
INTRODUCTION
As we know that various objects possess various
properties so we can group those objects who have properties
common to all. This common feature must be unique,
ascertainable, non-imaginary and must present in odd-one
out option. This phenomena of grouping various objects on
basis of their common properties is known as classification.
Classification allows us to make a systematic and a
homogeneous group out of a heterogeneous group. While
solving these questions one should have basic awakener
about general knowledge including science, Geography,
History, Mathematics etc.
Questions relating to classification normally enable the
examiner to check the candidate's ability to classify given
objects.
SAMPLE EXAMPLE
1. There of the following four are same in a certain way and hence
form a group. Find out the one which does not belong to that
group.
(a) Mountain (b) Hill
(c) Plateau (d) Rage
Ans. (c) Plateau
Explanation : All other items height except plane.
2. Three of the following four are same in a certain way and hence
form a group. Find out the one which does not belong to that
group.
(a) Swans (b) Geese (c) Duck (d) Cuckoo
Ans. (d) Cuckoo
Explanation : All others are water-birds except cuckoo.

(24)
Reasoning-Classifications 25

3. Three of the following four are same in a certain way and hence
form a group. Find out the one which does not belong to that
group.
(a) Mercury (c) Uranium (c) Silver (d) Iron
Ans. (a) Mercury
Explanation : Mercury is a liquid metal whereas all others are
solid ones.
4. Three of the following four are same in a certain way and hence
form a group. Find out the one which does not belong to that
group.
(a) Ego (b) Anger (c) Greed (d) Brave
Ans. (d) Brave
Explanation : All other options are indicating negative
human traits except option (d).
5. There of the following four are same in a certain way hence
form a group. Find out the one which does not belong to that
group.
(a) CPU (b) Computer (c) Monitor (d) Keyboard
Ans. (b) Computer
Explanation : All others are parts of a computer.
6. Three of the following four are same in a certain way and hence
form a group. Find out the one which does not belong to that
group.
(a) Yellow (b) Indigo (c) Orange (d) Pink
Ans. (d) Pink
Explanation : All others are constituents of the pattern
VIBGYOR.
7. Three of the following four are same in a certain way and hence
form a group. Find out the one which does not belong to that
group.
(a) Helmet (b) Turban (c) Veil (d) Cap
Ans. (c) Veil
Explanation : All other cover the head, while veil covers the
face.
8. Three of the following four are same in a certain way and hence
form a group. Find out the one which does not belong to that
group.
(a) Ayurveda (b) Yajurveda
(d) Rigveda (d) Atharvaveda
26 Fundamentals of Reasoning

Ans. (a) Ayurveda


Explanation : All except Ayurveda are names of holy
scriptures, Ayurveda is a branch of medicine.
9. Three of the following four are same in a certain way and hence
form a group. Find out the one which does not belong to that
group.
(a) Bhagawad Gita (b) Bible
(c) Quran (d) Jainism
Ans. (d) Jainism
Explanation : Jainism is a religion whereas others are holy
books of different religions.
10. Three of the following four are same in a certain way and hence
form a group. Find out the one which does not belong to that
group.
(a) Euro (b) Shora (c) Dinar (d) Pero
Ans. (b) Shora
Explanation : All others are currencies, while shore is the
parliament of Afghanistan.
11. Three of the following four are same in a certain way and hence
form a group. Find out the one which does not belong to that
group.
(a) Asia (b) Australia
(c) Argentina (d) Antarctica
Ans. (c) Argentina
Explanation : All except Argentina are continents, while
Argentina is a country
12. Three of the following four are same in a certain way and hence
form a group. Find out the one which does not belong to that
group.
(a) Cube (b) Square (c) Cuboid (d) Cone
Ans. (b) Square
Explanation : All other are three-dimensional bodies.
13. Three of the following four are same in a certain way and hence
form a group. Find out the one which does not belong to that
group.
(a) Kidnap (b) Murder
(c) Assassinate (d) Kill
Ans. (a) Kidnap
Reasoning-Classifications 27

Explanation : All other are actions of killing.


14. Dues same as above :
(a) Kabul (b) London (c) Shangai (d) Dublin
Ans. (c) Shangai
Explanation : All others are capitals of the countries, while
'Shangai' is the city of china.
15. Ques same
(a) Artery (b) Neuron (c) Ciorta (d) Ventricile
Ans. (b) Neuron
Explanation : Except neuron all other terms are related to
heart.
PRACTICE SET-1
Directions : (Q no. 1-15) In the following questions,
three alternatives are same in a certain way out of four and so
form a group. Find the odd word that does not belong to the
group.
1. (a) Gangtok (b) Bhubaneshwar
(c) Surat (d) Mahavira
Ans. (c) Surat
2. (a) Mahatma Gandhi (b) Guru Nanak
(c) Jesus (d) Mahavira
Ans. (a) Mahatma Gandhi
3. (a) Mile (b) Yard
(c) Kilometre (d) litre
Ans. (d) litre
4. (a) Papaya (b) Matermelon
(c) Guava (d) Jackfruit
Ans. (b) Matermelon
5. (a) Ample (b) Abundance
(c) Adequate (d) Plentiful
Ans. (b) Abundance
6. (a) Vulgar (b) Coarse
(c) Blunt (d) Unrefined
Ans. (c) Blunt
7. (a) Poet (b) Author (c) Novelist (d) Publisher
Ans. (d) Publisher
28 Fundamentals of Reasoning

8. (a) Run (b) Walk (c) Think (d) Jump


Ans. Think
9. (a) Universe (b) Moon (c) Star (d) Sun
Ans. (a) Universe
10. (a) Kilogram (b) Kilometre
(c) Tonnes (d) Quintals
Ans. (b) Kilometre
qqq
5
REASONING-CLOCK
INTRODUCTION
The face or dial of a clock is a circle whose circumference
is divided into 60 equal parts called minute spaces and 12
equal parts called hour spaces. A clock has two hands, the
smaller one is called the hour hand which gives hour while
the larger one is called minute hand which gives minutes.
1 hour = 60 minutes = 3600 seconds.
The circumference of the dial of a clock is equal to 360°.
The angle between two consecutive digits/numbers of a
clock is 30°.
When minute hand completes one round, then it crosses
60 min spaces of dial.
i. e., 60 min spaces = 360°
360°
⇒ 1 min space = = 6°
60
∴ minute hand traces an angle of 6° in 1 min.
When hour hand completes a round, then it crosses 12 h
spaces of dial.
360°
12h = 360° ⇒ 1h = = 30°
12
Hour hand traces an angle of 30° in 1h
1 h = 30° ⇒ 60 min = 30°
30°  1 °
⇒ 1 min = = 
60°  2
°
 1
Clearly, hour hand traces an angle of   in 1 min.
 2

(29)
30 Fundamentals of Reasoning

Key Notes :
l Between n and n + 10 'clock, the two hands of a clock
 60 
will coincide at  n  min past n.
 11 

l Between n and n + 10 'clock, the two hands of a clock


12
will be at (5 n ± 15) × min past n.
11
l Between n and n + 10 'clock, the hands of a clock will
be in opposite directions at :
12
l (5n − 30) × min past n, when n > 6
11
12
l (5n + 30) × min past n, when n < 6
11
l Between n and n + 10 'clock, the hands of a clock are x
12
min apart at (5n ± x ) × min past n. ‘ + ’ sign
11
indicates that the minute hand is ahead and ‘ – ’ sign
indicates that the hour hand is ahead.
l If the minute hand of a clock overtakes the hour hand
at an interval of x min of the correct time, then the
 720   60 × 24 
clock loses or gains by  − x   min. in a
 11  x 
day. If the result is ‘ + ’ then clock gains and if result is
‘ – ’, then clock loses.
SAMPLES EXAMPLES
1. When the minute hand covers a distance of 2 h 10 min, then
what is the angular distance covered by it :
(a) 690° (b) 420° (c) 310° (d) 780°
Ans. (c) 310°
Explanation : Angle traced by hour hand in
°
1
1 min =  
 2
Reasoning-Clock 31

∴ Angle traced by hour hand in


1 °
1 h =   × 60 = 30°
 2
Time period between 8 O'clock to 3 O'clock = 7 h
∴ Angle traced by hour hand in 7 h = 30° × 7 = 210°
2. How much angular distance will be covered by the minute hand
of a correct clock in a period of 1 h 40 min :
(a) 600° (b) 840° (c) 450° (d) 520°
Ans. (a) 600°
Explanation :
Angle traced by minute h and in 1 min = 6°
∴ Angle traced by minute h and in 1 h 40 min
= [1 × 60 + 40] × 6°
= 100 × 6°
= 600°
3. Find the angle between the two hand of clock at 4 :10.
(a) 61° (b) 68° (c) 55° (d) 65°
Ans. (d) 65°
Explanation :
1 °
Q Angle traced by hour hand per minute =  
 2
∴ Angle traced by hour hand in 4 h 10 min

= [( 4 × 60) + 10] ×
2

= 250 × = 125°
2
Q Angle traced by minute hand per minute = 6°
∴ Angle traced by minute hand in 10 min = 10 × 6°
= 60°
∴ Required angle = 125°− 60° = 65°
4. At what time between 2 and 3 O'clock will the hands of a clock
be together :
10 10
(a) 9 min past 2 (b) 12 min past 2
11 11
32 Fundamentals of Reasoning

10 10
(c) 11 min past 2 (d) 10 min. past 2
11 11
10
Ans. (d) 10 min. past 2
11
Explanation :
60 n 60 × 2
= min past 2
11 11
120
= min past 2
11
10
= 10 min past 2
11
5. At what time between 7 :15 and 8 :15, the hands of a clock will
coincide each other :
5 4
(a) 39 past 7 :15 (b) 39 past 8 :15
11 11
5 4
(c) 38 past 7 :15 (d) 39 past 7 :15
11 11
5
Ans. (a) 39 past
11
15 29
Explanation : When n = 715 . =7 =
60 4
60 × n 60 29 5
= × = 39 past 7 : 15
11 11 4 11
6. At what particular time, between 9 and 10 O'clock, both the
hands will be at a right angle to each other :
5 8
(a) 65 and 32 min past 9
11 11
2 2
(b) 65 and 32 min past 9
11 11
3 3
(c) 65 and 32 min past 9
11 11
1 1
(d) 65 and 32 min past 9
11 11
5 8
Ans. (a) 65 and 32 min past 9
11 11
Explanation :
12 12 12 60 × 12
( 5 n ± 15) × = ( 5 × 9 ± 15) × = ( 45 ± 15) × =
11 11 11 11
Reasoning-Clock 33

30 × 12 720
or =
11 11
360 5 8
or = 65 and 32 min past 9
11 11 11
7. In between 4 and 5 O'clock, when will both the hands of a clock
form a straight line :
10 6
(a) 10 min past 4 (b) 4 min past 4
11 11
6
(c) 54 min past 4 (d) 45 min past 4
11
6
Ans. (c) 54 min past 4
11
Explanation :
12 12
( 5n + 30) × min past n = [5 × 4 + 30] × min past 4
11 11
50 × 12
= min past 4
11
600
= min past 4
11
6
= 54 past 4
11
8. In how much time hands of a clock will be in the 30 min, space
apart when they are between 12 and 1 p.m. on Monday :
1080 360
(a) and min past 12
12 11
1080 300
(b) and min past 12
11 11
1080 300
(c) and min past 12
12 12
1080 300
(d) and nub past 12
12 12
1080 300
Ans. (d) and min past 12
12 12
Explanation :
12 12
( 5n ± x) × = ( 5 × 12 ± 30) ×
11 11
90 × 12 30 × 12
= and min past 12
11 11
34 Fundamentals of Reasoning

1080 360
= and min past 12
11 11
9. How many times do the hands of a clock coincide in a day :
(a) 20 (b) 22 (c) 24 (d) 21
Ans. (b) 22
Explanation : In every 12 h, both hands coincide 11 times.
(between 11 and O'clock there is a common position at 12
O'clock)
11 × 24
∴ In 24 h, both hands coincides = 22 times.
12
10. How many times the hand of a clock are at right angle in a day :
(a) 44 (b) 11 (c) 22 (d) 48
Ans. (c) 22
Explanation : In every 12 h, the two hands are at right angles
22 times.
(between 2 and 4 O'clock there is a common position at 3
O'clock and also between 8 and 10 O'clock there is a common
position at 9 O'clock)
22 × 14
∴ In 24 h, both the hands are at right angles times = 44
12
times
11. How many times in 24 h the hands of a clock are straight :
(a) 48 (b) 22 (c) 24 (d) 44
Ans. (d) 44
Explanation : In every 12 h, the two hands are in the same
straight line 22 times
22 × 24
∴ In 24 h, the two hands are in the same straight line
12
times = 44 times.
12. How many times in 24 h the hands of a clock are in straight line
but opposition in directions :
(a) 20 (b) 22 (c) 24 (d) 48
Ans. (b) 22
Explanation : In every 12 h, both hands are in opposite
direction 11 times.
(between 5 and 7 O'clock there is a common position at 6
O'clock)
Reasoning-Clock 35

11 × 24
∴ In 24 h, both hands are in opposite directions times
12
= 22 times
13. What is the angle traced by hour hand in 25 min :
(a) 150° (b) 75° (c) 12.5° (d) 50°
Ans. (c) 12.5°
Explanation :
1
Q Angle traced by hour hand in 1 min =  
 2
∴ Angle traced by hour hand in 25 min = 25 min

= 25 × = 12 . 5°
2
14. The minute hand of a clock overtakes the hour hand at interval
of 61 min. then the clock loses or gain by how much time :
63 3
(a) 104 min. (gain) (b) 104 min. (loss)
671 671
105 8
(c) 105 min. (gain) (d) 105 min. (gain)
671 671
105
Ans. (c) 105 min. (gain)
671
Explanation :
 720 − 61 ×  60 × 24 = 49 × 1440 = 70560
   
 11   61  11 61 671
105
105 min. (gain as sign is positive)
671
15. The minute hand of a clock overtakes the hour hand at
intervals of 58 min of the correct time. How much does a clock
gain or lose in a day :
25 25
(a) 185 min (gain) (b) 185 min (loss)
319 319
25 25
(c) 184 min (gain) (d) 185 min (loss)
319 319
25
Ans. (a) 185 min (gain)
319
Explanation :
720 60 × 24
Required result =  − x   min
 11  x 
Here, x = 58
36 Fundamentals of Reasoning

720 60 × 24
∴ Required result =  − 58   min
 11   58 
82 720 25
= × min = 185 min gain
11 29 319
(gain as sign is positive)
PRACTICE SET
1. When the minute hand covers a distance of 2 h 10 min, then
what is the angular distance covered by it :
(a) 690° (b) 420° (c) 310° (d) 780°
Ans. (d) 780°
2. What angle will be traced by the hands of a clock at 12 : 55 :
1° 1° 1° 1°
(a) 87 (b) 6 (c) 57 (d) 27
2 2 2 2

Ans. (a) 87
2
3. Find the angle between the two hands of clock at 4 : 10.
(a) 61° (b) 68° (c) 55° (d) 65°
Ans. (b) 68°
4. At what time between 4 and 5 0'clock will the hands of a clock
be at right angle :
3
(a) 30 min past 4 (b) 16 min past 4
4
2
(c) 38 min past 4 (d) 33 min past 4
11
2
Ans. (c) 38 min past 4
11
5. At what time between 5 and 6 are the hands of a clock
coincident :
3
(a) 22 min past 5 (b) 27 min past 5
11
8
(c) 30 min past 5 (d) 22 min past 5
11
3
Ans. (b) 27 min past 5
11
6. The minute hand of a clock overtakes the hour hand at intervals
of 76 min of the correct time. How much does a clock gain or
lose in a day :
169 169
(a) 199 min (loss) (b) 198 min (gain)
209 209
Reasoning-Clock 37

169 169
(c) 199 min (gain) (d) 198 min (gain)
209 209
169
Ans. (a) 198 min (loss)
209
7. At what time between 7 O'clock and 8 O'clock, will the hands of a
clock be in the same straight line but not together :
5 5
(a) 6 min past 7 (b) 5 min past 7
11 11
5 5
(c) 5 min past 7 (d) 4 min past 7
11 11
5
Ans. (c) 5 min past 7
11
8. At what time between 3 O'clock and 4 O'clock, will the hands of a
clock be 4 min apart :
7
(a) 20 min past 3 and 12 min past 3
11
8
(b) 20 min past 3 and 13 min past 3
11
8
(c) 20 min past 3 and 14 min past 3
11
8
(d) 20 min past 3 and 12 past 3
11
8
Ans. (d) 20 min past 3 and 12 past 3
11
9. The minute hand of a clock overtakes the hour hand at
intervals of 50 min of the correct time. How much does a clock
gain or lose in a day :
28 28
(a) 443 min (loss) (b) 443 min (gain)
55 55
28 28
(c) 445 min (gain) (d) 444 min (loss)
55 55
28
Ans. (c) 445 min (gain)
55
10. Find at what time between 8 and 9 O'clock will the hands of a
clock be in the same straight line but not together :
10
(a) 10 min past 8 (b) 50 min past 8
11
12 10
(c) 10 min past 8 (d) 10 min past 8
11 11
12
Ans. (c) 10 min past 8
11
qqq
6
CODED BINARY NUMBERS
DECIMAL NUMBERS
To know what binary number is, first we have to know
about decimal numbers. A decimal number is a positional
numeral system with 10 as the base and requires 10 different
numerals, the digits 0,1,2,3,4,5,6,7,8,9. In this scheme the
numerals used in denoting a number take different place
values depending upon position. In a base-10 system the
number 5379 represents the sum (5 × 10 3 ) + (3 × 10 2 )
+ ( 7 × 101 ) + (9 × 10° ).
Note :
We get the value of number in this case by multiplying
different digits of sequence by powers of 10 and adding. Here
this 10 is called base or radix.
BINARY NUMBERS
A binary number is a positional numeral system with two
as the base. The binary number system consists of two
different numerals, namely 0 (zero) and 1 (one). These can be
used to represent all other numbers like decimal numeral
system, binary numbers can do arithmetic operations like
addition, subtraction, multiplication and division. Binary
numbers can also be converted to numbers of decimal system.
Example of a binary number is 1101. Here 4 digits are
present in the sequence of digits : 1, 1, 0, 1. We get the value in
the following manner.
1101 = 1 × 2 3 + 1 × 2 2 + 0 × 21 + 1 × 2 0
= 8 + 4 + 0 + 1 = 13
Thus, the binary number represents thirteen in decimal
system. The value can be determined by multiplying
different digits of sequence by powers of 2 and adding.
(38)
Coded Binary Numbers 39

Converting Binary Number into Decimal number


Conventional method : In the above example, we see
that to get the value of any binary number, we use the
following rule :
1. The first digit from right is multiplied by 2° = 1
2. The second digit from right is multiplied by 21 = 2
3. The third digit from right is multiplied by 2 2 = 4
4. Similarly, n th digit from right is multiplied by 2 n − 1
5. Finally, all these are added.
QUICKER METHOD
Step I : Starting from right digit of given number, write
1, 2, 4, 8, 16, 32…… and so on below each digit as you proceed
towards left.
Step II : Ignore the numbers below 0 s. Add all numbers
below 1s.
1. Convert 1001 in decimal.
(a) 10 (b) 9 (c) 7 (d) 12
Ans. (b) 9
Explanation : We will use 1, 2, 4, 8 because here 4 digits are
present.
We get,
1 0 0 1
8 4 2 1

4 and 2 fall below the zeroes. We ignore them and add the
remaining. We get 8 + 1 = 9.
Converting Decimal Number into Binary Number
Decimal number can be converted to binary by method of
successive divisions. The number is divided by 2. Get the
remainder for binary digit and quotient becomes the next
dividend, which is again divided by 2. The process is repeated
until the quotient is equal to 0.
40 Fundamentals of Reasoning

2. Convert 17 into binary :


(a) 0001 (b) 10100 (c) 10010 (d) 10001
Ans. (d) 10001
Explanation : Divide 17 by 2 until quotient is 0
Remainder
17 1
8 0
4 0
2 0
1 0

So, the binary form of 17 is 10001.


Direction : Q. no. 3-6 Study the following information
and answer the questions given below :
In a certain code, the symbol for 0 is + and for 1 is #. There
are No. number or symbols for all other number greater than
1. The value of symbol for 1 doubles itself every time it shifts
one place to the left.
‘0’ is depicted as +
‘1’ is depicted as #
‘2’ is depicted as # +
‘3’ is depicted as # #
‘4’ is depicted as # + + and so on
3. Which of the following will represent 11 :
(a) # + # + + (b) # + # #
(c) # # + + (d) + # # +
(e) none of these
Ans. (b) # + # #
Explanation : To get equivalent of 11, we use the method of
successive division by 2.

11 1
Coded Binary Numbers 41

5 1
2 0
1

So the binary form will be 10111. Replacing 1 and 0 by #


and + we will get # + # #.
4. Which of the following will represent 8 :
(a) # # + + (b) + # # +
(c) # # + + (d) + + # #
(e) none of these
Ans. (e) none of these
Explanation : for 8, we have

8 0
4 0
2 0
1

i. e., 1000 or # + + +. Correct choice is (e)


5. Which of the following number will be represented by # + + + # :
(a) 17 (b) 31 (c) 22 (d) 14
(e) none of these
Ans. (a) 17
Explanation : # + + + # = 10001
= 1 × 2 4 + 0 × 2 3 + 0 × 2 2 + 0 × 21 + 1 × 2 0
= 16 + 1 = 17
6. Which of the following will be represented by # # + # :
(a) 11 (b) 13
(c) 12 (d) 8
(e) none of these
Ans. (e) none of these
Explanation :
# # + # = 1101 = 1 × 2 3 + 1 × 2 2 + 0 × 21 + 1 × 2 0 = 15
42 Fundamentals of Reasoning

SOLVED EXAMPLES
Directions (Q.7-10) : In a certain code, the symbol for 0
is # and for 1 is = . There is no symbol for rest of the numbers.
Numbers greater than 1 are needed to be written using the
two given symbols. Left shifting of 1 doubles its value each
time. Study the following example.
0 is written as #
1 is written as =
2 is written as = #
3 is written as = =
4 is written as = # # and so on
7. Which of the following number will be represented by = # # # =
(a) 31 (b) 25 (c) 29 (d) 17
Ans. (d) 17
Explanation : = # # # = can be represented as 10001 it means
16 + 0 + 0 + 0 + 1 = 17.
8. Which of the following will represent 2 × 2 + 3 × 2 + 3 × 3 + 2 × 2
:
(a) # = = = # (b) = # = = = (c) = = = = = (d) # # # # #
Ans. (b) = # = = =
Explanation : Here the solution is 23. After dividing this
number with 2, we will get remainder as 101111. This means
the symbol will be = # = = =.
9. Which of the following will represent 15 * 6 ÷ 2 + 1 :
(a) = # = # # = (b) = = = = #
(c) = # = = = # (d) = = = = # #
Ans. (c) = # = = = #
Explanation : 15 × 6 ÷ 2 + 1 = 15 × 3 + 1 = 45 + 1 = 46
After dividing 46 by 2, we get 101110.
This means the symbol will be = # = = = #.
10. Which of the following will represent = # # = # # :
(a) 36 (b) 68 (c) 32 (d) 20
Ans. (a) 36
Explanation : = # # # # # can be represented as 100100. This
is nothing but 32.
Coded Binary Numbers 43

Directions (Q. 11-15) : In a certain code, the symbol for


0 is # and for 1 is # and for 1 is = . There is no symbol for rest of
the numbers. Numbers greater than 1 are needed to be
written using the two given symbols. Left shifting of 1 doubles
its value each time. Study the following example.
0 is written as #
1 is written as =
2 is written as = #
3 is written as = =
4 is written as = # # and so on
11. Which of the following numbers will be represent 29 :
(a) = # = = = (b) = = = = # (c) = = = # # (d) = = = # =
Ans. (d) = = = # =
Explanation : The number 29 can be represented as 11101. It
means = = = # =
12. Which of the following numbers will be represented by = # = = #
:
(a) 22 (b) 9 (c) 20 (d) 21
Ans. (a) 22
Explanation : The number is = # = = # = 10110 2 = 2210 :
13. Which of the following numbers will be represented by
(= # # # + = # ÷ # = # ) :
(a) 5 (b) 8 (c) 9 (d) 6
Ans. (c) 9
Explanation : = # # # + = # ÷ # = #
⇒ 1000 + 10 ÷ 010
⇒ 8+ 2÷ 2
⇒ 8+1 = 9
14. Which of the following will represent the multiplication of = = =
and = = # :
(a) = # # # # = (b) = = = = #
(c) = # = # = # (d) = # # # = =
Ans. (c) = # = # = #
15. Which of the following will represent the HCF of 18, 36 and 48 :
(a) = # = (b) = = #
(c) = # # (d) None of these
44 Fundamentals of Reasoning

Ans. (b) = = #
Explanation : HCF of 18, 36 and 48 is 6.
The number 6 is represented as 110 which means = = #.
PRACTICE SET
1. convert 293 to binary :
(a) 100111101
(b) 101100101
(c) 100100101
(d) 101100101
Ans. (c) 100100101
2. Convert 1100010 to decimal :
(a) 49 (b) 98 (c) 54 (d) 113
Ans. (b) 98
Directions (Q.3-10) : Study the following information
and answer the questions that follow :
In a certain code, the symbol for 0 is + and for 1 is ∆.
There are no symbols for all other numbers. The numbers
greater than 1 are to be written only by using two symbols for
all other numbers. The numbers greater than 1 are to be
written only by using two symbols given above. The value of
the symbol 1 doubles itself every time it shifts one place to the
left study the following example.
0 is written as *
1 is written as ∆
2 is written as ∆ *
3 is written as ∆ ∆.
3. Which of the following represents 22 :
(a) ∆ * ∆ * * (b) ∆ * ∆ ∆ * (c) ∆ * ∆ ∆ ∆ (d) ∆ ∆ * ∆ *
Ans. ∆ * ∆ ∆ *
4. Which of the following numbers will be represented by
∆*∆∆∆*
(a) 46 (b) 29 (c) 23 (d) 41
Ans. (a) 46
5. Which of the following will represent 17 * 18 ÷ 9 − 8 :
(a) ∆ * ∆ ∆ ∆ (b) ∆ ∆ * ∆ *
Coded Binary Numbers 45

(c) ∆ * ∆ * ∆ * (d) ∆ * * * *
Ans. ∆ ∆ * ∆ *
6. What of the value of ∆ ∆ ∆∗ ∗ divided by ∆ ∆ ∆ :
(a) ∆ ∆ * (b) ∆ * ∆ (c) ∆ ∆ ∆ (d) ∆ * *
Ans. (d) ∆ * *
7. Which of the following will represent the multiplication of
∆ * *∆ and ∆ ∆ * :
(a) ∆ ∆∗∗ ∗ ∗ (b) ∆ ∆ ∆ ∆ ∗ ∗
(c) ∆ * ∆ ∆ * ∆ (d) ∆ ∆ * ∆ ∆ *
Ans. ∆ ∆ * ∆ ∆ *
8. Which of the following will represent the Lon of 9, 18 and 36 :
(a) ∆ ∆ * ∆ ∆ * (b) ∆ * *∆ * *
(c) ∆ * ∆ ∆ * ∆ (c) None of these
Ans. (b) ∆ * *∆ * *
9. Which of the following is the HCF of ∆ * *, ∆ ∆ * and ∆ * ∆ * :
(a) ∆ * * (b) * *
(c) ∆ ∆ (d) None of these
Ans. (d) None of these
10. Which of the following will be represented by ∆ ∆ ∆ ∆ ∆ :
(a) 31 (b) ** (c) 30 (d) 63
Ans. (a) 31
qqq
7
CODED RELATIONSHIP
INTRODUCTION
Coded blood relationship questions, certain symbols and
codes such as +, –, ×, +, $, *, ∆ etc., are used to present
information in coded form. The candidate is required to
decode the information with the help. of symbols to be used
for making family diagram and then find out the relationship
between the two required persons.
1. Given, M % N means M is the son of N, M @ N means M is the
sister of N and M $ N means M is the father of N. which of the
following shows the relation that c is the granddaughter of E :
(a) C % B $ F $ E(b) C@B%F%E
(c) E % B $ F $ C(d) B$F$E%C
Ans. (b) C @ B % F % E
Explanation : C @ B → C is the sister of B
B % F → B is the son of F.
F % E → F is the son of E
E
c is grand
son daughter of E
F
son
B C
2. If A + B means A is the brother of B, A − B means A is the sister
of B and A × B means A is the father of B. which of the following
means that C is the son of M :
(a) N + M − F × C (b) M − N × C + F
(c) F − C + N × M (d) M × N − C + F
M
M is father of C
Father

N C F
Sister Brother

(46)
Coded Relationship 47

Directions (Q. 3-4) : Read the following information and


answer the questions given below :
l ‘ P # Q ’ means ‘Q is father of P ’
l ‘ P * Q ’ means ‘Q is mother of P ’
l ‘P = Q ’ means ‘Q is brother of P ’
l ‘ P $ Q ’ means ‘ Q is sister of P ’
3. Which of the following means ‘X is the grand mother of Y ’ :
(a) Z $ X * K # L £ Y (b) Y £ L # K * X $ Z
(c) Y * K # L £ Y (d) can't be determined
(e) None of these
Ans. (b) Y £ L # K * X $ Z
Explanation : Y £ L → is brother of Y
L # K → K is father of L
K * X → X is mother of K
X $ Z → Z is sister of X.
sister
Z Z
mother

K
father

L Y
brother

Hence X is the grandmother of Y .


4. Which of the following means ‘A is the nephew of D ’ :
(a) C £ D # B £ A (b) D £ C # B£ A
(c) A £ B # D £ C (d) Can't be determined
(e) None of these
Ans. (e) None of these
Explanation : Choice 2 is wrong because gender of ‘A’ is not
known. Choices 1 and 3 are wrong as A must be before D. So, the
answer will be none of these.
SOLVED EXAMPLES
Directions (5-7) : Read the following information
carefully and answer the questions given below :
A + X means A is father of X
A − X means A is mother of X
48 Fundamentals of Reasoning

A * X means A is son of X
A / X means A is daughter of X
5. How is P related to R in P + Q − R. :
(a) Grandfather (b) Father
(c) Grandmother (d) Mother
Ans. (a) Grandfather
Explanation : P + Q → P is father of Q ;
Q − R → Q is mother of R
P
father
Q P is grandfather of R
mother
R
6. In A * B / C − D how is B related to D :
(a) Grand mother (b) Sister
(c) Mother (d) Niece
Ans. (b) Sister
Explanation : C − D → C is mother of D
B / C → B is daughter of C
∴ B is sister of D.
7. In M + N + C , how is C related to M :
(a) Daughter (b) Grandson
(c) Granddaughter (d) Son
Ans. (b)
Explanation :
N + C → N is father of C
∴ M + N → M is father of N
Directions (Q. no. 8 – 13) : Read the following
information carefully to answer the questions given below it.
I. ‘ A + B ’ means that ‘A is the father of B ’.
II. ‘ A − B ’ means that ‘A is the wife of B ’.
III. ‘ A × B ’ means that ‘A is the brother of B ’.
IV. ‘A + B ’ means that ‘A is the daughter of B ’.
Coded Relationship 49

8. If it is given ‘ P − R + Q ’, then which of the following statements


is true :
(a) P is the mother of Q (b) P is the sister of Q
(c) Q is the daughter of p (d) P is the aunt of Q
Ans. (a) P is the mother of Q
Explanation : P − R → P is wife of R
R + Q → R is father of Q
P R
wife
Father
P is the
mother of Q Q

Hence, P is the mother of Q.


9. If ‘ P + R ÷ Q ’, then which of the following is true :
(a) P is the brother of Q (b) P is the father of Q
(c) p is the son of Q (d) P is the husband of Q
Ans. (d) P is the husband of Q
Explanation : P + R → P is wife of R
R × Q → R is brother of Q
Married
couple
P Q

Father

R
Therefore, P is the husband of Q.
10. If it is given P × R ÷ Q , then which of the following is true :
(a) P is the uncle of Q (b) P is the father of Q
(c) P is the son of Q (d) P is the brother of Q
Ans. (c) P is the son of Q
Explanation :
P × R → P is brother of R
R ÷ Q → R is daughter of Q
P is the son Q
of Q

Daughter

P R
Brother
50 Fundamentals of Reasoning

Hence, P is the son of Q.


11. If ‘ P − R × Q ’, then which of the following is true :
(a) P is the sister of Q (b) Q is the son of P
(c) Q is the husband of P (d) P is the sister-in-law of Q
Ans. (d) P is the sister-in-law of Q
Explanation :
P − R → P is wife of R
R × Q → R is brother of Q
wife Brother
P R Q

P is sister-in-law of Q

Therefore, P is sister-in-law of Q.
12. If it is given ‘ P × R × Q ’ , then which of the following is true :
(a) P is the father of Q
(b) P is the brother-in-law of Q
(c) P is the brother of Q
(d) P is the uncle of Q
Ans. (b) P is the brother-in-law of Q
Explanation :
P × R → P is brother of R
R − Q → R is wife of Q

Brother wife
P R Q

P is brother-in-law of Q

Therefore, P is the brother-in-law of Q.


13. If it is given ‘ P ÷ R + S + Q ’, then which of the following is true :
(a) P is the aunt of Q (b) Q is the aunt of P
(c) P is the daughter of Q (d) P is the mother of Q
Ans. (a) P is the aunt of Q
Explanation : P ÷ R → P is daughter of R
R + S → R us father of S
S + Q → S is father of Q
Coded Relationship 51

Daughter Sistrt
R P S

P is the
aunt of Q

Hence, P is the aunt of Q.


14. If ‘ P − Q ’, means Q is son of P, ‘ P × Q ’ means P is brother of Q,
‘ P ÷ Q ’ means Q is sister of P and ‘ P + Q ’ means P is mother of Q,
then which of the following is definitely true about
‘ N × K − M ÷ L’ :
(a)K is father of L and M. his son and daughter respectively
(b) L is daughter of K and niece of uncle N
(c) M is uncle of K's brother N
(d) K is father of L and M
Ans. (b) L is daughter of K and niece of uncle N
Explanation : N Brother K
N × K → N is brother of K
K − M → M is son of K Son

M ÷ L → L is sister of M
15. If P $ Q means P is the brother of Q , M Sister
L
P # Q means P is the mother of Q,
P * Q means P is the daughter of Q, in
A # B $ C * D , who is the father :
(a) B (b) C (c) D (d) Inadequate Data
Ans. (c)D
Explanation : A # B → A is mother of B
B $ C → B is brother of C
C * D → C is daughter of D
(Parents)
A A
Mother Daughter

A A
Brother

Hence D is the father


52 Fundamentals of Reasoning

PRACTICE SET-1
1. If A $ B means A is the brother of B, B * C means B is son of C,
C @ D means C is wife of D and A # D means A is the son of D, how
is C related to A :
(a) Maternal grandmother (b) Mother
(c) Aunt (d) Maternal aunt
Ans. (b) Mother
Directions (Q. no. 2-3) Read the following information
carefully and answer the questions given below.
‘ P + Q ’ means that P is the son of Q,‘ P − Q ’ means that P
is the wife of Q.‘ P × Q ’ means that P is the brother of Q ,‘ P + Q ’
means that P is the mother of Q and ‘ P = Q ’ means that P is
the sister of Q.
2. Then, what does ‘ X + Y + Z ’ mean :
(a) Z is the father of Y (b) Z is the son of Y
(c) Z is the uncle of Y (d) Z is the brother of Y
Ans. (a) Z is the father of Y
3. If A × B ÷ C given, which of the following statement is true :
(a) A is the nephew of C (b) A is the father of C
(c) A is the maternal uncle of C
(d) A is the brother of C
Ans. (c) A is the maternal uncle of C
Directions (Q. No. 4-5) : Read the following information
carefully and answer the questions that follow.
I. P + Q means P is the father of Q.
II. P − Q means P is the wife of Q.
III. P × Q means P is the brother of Q.
IV. P ÷ Q means P is the daughter of Q.
4. If A − C + V , the which of the following statements is true :
(a) A is the aunt of B (b) A is the daughter of B
(c) A is the mother of B (d) B is the aunt of A
Ans. A is the mother of B
5. If A ÷ C + D + B, then which of the following statement is true :
(a) A is the daughter of B (b) A is the aunt of B
(c) A is the mother of B (d) B is the aunt of A
Ans.
Coded Relationship 53

Directions (Q. no. 6-8) : Read the following information


carefully and answer the questions given below:
I. ‘ P × Q ’ means ‘ P is the father of Q’.
II. ‘ P − Q ’ means ‘ P is the mother of Q ’.
III. ‘ P + Q ’ means ‘ P is the mother of Q’
IV. ‘ P + Q ’ means ‘ P is the brother of Q’.
6. In the expression B + D × M ÷ N , how is M related to B :
(a) Grandson (b) Son
(c) Granddaughter (d) Granddaughter or Grandson
Ans. (a) Grandson
7. Which of the following represent ‘J is the son of F :
(a) M ÷ K × T − R (b) M − J + R − N
(c) R − M × T ÷ W (d) Can not be determined
(e) None of these
Ans. M − J + R − N
8. Which of the following represent ‘ J is the son of F :
(a) J + R − T × F (b) J ÷M−N ×F
(c) J ÷ R − T × F (d) Can not be determined
(e) None of these
Ans. (e) None of these
9. If A * B means A is the sister of B, A $ B means B is the mother of
A, A + B means A is the brother of B, A = B means B is the father
of A. which of the following means M is the maternal uncle of N :
(a) M = P + Q * N (b) N + P = Q * M
(c) N * P $ Q * M (d) None of these
Ans. (d) None of these
10. If A + B means A is son of B , A − B means A is wife of B, A ÷ B
means A is mother of B, A = B means A is sister of B. what does
‘ X = Y ÷ Z ’ mean :
(a) X is sister of Z (b) X is aunt of Z
(c) Z is niece of X (d) Z is daughter if X
Ans. (b) X is aunt of Z
qqq
INTRODUCTION
A calendar is a systematic arrangement of day, week and
month in a defined pattern with which we can easily
recognize the required date, month or week of a particular
day.
Different types of questions covered in this chapter are as
follows :
l Finding the day on a particular date when reference
day is given.
l Finding the day on a particular date when reference
day is not given.
l Finding a week day on the basis of another week day.
Gregarian calender is the world's most popular calendar.
It also happens to be the Indian National Calendar and was
adopted by India on 22nd March, 1957.
Leap year
l A leap year is a year which has 366 days (52 weeks +
2 days) Such years are exactly divisible by 4. e. g.,
2004, 2008, 2012, 2016, etc.
l Leap years in the form of a century are exactly
divisible by 400. e. g., 400, 800, 1200, 1600, 2000,
2400, etc.
Ordinary year
l An ordinary year is a year which has 365 days (52
weeks + 1 odd day). Such years are not divisible by 4
e. g., 2001, 2002, 2003, 2005, 2006, 2007, 2009, 2010,
etc.

(54)
Reasoning-Calendar 55

l Ordinary years in the form of century are not exactly


divisible by 400. e. g., 100, 200, 500, 600, 700 and 900
etc.
To calculate odd number of days in a given period, follow
the rule given below :
 Period 
Divide the period by 7  i. e., 
 7 
Period
l If leaves no remainder, then there is zero odd
7
days.
Period
l If leaves any remainder, then that remainder
7
will be number of odd days.
Counting of Odd Days
1. Number of days in an ordinary year = 365
days
7) 365 ( 52
35
15
14
1 odd day or
365 = 52 × 7 + 1 = 1 odd day

So, an ordinary year has 1 odd day.


2. Number of days in a leap year = 366 days.
7) 366 ( 52
35
16
14
1 odd day or
366 = 52 × 7 + 2 = 2 odd days

So, a leap year has 2 odd days.


3. 100 yrs = 76 ordinary years + 24 leap years
∴ Number of odd days in 100 yrs
= 76 odd days of 76 ordinary years + (24 × 2) odd
days of 24 leap years
= 76 odd days + 48 odd days = 124 odd days
56 Fundamentals of Reasoning

7) 124 ( 17
7
54
49
5 odd day or

Odd days in 100 yrs = 5 odd days


l Odd days in 200 yrs = (Odd days in 100 yrs)
× 2 = 5 × 2 = 10 days = 1 week + 3 days = 3 odd days
l Odd days in 300 yrs = (Odd days in 100 yrs)
× 3 = 5 × 3 = 15 days = 2 weeks + 1 day = 1 odd day
l Odd days for 400 yrs = (Odd days in 100 yrs) × 4 + 1
day = (5 × 4 + 1) days = 21days = 3 weeks = 0 odd day
As 400 yrs is a leap year, therefore 1 more day has been
taken.
A leap year calendar repeats in 28 yr while the calendar
of an ordinary year repeats after 6 or 11 yr.
Number of odd days in each month of the year are as
follows :
Months Odd days
January 3
February O (Ordinary year), 1
(leap year)
March 3
April 2
May 3
June 2
July 3
August 3
September 2
October 3
November 2
December 3
Reasoning-Calendar 57

Types of Problems
Type I : To find the day of a week by the help of number
of odd days, when reference day is given.
Working Rule
l Find the net number of odd days for period between
the reference date and given date. The day of the
week on the particular date is equal number of net
odd ahead of reference day, if the reference day was
before this day.
SOLVED EXAMPLES
1. January 5, 1991 was a Saturday. What day of the week was on
March 4, 1992 :
(a) Friday (b) Monday
(c) Wednesday (d) Sunday
Ans. (c) Wednesday
Explanation :
Number of odd days in 1991 for
( 365 − 5 = 360) = 3 ( 360 = 51 × 7 + 3)
Number of odd days in January 1992 = 3
Number of odd days in February 1992 = 1 (Leap year)
Number of odd days from 1st to 4th March, 1992 = 4
∴ Total odd days = 3 + 3 + 1 + 4 = 11 = 1 week + 4 days = 4 odd
days
∴ Required day = Saturday + 4 = Wednesday
2. Today is 21 st August. The day of the week is Monday. This is a
leap year. What will be the day of the week on this day after
three years :
(a) Monday (b) Wednesday
(c) Friday (d) Thursday
Ans. (d) Thursday
Explanation : Since this is a leap year, so one of the next 3
years is a leap year. Hence the number of odd days = 3. So, the
day of the week will be 3 days beyond Monday i. e., it will be
Thursday.
Type II : To find the day on a particular by the help of the
number of odd days, when reference day is not given.
58 Fundamentals of Reasoning

If odd day − 0, then required day = Sunday


If odd day − 1, then required day = Monday
If odd day − 2, then required day = Tuesday
If odd day − 3, then required day = Wednesday
If odd day − 4, then required day = Thursday
If odd day − 5, then required day = Friday
If odd day − 6, then required day = Saturday
If odd day − 7, then required day =
3. Find the day of the week on 27 th dec 1985.
(a) Friday (b) Sunday (c) Monday (d) Thursday
Ans. (a) Friday
Explanation :
Odd days for 1600 years = 0
Odd days for 300 year = 1
Odd days for 84 year = ( 21 × 2 + 63 × 1)
= 105 [Since there are 21
leap years and 63 ordinary years in 84 years]
Odd days for period for 1 st Jan 1985 to 27 th Dec 1985 =
Jan Feb Mar Apr May June July Aug Sep Oct Nov Dec
3+ 0+ 3+ 2+ 3+ 2+ 3+ 3+ 2+ 3+ 2+ 6
= 32
∴ Total odd days = 0 + 1 + 105 + 32 = 138
= 19 weeks +5 days
= 5 odd days
∴ Required Day = Friday
4. Find the day of the week on 11 th April 1980.
(a) Sunday (b) Tuesday (c) Monday (d) Friday
Ans. (d) Friday
Explanation : Odd days for 1600 years = 0
Odd days for 300 years = 1
Odd days for 79 years = (19 × 2 + 60 × 1) = 79
[19 leap years + 60 ordinary years]
th
Odd days for 1 Jan 1980 to 11 th April 1980
Reasoning-Calendar 59

= 3 + 1 + 3 + 4 = 11
Total odd days = 1 + 98 + 11 = 110
= 15 weeks + 5 days = 5 days
∴ Required day = Friday
5. Today is Friday, what will be the day after 59 days :
(a) Monday (b) Tuesday (c) Sunday (d) Wednesday
Ans. (a) Monday
Explanation : Every day of the week is repeated after 7 days.
Hence, it will be Friday after 56 days because 8 × 7 = 56 . After
this, 3 more days are left which means it is Monday.
6. Today is Monday, what will be the day after 32 days :
(a) Sunday (b) Tuesday (c) Monday (d) Friday
Ans. (d) Friday
Explanation : Every day of the week is repeated after 7 days.
Hence, it will be Monday after 28 days because 4 × 7 = 28. After
this, 4 more days are left which means it is Friday, hence option
D is correct.
7. Today is Wednesday, what will be the day after 91 days :
(a) Sunday (b) Wednesday
(c) Monday (d) Tuesday
Ans. (b) Wednesday
Explanation : Every day of the week is repeated after 7 days.
Hence, it will be Wednesday after 91 days because 13 × 7 = 91.
8. Today is Ram's birthday which falls on Tuesday, Gopal's
birthday falls after 79 days after Ram's birthday. On which day
Gopal's birthday falls :
(a) Monday (b) Thursday (c) Sunday (d) Wednesday
Ans. (b) Thursday
Explanation : Every day of the week is repeated after 7 days.
79 days = ( 7 × 11 + 2) days = 11 weeks + 2days
∴ No of odd days = 2
9. If day before yesterday was Thursday, then when will b e the
Monday :
(a) Two days after days (b) Today
(c) Day after tomorrow (d) Tomorrow
Ans. (c) Day after tomorrow
Explanation :
A day before yesterday = Thursday
60 Fundamentals of Reasoning

Yesterday = Thursday + 1 = Friday


Today = Friday +1 = Saturday
∴ Sunday = Saturday +1 = Tomorrow
10. Which year will have the same calendar as that of 2016 :
(a) 2020 (b) 2040 (c) 2044 (d) 2036
Ans. (c) 2044
Explanation : Since 2016 is a leap year.
∴ 2016 + 28 = 2044
So, year 2044 will have the same calendar as that of 2016.
PRACTICE SET
1. Today is Thursday. The day after 38 days will be :
(a) Sunday (b) Tuesday (c) Monday (d) Wednesday
Ans. Sunday
2. What was the day on 31st October, 1984 :
(a) Friday (b) Thursday (c) Sunday (d) Saturday
Ans. (c) Sunday
3. Which of the given years will have the same calendar as that of
year 2003 :
(a) 2013 (b) 2016 (c) 2015 (d) 2016
Ans. (b) 2016
4. Find the number of days from 26th January, 2011 to 23rd
September, 2011 (both days are included).
(a) 219 (b) 241 (c) 238 (d) 251
Ans. 241
5. If Republic day was celebrated in 1996 on Friday, on which day
in 2000 Independence day was celebrated :
(a) Monday (b) Tuesday
(c) Wednesday (d) Saturday
Ans. (b) Saturday
6. If 15th August, 2011 was Tuesday, then what day of the week
was it on 17th September, 2011 :
(a) Friday (b) Sunday (c) Saturday (d) Thursday
Ans. (b) Sunday
7. What was the day of the week on 28th May, 2006 :
(a) Sunday (b) Friday (c) Thursday (d) Saturday
Ans. Sunday
Reasoning-Calendar 61

8. Ashu was born on August 19, 1992 what day of the week was
the born :
(a) Monday (b) Sunday (c) Tuesday (d) Wednesday
Ans. (d) Wednesday
9. If a day before yesterday was Thursday, then when will Sunday
fall :
(a) Today (b) A day after tomorrow
(c) 2nd day after today (d) Tomorrow
Ans. (b)

qqq
9
ASSERTION AND REASON
INTRODUCTION
Assertion is a strong and forceful statement or claim
made in regard with a thing, element for its use and effects.
Reason means a fact, event or statement that provides an
explanation to the assertion. In this chapter, we will be given
an assertion followed by a reason to support it. It is required
to analyse whether the reason is an optimum and correct
explanation of the assertion. Some times both assertion and
reason are correctly stated facts but the reason does not
correctly explain the assertion. So, different possibilities can
exist between these two statements and accordingly the
correct answer is marked from the given alternatives.
Assertion is denoted as ‘‘A’’ in the option in the short form
and similarly, reason will be shown as ‘‘R’’ in the answering
options.
1. Assertion A : Fish possess a streamlined today.
Reason R : It helps the fish to swim more efficiently and with
more speed.
(a) Both A and R are true and R is the correct explanation of
A
(b) Both A and R are true but R is not the correct explanation
of A.
(c) A is true but R is false.
(d) Both A and R are false.
Ans. (a) Both A and R are true and R is the correct
explanation of A.
Explanation : Here our first step will be to judge the truth of
both the sentences. The assertion is correct as fish is living in
water, so its body is designed in a streamline manner. Coming
to the reason part, this is also correct as the streamline
structure of the fish helps it to swim more efficiently and with

(62)
Assertion and Reaction 63

more speed. Now, it is time to judge whether the reason gives a


true explanation of the assertion or not. Yes it gives a true
explanation of the assertion. Hence we can say that the
assertion and reason both are correct and reason is the correct
explanation of assertion. Hence our answer option will be A .
From first, second, and third sentences, we can conclude the
meaning of mutation orientation.
2. Assertion A : If the weather is hot and humid, we feel more
comfortable.
Reason R : If the climate is humid, th evaporation of the sweet
is faster.
(a) Both A and R are true and R is the correct explanation of
A.
(b) Both A and R are true but R is NOT the correct
explanation of A.
(c) A is true but R is false.
(d) Both A and R are true.
Ans. (e) Both A and R are false
Explanation : The assertion is not correct as we all know that
when the weather is humid, we do not feel comfortable. Hence
option A or B or C cannot be our answer. Now, let's come to the
reason part. This is also not correct as it takes longer time for
the sweat to evaporate if the weather is humid.
Hence we know from the above discussion that both assertion
and reason are incorrect. Therefore option E will be our
answer.
Directions (Q. no. 3-10) : In each of the following
questions, there are two statements Labeled as Assertion (A)
and Reason (R).
Give Answer
(a) Both A and R are true and R is correct explanation of A.
(b) Both A and R re true but R is not the correct explanation
of A.
(c) A is true and R is false
(d) A is false and R is true
(d) Both A and R are false.
3. Assertion (A) : Increase in carbon dioxide would melt polar
ice.
64 Fundamentals of Reasoning

Reason (R ) : Global temperature would rise.


Ans. (a)
Explanation : The carbon dioxide envelope in Earth's
atmosphere traps the heat. With increase in the proportion of
carbon dioxide, the global temperature would rise, thus
causing the polar ice to melt.
4. Assertion (A) : Earth is the only planet known to have life.
Reason (R ) : Earth has an atmosphere which is a mixture of
oxygen, nitrogen and carbon dioxide.
Ans. (b)
Explanation : Earth has presence of water which is the
sustainer of life on it.
5. Assertion (A) : We prefer to wear white clothes in winter.
Reason (R ) : White clothes are good reflectors of heat.
Ans. (d)
Explanation : We prefer to wear dark clothes in winter as they
absorb the heat and keep the body warm. However, white
clothes are good reflectors of heat and are worn in summer.
6. Assertion (A) : IIT students are very
Reason (R ) : IIT students go high package salaries.
Ans. (b)
Explanation : Here both A and R are true but R is not the
correct explanation of A.
7. Assertion (A) : Software companies are not interested in
taking mechanical students in their company.
Reason (R ) : Mechanical students are generally lazy.
Ans. (c)
Explanation : R is false because we can't say that mechanical
students are lazy or not but A is true.
8. Assertion : India's rank in FIFA is below 50 :
Reason (R ) : Aditi Singh Chauhan became the first Indian
woman footballer to be singed by a top English club.
Ans. (d)
Explanation : Here A is false but R is true.
9. Assertion : TATA steel has opened another industry in
Odisha.
Reason (R ) : Odisha is rich minerals.
Assertion and Reaction 65

Ans. (a)
Explanation : Odisha is rich in minerals so option A is correct.
10. Assertion : Regular blood pressure check-up can develop your
health.
Reason : Increase in blood pressure can be silent killer for your
health.
Ans. (a)
Explanation : Blood pressure is a silent killer so regular
check-up can be beneficial.
PRACTICE SET
Directions (Q. no. 1-10) : Each of these questions has
an Assertion ( A ) and a Reason ( R).
Give answer
(a) Both A and R are true and R is the correct explanation of
A.
(b) Both A and R are true but R is not the correct explanation
of A.
(c) A is true but R is false.
(d) A is false but R is true.
(d) Both A and R are false.
1. Assertion (A) : Bulb filament is made of titanium.
Reason (R ) : The filament should have low melting point.
Ans. (e)
2. Assertion (A) : Shimla is colder than Delhi.
Reason (R ) : Shimla is at higher altitude as compared to Delhi.
Ans. (a)
3. Assertion (A) : Uttar Pradesh is called the 'Sugar Bowl' of
India.
Reason (R ) : Uttar Pradesh is the leading sugarcane producer.
Ans. (a)
4. Assertion (A) : Forest cutting is undesirable from the point of
view of soil erosion.
Reason (R ) : Cutting of forests reduces the interception of rain
water.
Ans. (a)
66 Fundamentals of Reasoning

5. Assertion (A) : There is rainbow in the sky only after rains.


Reason (R ) : Water drops suspended in the air break up Sun's
rays into seven colours.
Ans. (a)
6. Assertion (A) : Leaves of plants are green.
Reason (R ) : Plants contain chromoplasts, the green pigment.
Ans. (c)
7. Assertion (A) : India's 'Republic Day' falls on 26th January.
Reason (R ) : Constitution of India, declaring India as a
'Republic', came into force on 26th January 1950.
Ans. (a)
8. Assertion (A) : Mahatma Gandhi is considered as 'Father of
the nation' in India.
Reason (R ) : Mahatma Gandhi was the first Governor General
of Independent India.
Ans. (c)
9. Assertion (A) : In India 2nd October is observed as 'Martyr's
Day'.
Reason (R ) : Pt Nehru was the first Prime Minister of India.
Ans. (d)
10. Assertion (A) : Good performance at work causes satisfaction.
Reason (R ) : Job satisfaction results in good performance.
Ans. (c)
qqq
10
REASONING-DICE AND CUBE
INTRODUCTION
Die/Dice is /are a three-dimensional figure with each of
its six sides/faces showing different numbers/letters/colours
etc. It has 8 corners and 12 edges.
There are 6 faces in the dice and every face has a opposite
face. If we see any dice from any angle, then we can see a
maximum of only three faces of it.

1
1
2 3 4 5
4 5 2 3
6
6
Invisible faces Visible faces

1 is opposite to 6. 6 is opposite to 1. 2 is opposite to 5.


5 is opposite to 2. 3 is opposite to 4. 4 is opposite to 3.
Types of Dice :
Dice having digits from 1 to 6 on its surfaces can be
divided into two types :
1. Standard Dice
When, the sum of digits on opposite faces is equal to 7,
then the dice is called standard dice.
Let us see

1
2
3
4 5

(67)
68 Fundamentals of Reasoning

Sum of the opposite faces


1+ 6 = 7 4+ 3 = 7 2+ 5 = 7 5+ 2 = 7
3+ 4 = 7 6+1 = 7
Note :
If sum of any two adjacent faces of dice is not equal to 7,
this is called as a standard dice.
2. General Dice
When the sum of digits on opposite faces is not equal to 7,
then the dice is called general dice.
Let us see

3 4
5

Sum of the opposite faces


1+ 3 = 4 4+ 5 = 9 2+ 6 = 8 5+ 4 = 9
3+1 = 4 6+ 2 = 8
Note :
If the sum of digits of adjacent faces of a dice is equal to 7,
this is called a general dice.
The questions asked on dice and cube may be of the
following types ;
Type I : Finding digits on the opposite face of any
particular face.
1. Find the digit at the face opposite to the face having digit 2 in
the dice given below:

6
2 6

(a) 3 (b) 4 (c) 6 (d) 2


Ans. (c)
Explanation : According to the question.
5+1 = 6
Reasoning-Dice and Cube 69

1+ 3 = 4
3+ 5 = 8
Clearly, it is a standard dice.
∴ Digit at the face opposite to 1 = 7 − 1 = 6.
2. Find the digit at the face opposite to the face having digit 6 in th
given dice :
3
5 6

(a) 5 (b)6 (c) 1/5/6 (d) 1/6


Ans. (d) 1/6
Explanation : According to the question,
4+ 3 = 7
4+ 2 = 6
3+ 2 = 5
Clearly, it is not a standard dice.
∴ Digit opposite to 2 = 10 or 6
3. What digit will be on the face opposite to face having digit 1 in
the two positions of a single dice given below :

6 5
1 2 3
2

I II
(a) 2 (b) 5 (c) 3 (d) 4
Ans. (c)
Explanation : As in this case, 5 and 6 are common digits while
4 and 2 are uncommon. As the rule given in note above, here
uncommon digits will be opposite to each other, Hence, 2 will
definitely be opposite to digit 4.
Note :
When two digits are common in both the positions of a
single dice, the uncommon digits in each are opposite to each
other.
4. In the following question the two positions of a single dice are
given. Find the digit on the face which is opposite to the face
having digit 4.
70 Fundamentals of Reasoning

4 5
6 6 2
5

I II
(a) 2 (b) 5 (c) 3 (d) 1
Ans. (a) 2
Explanation :
In different exams, questions on painted cubes are asked.
In such questions, a larger cube is painted with one, two,
three, four, five or maximum six different colours. This larger
cube is then cut into smaller cubes of same or different
dimensions and it is asked to determine the number of
smaller cubes with one or more surfaces painted (with same
or different colours).
10.1. LARGER CUBE PAINTED WITH A SINGLE COLOUR
In this type, a larger cube is painted with single and then
cuts are made to form smaller cubes.
I. Smaller cubes with one painted face
Such cubes are central cubes and they are neither
attached with any edges nor corners.
∴ Number of smaller cubes having one surface painted
= 6 ( n − 2) 2
side of larger cube
where n=
side of smaller cube
II. Smaller cubes with two painted faces
Such cubes are middle cubes and they are attached
with edges (12).
∴ Number of smaller cubes having 2 surfaces
painted in a larger cube = 12 ( N − 2) .
III. Smaller cubes with three painted surfaces
Such cubes are corner cubes. As a cube has 8 corners,
hence number of smaller cubes having 3 faces painted is
always 8.
IV. Smaller cubes with no painted surfaces
Such cubes are inner central or nucleus cubes and
they are invisible.
Reasoning-Dice and Cube 71

∴ Number of inner central or nucleus cubes = ( n − 2) 3


Directions (Q. no. 5 - 8) : Read the following
information carefully and answer the questions given below :
A cube is coloured red on all of its faces. It is then cut into
64 smaller cubes of equal size. The smaller cubes so obtained
are now separated.
5. How many smaller cubes have no face coloured :
(a) 24 (b) 8 (c) 16 (d) 10
Ans. (b) 8
Explanation : Number of smaller cubes = ( n ) 3
64 = ( n ) 3
⇒ n=4
No. of smaller cubes with no surface painted
= ( n − 2) 2 = ( 4 − 2) 3
= 8.
6. How many smaller cubes have only 3 surfaces painted with red
colour :
(a) 0 (b) 12 (c) 24 (d) 8
Ans. (d) 8
Explanation : Number of smaller cubes with two surfaces
painted
=8
7. How many smaller cubes have only one surface painted with
red colour :
(a) 24 (b) 8 (c) 48 (d) 12
Ans. (a) 24
Explanation : No. of Smaller cubes with one painted face
= 6 ( n − 1) 2 = 6 ( 4 − 2) 2
= 6 × 4 = 24
8. How many smaller cubes will have atleast two surfaces painted
with red colour :
(a) 4 (b) 32 (c) 18 (d) 24
Ans. (b) 32
Explanation : Number of smaller cubes with atleast two
surfaces painted
= Number of cubes with three surfaces painted
72 Fundamentals of Reasoning

+ Number of cubes with two surfaces painted


= 8 + ( n − 2) × 12 = 8 + ( 4 − 2) × 12 = 8 + 24 = 32
9. A bigger cube of 5 inch side is formed by keeping together
smaller cubes of 1 inch side. All the faces of this bigger cube is
painted red. When the bigger cube is divided into original
smaller cubes, then how many smaller cubes have two faces
painted :
(a) 4 (b) 8 (c) 36 (d) 12
Ans. (c) 36
Explanation :
side of larger cube 5
n= = =5
side of smaller cube 1
Required no. of cubes = 12 ( n − 2)
= 12 ( 5 − 2)
= 36
10. A solid cube has been formed with 216, smaller cubes. How
many smaller cubes are completely invisible :
(a) 64 (b) 8 (c) 36 (d) 48
Ans. (a)
Explanation : Number of invisible (inner central) cubes
n = 3 216 = 6
= ( n − 2) 3
= ( 6 − 2) 3 = 64
2. When a Larger cube is painted with more than one
colour
Case I : If a cube is painted on all of its surfaces with
different colours and then divided into various smaller cubes
of equal size.
(i) No. of smaller cubes having no face painted = ( n − 2) 3
(ii) No. of smaller cubes having one face painted
= 6 ( n − 2) 2
(iii) No. of smaller cubes having two faces painted
= 12 ( n − 2)
(iv) No. of smaller cubes having with three painted faces
= 8.
Reasoning-Dice and Cube 73

3. Case II : If a cube is painted on its surface in such a


way that one pair of opposite surfaces is left unpainted.
(i) No. of smaller cubes with three surfaces painted = 0
(ii) No. of smaller cubes with two surfaces painted
= 8 + ( n − 2) × 4
(iii) No. of smaller cubes one surface painted
= ( n − 2) × 8 + ( n − 2) 2 × 4
(iv) No. of smaller cubes with no side painted
= ( n − 2) 2 × 2 + ( n − 2) 3
11. A cube is segmented into 125 cubes. Before dividing the cube,
each face of it is varnished with different colours. How many
tiny cubes will be formed having two faces painted :
(a) 36 (b) 32 (c) 44 (d) 54
Ans. (a) 36
Explanation :
( n ) 3 = 125 ⇒ n = 3 125 = 5
No. of cubes having two faces painted = 12 ( n − 2)
= 5 − 2 × 12 = 36.
Hence option A is the answer.
12. A cube is segmented into 2016 equal tiny cubes. Before dividing
the cube, each face of it is painted with different colours. How
many tiny cubes will be formed having one faces painted.
(a) 78 (b) 64 (c) 48 (d) 96
Ans. (d) 96
Explanation :
( n ) 3 = 216 ⇒ n = 3 216 = 6
Total number of cubes having face painted.
6 ( n − 2) 2 = 6 ( 6 − 2) 2 = 6 × ( 4) 2 = 96
Hence option D is the answer.
Direction (Q. no. 13-15) : A cube of side 4 cm is painted
red on one pair of opposite surfaces, green on another pair of
opposite surfaces and one pair of opposite surfaces is left
unpainted. Now, the cube is divided into 64 smaller cubes of
side 1 cm each.
74 Fundamentals of Reasoning

(i) Number of smaller cubes with three surfaces painted


= 0.
(because each smaller cube at the corner is attached
to a surface which is unpainted).
(ii) Number of smaller cubes with two surfaces painted
= Number of cubes present at the coiner + Number of
middle cubes present of the four surfaces
= 8 + ( n − 2) × 4
⇒ 8 + 8 = 16
(iii) Number of smaller cubes with one surface painted
= Number of cubes present at the 8 edges + Number
of cubes present at the four surfaces
= ( n − 2) × 8 + ( n − 2) 2 × 4 = 2 × 8 + 4 × 4
⇒ 16 + 16 = 32
(iv) Number of smaller cubes with no side painted =
Number of central cubes on th two unpainted
surfaces + Number of cubes present inside the cube
= ( n − 2) 2 × 2 + ( n − 2) 3 = 4 × 2 + ( 2) 3
⇒ 8 + 8 = 16
13. How many smaller cubes have two surfaces painted :
(a) 8 (b) 0 (c) 4 (d) 16
Ans. (d) 16
Explanation : Required cubes = 8 + ( n − 2) × 4
= 8 + ( 4 − 2) × 4
= 8 + 8 = 16
14. How many smaller cubes have no surface painted :
(a) 16 (b) 0 (c) 24 (d) 8
Ans. (a) 16
Explanation : Required cubes = ( n − 2) 2 × 2 + ( n − 2) 3
= ( 4 − 2) 2 × 2 + ( 4 − 2) 3
= 8 + 8 = 16
15. How many smaller cubes have only one surface painted :
(a) 18 (b) 48 (c) 32 (d) 24
Ans. (c) 32
Reasoning-Dice and Cube 75

Explanation : Required cubes = ( n − 2) × 8 + ( n − 2) 2 × 4


= ( 4 − 2) × 8 + ( 4 − 2) 2 × 4
= 16 + 16 = 32
PRACTICE SET-1
1. Which number will appear on the face opposite to 3 :

2 2
3 4 1
4

I II
(a) 6 (b) 5 (c) 4 (d) 1
Ans. (d) 1
2. Find the digit at the face opposite to the face having digit 6 in th
given dice :

3
5 6

(a) 1 (b) 2 (c) 1/4 (d) 1/2/4


Ans. (d) 1/2/4
3. Find the digit at the face opposite to the face having digit 2 in
the dice given below :
(a) 4/5 (b) 3/4/5 (c) 3/4 (d) 3/5
Ans. (c) 3/4
4. From the given two positions of a single dice, find the digit at
the face opposite to the face having digit 3 :

2 1
4 5 3
5

I II
(a) 1 (b) 2 (c) 5 (d) 4
Ans. (d) 4
5. All the faces of a cube are painted in blue and then it is divided
into 343 smaller cubes. How many smaller cubes have only one
face painted :
(a) 125 (b) 150 (c) 60 (d) 300
Ans. (b) 150
76 Fundamentals of Reasoning

6. A painted cube having 25 cm side is divided into smaller cube of


5 cm side. How many smaller cube have atleast two faces
painted :
(a) 40 (b) 44 (c) 48 (d) 50
Ans. (b) 44
7. A solid cube is formed with 27 smaller cubes. One pair of
opposite faces of this cube is painted red, another pair is
painted yellow and the third pair of opposite faces is painted
white. How many smaller cubes have two faces painted :
(a) 12 (b) 16 (c) 8 (d) 24
Ans. (a) 12
8. A cube is painted red on two adjacent surfaces and black on the
surfaces opposite to red surfaces and green on the remaining
faces. Now, the cube is cut into 64 smaller cubes of equal size.
how may smaller cubes have less than three surfaces painted :
(a) 24 (b) 8 (c) 28 (d) 48
Ans. (d) 48
Directions (Q. no. 9-10) : A cube of side 5 cm is painted
blue on one pair of opposite surfaces, yellow on another pair of
opposite surfaces and one pair of opposite surfaces is less
unpainted. Now the cube is divided into 125 smaller cubes of
side 1 cm each.
9. How many cubes have only one surface painted :
(a) 20 (b) 60 (c) 18 (d) 125
Ans. (b) 60
10. How many cubes have no surface painted :
(a) 45 (b) 27 (c) 54 (d) 36
Ans. (a) 45
qqq
11
REASONING-FIGURE SERIES
INTRODUCTION
A series is described as a sequential arrangement of
figures following a certain pattern of transition from one to
other.
Different types of questions covered in this chapter are as
follows :
l Choosing the Next Term in the Series
l Choosing the Missing Figure in the Series
l Detection of Incorrect Order in the Series
l Detecting the Wrong Figure in the Series
In the questions based on this chapter, a series of figures
is given as problem figures and the candidate is require to
select one figure from the given set of answer figures which
will continue the given sequence. A series of figures is formed
when each of the consecutive figures of the series is obtained
from the previous figure by following a certain pattern like
clockwise or anti-clockwise rotation, movement of symbols
inside the figure, addition or deletion of designs, replacement
or rearrangement of designs etc.
Various types of problems are asked from this chapter,
are as follows :
Type-1 Choosing the Next Term in the Series
In this type of questions, Candidates are required to
select one figure from the answer figures which when placed
next to the last problem figures, will continue the series
following the same pattern/rule as used in the problem figure
series.
This type of questions are explained in the following
examples :

(77)
78 Fundamentals of Reasoning

Directions (Ex. No. 1-5) : In each of the following


questions, a group of five figures following a certain sequence
is given as problem figures. Problem figures are followed by
another group of five figures known as answer figures
marked as (a), (b), (c), (d) and (e). Find out the figure from the
answer figures which when placed next to the problem figures
will continue the sequence of problem figures.
1. Problem Figures Answer Figures

1 2 3 4 6 (a) (b) (c) (d) (e)


Ans. (c)
Explanation : It is clear from the pattern of series that
the outer figure rotates through an angle of 45° in each
subsequent block whereas the inner figure rotates
through an angle of 90° in anti-clockwise direction in
each subsequent block.
2. Problem Figures Answer Figures

1 2 3 4 6 (a) (b) (c) (d) (e)


Ans. (b)
Explanation : The number of sides in the outer figure is
increased by one and number of sides in the inner figure
is decreased by one side successively in each block.
3. Problem Figures Answer Figures

1 2 3 4 (a) (b) (c) (d)


Ans. (a)
Explanation : In question figure all blocks consist of two
figures touching each other and half of any of them is
shaded. The sequence in the question figure also shows
that in the odd number blocks, top figure is empty and
bottom is shaded while in the even number blocks top
Reasoning-Figure Series 79

figure is shaded while the bottom one is empty. Hence


figure (a) is same as question figure.
4. Problem Figures Answer Figures

1 2 3 4 (a) (b) (c) (d) (e)


Ans. (d)
Explanation : In the first step, the lower half of the
white figure gets shaded : in the second step, the complete
figure gets shaded and in the third step. the figure gets
replaced by a new white figure. The three steps are
repeated to continue the series.
5. Problem Figures Answer Figures

1 2 3 4 (a) (b) (c) (d) (e)


Ans. (c)
Explanation : The circle move sequentially one, two,
three, four, …spaces (each space is equal to half-a-side of
the square boundary) in anti-clockwise direction.
Type-2 Choosing the Missing Figure in the series
In this type of questions, one figure from the set of
problem figures, marked as (1, 2, 3, 4, 5) is missing and this
missing figure is represented by the sign ‘?’. Candidates are
asked to choose one figure from the set of answer figures
marked as (a, b, c, d, e) which completes the series of problem
figures, based on a certain rule and replaces the sign ‘?’.
This type of questions are explained in following
examples:
Direction (Q. no. 6-10) : Each of the following questions
consists of five problem figures marked as 1, 2, 3, 4 and 5,
followed by five answer figures marked as (a), (b), (c), (d) and
(e). If one of the answer figures is placed in the place of ‘?’ in
the problem figure, a series is established and the figure
which replaces the ‘?’ is your answer.
80 Fundamentals of Reasoning

6. Problem Figures Answer Figures

?
1 2 3 4 5 (a) (b) (c) (d) (e)
Ans. (c)
Explanation : The figure rotates 45° and 90°
alternatively in anti-clockwise direction and half leaf is
added everytime to the figure
7. Problem Figures Answer Figures
S ? ? S ∆ S ∆ ? ∆ C C S ∆ ? ? ∆ ∆ S
?
∆ C ∆ C ? C C S ? S ? ∆ S C C S C ?
1 2 3 4 5 (a) (b) (c) (d) (e)
Ans. (e)
Explanation : Two symbols, starting from the top, in
anti-clockwise direction are interchanged with each
other and the main figure rotates 90° in every successive
figure.
8. Problem Figures Answer Figures

1 2 3 4 5 (a) (b) (c) (d) (e)


Ans. In every alternate step, the whole figure rotates 90°
anti-clockwise and in alternate steps two arcs are added
on the anti-clockwise side.
9. Problem Figures Answer Figures

1 2 3 4 5 (a) (b) (c) (d) (e)


Ans. (a)
Explanation : In each step the whole figure rotate by 90°
clockwise while one element is added in each step
alternately on clockwise and anti-clockwise end.
Directions (Ex. 10-11) : In each of the following
questions, a set of four figure marked 1, 2, 3 and 4 forming a
figure series is given. out of which the last one i.e., Fig (4) is
Reasoning-Figure Series 81

missing. Identify from the four answer figures, marked (a),


(b), (c), and (d), the one which would replace the question
mark (?) in Fig. (4), so as to continue the series.
Type-4 Detection of Wrong Figure in the series :
In this of questions, a series of five figures is given. The
series begins with unnumbered figured. The other four
figures, marked as (a, b, c, d,), continue the series based on a
certain rule. However, one of these four figures does not fit
into the series.
A candidate is supposed to find out the one and only one
of the four numbered figures which does not fit into the series.
The number of that figure is the correct answer.
The following examples will give a better idea about this
type of questions.
10. Problem Figures Answer Figures

1 2 3 4 (a) (b) (c) (d)


Ans. (b)
Explanation : In each step one quarter of the figure is
shaded in clockwise direction.
11. Problem Figures Answer Figures

1 2 3 4 (a) (b) (c) (d)


Ans. (d)
Explanation : Like in 1st two figures common area is
shaded, in next set of figures common area is not shaded.
12. Find out the figure which does not fit into the series and is
your answer.

(a) (b) (c) (d)


Ans. (a)
82 Fundamentals of Reasoning

Explanation : In each step, two spaces of the main


figure is being shaded and one space is erased in next step
this sequence repeated throughout the series, according
to this sequence 1th figure must have two shaded figures
but not so, therefore is the answer figure.
13. Which figure is not following the same sequence as
followed by figures in problem figures :

(a) (b) (c) (d)


Ans. (c)
Explanation : In each step 2 lines are getting omitted
while one line is added in following step.
PRACTICE SET
1. Which figure should replace the? So as to continue the
series :

1 2 3 4 (a) (b) (c) (d)

Ans. (c)
Directions (2-4): Each of the questions, consist of four
figures marked 1, 2, 3 and 4 named as th problem figures and
is followed by four answer figures marked (a), (b), (c) and (d).
Select a figure from the answer figures which will continue
the series as established by the four problem figures.
2. Problem Figures Answer Figures

Ans. (b)
3. Problem Figures Answer Figures

1 2 3 4 (a) (b) (c) (d)


Ans. (a)
Reasoning-Figure Series 83

4. Problem Figures Answer Figures

1 2 3 4 (a) (b) (c) (d)


Ans.
5. Which figure is not following the same sequence as
followed by figures in problem figures :

(a) (b) (c) (d)


Ans. (c)
6. Which figures is not following the same sequence as
followed by figures in problem figures :

(a) (b) (c) (d) (e)


Ans. (c)
7. Find out the figure which does not fit into the series :

(a) (b) (c) (d) (e) (f)


Ans. (e)
8. Which figure must continue series of figure :
Problem Figures Answer Figures

1 2 3 4 (a) (b) (c) (d)


Ans. (b)
9. Which figure must continue series of figure :
Problem Figures Answer Figures

Ans. (a)
84 Fundamentals of Reasoning

10. Same as :

(a) (b) (c) (d) (e)


Ans. (b)
qqq
12
REASONING-INEQUALITY
INTRODUCTION
Inequality means state of being unequal. Inequality
based questions involve use of signs such as 'less than',
'greater than', less than equal to' 'not equal to' etc., in order to
derive a definite conclusion by establishing relation between
different sets of elements.
Important Signs
S. N. Sings Meaning Example Explanation
1. < Less than A< B (i) A is less than B.
(ii) A is neither greater
than not equal to B.
2. > Greater than A> B (i) A is greater than B.
(ii) A is neither smaller
than nor equal to B.
3. ≤ Less than or A≤ B (i) A is less than or
equal to equal to B.
(ii) A is not greater
than B.
4. ≥ Greater than A≥ B (i) A is greater than or
or equal to equal to B.
(ii) A is not less than B.
5. = Equal to A=B (i) A is equal to B.
(ii) A is neither greater
nor smaller than B.

For Example :
5 × 3 = 15 → equal → case of equality
( 5 × 3) ≠ 9 → Not equal → case of Inequality

(85)
86 Fundamentals of Reasoning

we can also write it as, ( 5 × 3) > 9


or ( 5 × 3) is greater than 9 [15 > 9]
Similarly,
( 5 × 3) ≠ 17 can be written as
( 5 × 3) < 17 or (15 × 3) is less than 17 [15 < 17] .
l A common term combine two inequalities.
l If the common term is higher than one and less than
the other, both the inequalities can be combined.
For example :
Inequality (1) : P > X , X > C.
Here, common term is X. X is greater than C but less than P.
So the combination will be like this : P > X > C or C < X < P
Inequality (2) : X < P , X ≥ C
Here, X is less than P, and greater than or equal to term C.
Since X is common, the combination is possible. That is :
P > X ≥ C or C ≤ X < P .
Inequality (3) : A > B,C > D
Here four terms are used but there is no common term. So
these two inequalities cannot be combined.
Inequality (4) : A ≤ B, X ≥ Y
So, here also common term is missing. So they cannot be
combined.
Two types of problems based on inequalities are asked in
various competitive exams. In one type, real symbols are used
to establish the relations between different sets of elements
while in other type, coded symbols in place of real symbols are
used to establish the relations between different sets of
elements. In both the cases the candidate is required to
analyse the given statements and then decide which of the
relations given as alternatives follows from those given in the
statements.
Directions (Q. No. 1-5) : In these questions,
relationship between different elements is shown in the
statements.
These statements are followed by two conclusions.
Reasoning-Inequility 87

Give answer :
(a) If only Conclusion I is true
(b) If only Conclusion II is true
(c) If either Conclusion I or II is true
(d) If neither Conclusion I nor II true
(e) If both Conclusions I and II are true
1. Statements : L > M, M > N , N > P
Conclusion : I. L> P
II. M > P
Ans. (e)
Explanation : Given that,
L> M …(i)
M>N …(ii)
N>P …(iii)
On combining all th three statements, we get
L> M > N > P
Conclusions : I. L > P (true)
II. M > P (true)
So, it is clear that both Conclusions I and II follow from the
given statements.
Ans. (b)
2. Statements : A < B < C ≤ D = E
Conclusions : I. B < E
II. B ≤ E
Given that,
A< B< C ≤ D = E
Here, statements are already combined.
Conclusions :
I. B < E (true)
II. B ≤ E (false)
So, it is clear that only Conclusion I follow from the given
statements.
Ans. (e)
88 Fundamentals of Reasoning

3. Statements : P > Q , Q ≥ R = S < T


Conclusions : I. C ≤ Q
II. R < P
Ans. (a)
Explanation : Statement :
P > Q, Q ≥ R = S < T
∴ P>Q≥R=S<E
Conclusions :
I. Q ≥ S it also follows, because Q ≥ R = S
II. P > R it follows, because P > Q ≥ R
So, both Conclusions I and II follow.
4. Statements : P > M > Q , Q > Z > N
Conclusions : I. M ≥ Z
II. N < P
Ans. (b)
Explanation : Given that,
P>M>Q …(i)
Q>Z>N …(ii)
On combining the statements (i) and (ii) we get
P>M>Q>Z>N
Conclusions :
I. M ≥ Z (false)
II. N < P (true)
So, it is clear that only Conclusion II follows the given
statements.
5. Statements : D > E ≤ F , J < F
Conclusions : I. E < J
II. D > J
Ans. (d)
Explanation : Given that,
and D > E ≤ F …(i)
On combining the statements (i) and (ii), we get
D> E≤ F> J
Conclusions : I. E < J
Reasoning-Inequility 89

II. D > J
So, it is clear that neither Conclusion I not II is true.
Coded Inequalities :
Unlike the simple inequalities, coded inequalities have
all the signs (> , < , = , ≥ , ≤ ) in coded form. e.g., the sign of
greater than (> ) can be coded as φ / @ / * / $ etc. The candidates
are required to replace the codes with real signs and then
solve the questions in the same way as the questions of simple
inequalities are solved.
Directions (6-8) : In the following questions, the symbol
@ , $ = , ( c ) and # are used with following meanings.
P @ Q ⇒ P is greater than Q.
P ( c ) Q ⇒ P is smaller than Q.
P = Q ⇒ P is equal to Q.
P $ Q ⇒ P is either greater than or equal to Q.
P # Q ⇒ P is either smaller than or equal to Q.
Give answer :
(a) If only Conclusion I is true
(b) If only Conclusion II is true
(c) If either Conclusion I or II is true
(d) If neither Conclusion I or II is true
(e) If both Conclusion I and II ane true
6. Statements : U = M , P $U , M $ B
Conclusions : I. P @ B
II. S @ T
Ans. (c)
Explanation : According to the question.
U = M ⇒U = M …(i)
P $U ⇒ P ≥ U …(ii)
M $B ⇒M ≥ B …(iii)
On combining the statements (i), (ii) and (iii), we get
Conclusions : I. P ≥ U = M ≥ B (may be)
90 Fundamentals of Reasoning

II. P = B ⇒ P = B (may be)


So, it is clear that either Conclusion I or II is true.
7. Statements : B@ V , K ( c) C , C # B
Conclusions : I. B @ K II.V @ C
Explanation : According to the question,
B@ V ⇒ B > V …(i)
K ( c) C ⇒ K < C …(ii)
C# B ⇒ C ≤ B …(iii)
On combining the statements (i), (ii) and (iii), we get
K < C ≤ B> V
Conclusions : I.B@ K ⇒ B > K (false)
V @C ⇒ V > C (true)
So, it is clear that only Conclusion I is true.
8. Statements : K @ T , S = K , T ( c) R
Conclusions : I. T = R II. S @ T
Explanation : According to the question,
K @T ⇒ K > T …(i)
S = K ⇒S = K …(ii)
T ( c)R ⇒ T < R …(iii)
On combining the statements (i), (ii) and (iii), we get
S =K >T <R
Conclusions : I. T = R ⇒ T = R (true)
II. S @ T ⇒ S > T (false)
So, it is clear that only Conclusion II is true.
Directions (9-10) : In the following question, find the
relationship that can definitely be deduced on the basis of the
relations given. The symbols used to define the relationship
are as follows.
@ means 'greater than'
$ means 'not' equal to'
# lt means 'less than'
% means 'equal to'
9. If it is given that, 4 × % 54 and × % 40, than
(a) 50% M (b) M # (c) @ M (d) y % 32
Reasoning-Inequility 91

Ans. (d)
Explanation : 4 × %4 y and X % 40
⇒ 4X = 5Y and X = 40
⇒ 4 × 40 = 54
⇒ 32 = 4 ⇒ 4%32
10. If it is given that, N @ P , P # 0, 0 @ M and N % M , then :
(a) O # N (b) O $ N (c) O @ N (d) None
Ans. (c)
Explanation : N @ P , P # O , O @ M and N %M
⇒ N > P , P > O , O > P and N = M
⇒ M = N > P <O ⇒O> M = N
⇒ O > N ⇒ O@ N
PRACTICE SET
Directions (Q. no. 1-5) : In these questions,
relationship between different elements is shown in the
statements.
These statements are followed by two conclusions.
Give answer :
(a) If only Conclusion I is true
(b) If only Conclusion II is true
(c) If either Conclusion I or II is true
(d) If neither Conclusion I nor II true
(e) If both Conclusions I and II are true
1. Statement : D = E ≥ N ; M > A; H < K
Conclusions : I. K > A II. F < K
Ans. (b)
2. Statements : P < Q > T , R ≥ Q
Conclusions : I. R > P II. T < R
Ans. (e)
3. Statements : D > E ≤ F , J < F
Conclusions : I. E > J II. D > J
Ans. (a)
4. Statements : A > B, B = H , H > G
Conclusions : I. A > G II. A > H
92 Fundamentals of Reasoning

Ans. (e)
5. Statements : L < J , F < L, I ≤ M = K
Conclusions : I. I ≥ F II. B ≤ E
Ans. (d)
Directions (Q. no. 6-10) : In the following question,
some symbols are used with the letters having the following
meanings as illustrated below :
l R * S suggests that R is not greater than S
l R @ S suggests that R is neither smaller than nor
equal to S
l R ! S suggests that R is neither greater than nor
equal to S
l R # S suggests that R is neither smaller nor greater
than S
Now, in each of th following questions, assuming the
given statements to be valid, find out which of the two
conclusion I and II given below them is/are definitely true.
Give answers :
(a) If only conclusion I is valid.
(b) If only conclusion II is valid.
(c) If either conclusion I or II is valid
(d) If neither conclusion I or II is valid.
(e) If both the conclusion I and II are valid.
6. Statements : P @ Q , Q ! R, R$S
Conclusions : I. S # P
II. S # Q
Ans. (d)
7. Statements : P # S , S @ R, P * N
Conclusions : I. N $ R
II. P @ R
Ans. (b)
8. Statements : N $ S , S @ R, R # Q , Q # T
Conclusions : I. N @ R
Reasoning-Inequility 93

II. R # T
Ans. (e)
9. Statements : P # S , S @ R, P * N
Conclusions : I. N $ S
II. N @ S
Ans. (c)
10. Statements : C $ D , D @ E , D !G , C # P , P @ N
Conclusions : I. D # P
II. C ! G
Ans. (e)
qqq
13
REASONING-DATA SUFFICIENCY

INTRODUCTION
Data Sufficiency is to check and test the given set of
information, whether it is enough to answer a question or not.
This test is designed to test candidate's ability to relate given
information to reach to a conclusion.
Generally, a question on any of the topics of sequences,
ranking, puzzle test, coding-decoding, blood relations,
ordering or mathematical calculations etc., is given followed
by two, statements. These statements may contain
information to arrive at the answer to the question. You have
to decide which of the statement (s) is/are sufficient to answer
the given questions.
The examples given below will give you a better idea
about 'Data Sufficiency'.
Directions (Ex. no. 1-10) : Each of the questions below
consists of a question and two statements numbered I and II
given below. You have to decide whether the data provided in
the statements are sufficient to answer the question. Read
both the statements.
Give answer :
(a) If the data in Statement I alone are sufficient to
answer the question, while the data in Statement II
alone are not sufficient to answer the question.
(b) If the data in Statement II alone are sufficient to
answer the question, while the data in Statement I
alone are not sufficient to answer the question.
(c) If the data in both the Statements I and II together
are necessary to answer the question.

(94)
Reasoning-Data Sufficiency 95

(d) If the data given in both the Statements I and II


together are not sufficient to answer the question.
(e) If the data either in Statements I alone or in
Statement II alone are sufficient to answer the
question.
1. Is X an even number ?
I. X is multiplied by an even number then result is an
even number.
II. X is divisible by 2.
Ans. (e)
Explanation : Statement I alone is sufficient as an even
number when multiplied by another even number results in an
another even number. Statement II is clear definition of even
number.
2. The sum of ages of P , Q and R is 50 years. What is R age :
I. is 30 years old.
II. P is 10 years older than R.
Ans. (c)
Explanation : Both I and II are necessary to answer the
question. By subtracting Q' s age 30 years from 50, we get 20
years. Then from II comparing N's age and M's age we can get
the answer. Hence option ( c) is correct.
3. In a certain coding system, 38 means adopt good habits. What
is the coding for habit in that systems :
I. 873 means like good pictures.
II. 526 equals passion becomes habits.
Ans. (b)
Explanation : Point II individually is adequate to answer the
question because by comparing the question and the point II,
we can get the coding for habit. Hence option B is the correct
answer.
4. How many children does Anirudh have :
I. Gagan and Tapi have no more siblings.
II. Gagan is the only brother of Tapi who is the only daughter of
Anirudh.
Ans. (b)
96 Fundamentals of Reasoning

Explanation : From Statement II, It is clear that Gagan and


Tapi the two children of Anirudh.
From Statement I, It is not clear that what is that what is the
relation of Aninudh with Gagan and Tapi and Tapi.
So, Statement II alone is sufficient to answer the question.
5. What is the value of x :
I. x − y = 2n − ( 4 + y)
II. 3 n + 2 y = 18
Ans. (a)
Explanation :
From Statement I. n − y = 2 n − ( 4 + y)
From Statement II. 3 x + 2 y = 18
⇒ x − y = 2x − 4 − y
⇒ 2x − n = 4 + y − y
⇒ x=4
So, the value of x can be calculated using Statement I alone but
Statement II alone is not sufficient.
6. Ravi, Jay and Sandeep's wage is in the ratio 4 : 5 : 7,
respectively. How much is Jay's wage :
I. The difference between Jay and Sandeep's wage is
double that of Ravi and Jay.
II. Jay gets 4000 less than that of Sandeep.
Ans. (b)
Explanation : As given in the question, the salaries of Ravi,
Jay and Sandeep are 4x, 5x and 7x, respectively.
From Statement II. 7x − 5x = 4000 and x = 2000
∴ Jay's salary = 5 × 2000 = 10000
So, Statement II alone is sufficient to answer the question.
7. How is R related to M
I. R has two brothers and one sister.
II. Brother of M is married to R's sister.
Ans. (d)
Explanation : Statement I is insufficient in itself.
From Statement II,
Reasoning-Data Sufficiency 97

M M's brother R's sister D

No definite relation is the possible as


gender of both M and R are unknown
Hence, Statement II is also insufficient.
Further, no relation is possible from Statements I and II also.
So, both statements are insufficient to answer the question.
8. How will 'INDIA' be written in a certain code ;
I. In this code SALTY is written as ASLYT.
II. In this code MANGO is as AMNOG.
Ans. (e)
Explanation : From Statement I,
Number of students in class = 21 or 22 or 23 or 24 or 25 or 26
From Statement II.
Number of students in class = 24 or 25 or 26 or 27 or 28 or 29
Since, number of students can be divided into gr4oups of 4
students, therefore number of students must be a multiple of 4.
∴ Number of students in the class = 24 or 28
On combining Statements I and II,
Number of students in the class = 24
Hence, the data in both the Statements I and II together are
necessary to answer the question.
10. Who amongst L ,Q , R,O and P is the tallest :
I. L is taller than M. P is not the tallest.
II. N is taller than L. O is not the tallest.
Ans. (c)
Explanation : It is clear that neither of the statements alone is
sufficient.
From Statements I and II, N > L > M and P and O are not the
tallest.
Hence, N is the tallest. So, both Statements I and II are
required to answer the question.
PRACTICE SET
Directions (Q. no. 1-10) : Each of the following
questions below consists of a question and two statements
98 Fundamentals of Reasoning

numbered I and II given below it. You have to decide whether


the data provided in the statements are sufficient to answer
the question.
Give answer :
(a) If the data in Statement I alone are sufficient to
answer the question, while the data in Statement II
alone are not sufficient to answer the question.
(b) If the data in Statement II alone are sufficient to
answer the question, while the data in Statement I
alone are not sufficient to answer the question.
(c) If the data either in Statement I alone or Statement
II alone are sufficient to answer the question.
(d) If the data in both the Statements I and II together
are not sufficient to answer the question.
(e) If the data in both the Statements I and II are
together necessary to answer the question.
1. How is 'call' written in a code language :
I. 'call me back' is written as 794 in that code language.
II. 'you can call me any time' is written as 51409 in that code
language.
Ans. (d)
2. Who is the heaviest among Anmol, Deepak, Ankit and Neeraj.
I. Anmol and Deepak are of the same weight.
II. Deepak's weight is more than Ankit but less than Neeraj.
Ans. (e)
3. On which day in July did the school reopen :
I. Govind correctly remembers that the school reopened after
12th July but before 16th July.
II. Govind's classmate Abhay correctly remembers that the
school reopened after 14th June but before 18th June.
Ans. (e)
4. On which day of the same week is vipul's exam scheduled
(Monday being the first day of the week) :
I. Vipul correctly remembers that his exam is scheduled on a
day after Tuesday but before Thursday of this same week.
Reasoning-Data Sufficiency 99

II. Vipul's friend correctly remembers that vipul's exam is


scheduled on the third day of the week.
Ans. (c)
5. Six persons A, B,C , D , E and F are sitting in a row. A and F are
sitting at two extreme ends of the row. B is to the immediate
right of A and D is 2nd left of F. What is the position of E with
respect to A.
Ans. (b)
6. What is the difference in the ages of x and y :
I. X is 15 yr older than T.
II. T is 4 yr younger than S.
Ans. (d)
7. If x and v are both positive integers, then how much greater is u
than v :
I. u = v 2
II. u + v = 20
Ans. (c)
8. How is Radha related to Rashmi :
I. Mother of Radha is the sister of Rashmi's father.
II. Rashmi is the daughter of only son of Radha grandfather.
Ans. (a)
9. If Vijay is exactly 4 yr older than Pragya, how old is Vijay :
I. Pragya is more than 9 yr old.
II. Exactly 9 yr ago vijay was 5 times as old as was then.
Ans. (b)
10. Towards which direction is G with respect to the starting point :
I. G walked 50 m, took a right turn and walked 30 m again took
right turn and walked 50 m towards West.
II. G walked 40 m, took a left turn and walked 15 m again took
left turn and walked 40 m towards East.
Ans. (c)
qqq
14
REASONING-DECISION MAKING

INTRODUCTION :
It is a process in which a final outcome is derived by
analyzing the given information. In these type of questions, a
set necessary conditions and qualifications required to be
fulfilled by the candidate for a certain vacancy in
job/promotion/admission in a college, along with the bio data
of certain candidates who have applied for the same is given.
You are required to evaluate and assess a candidate's
eligibility and there by decide upon the appropriate course of
action to taken from among the given alternatives.
Let us consider following example to understand the
format of questions asked and the basic step involved in
solving these questions.
Directions (Ex. no. 1-5) : Read th following information
carefully and answer the questions given below it.
Following are the conditions for short listing candidates
for the post of Customer Relations Officers (CRO) for PQR
Limited. The candidate to be called for interview must.
(a) Be a graduate in Science i. e., B.Sc. with minimum
55% marks.
(c) Have atleast 3 yr experience in selling/marketing.
(c) Have participated in debating or drama or sports at
the university level.
(d) Have secured minimum 60% marks in the written
examination.
Give answer :
(a) Selected for interview
(b) Not. to be selected
(100)
Reasoning-Decision Making 101

(c) Data inadequate


(d) Refer to the Chief Manager (CR)
(e) Refer to the DGM (CR).
1. Deepa has 4 yr of experience in marketing consumer products.
She has completed her B.Sc., M.Sc. and MBA with 58%, and
73% marks, respectively. He has obtained 63% marks in
written exam.
2. Gaurav is a 30 yr old officer, who has worked as sales
representative for 5 yr. He is a keen sportsman and actor and
has won many prizes in college/university level events. He has
obtained 67%, 54% and 55% marks at B.Sc., written exam and
diploma in business management, respectively.
3. Akshit is the son of a marketing executive, Akshit has obtained
money as prizes in inter college/university debates and drama
events. He has worked as marketing executive for 5 yr after
completing his B.Sc., M.Sc. and MBA marks in written exam.
4. Shikha is a 26 yr old girl having obtained 53% marks in M.S.c,
51% and 64% at B.Sc. and written exam. respectively. Deepali
1
has worked as sales executive for 3 yr. She has obtained
2
many prizes in inter college events in debating.
5. Vishal is a successful sports person and has won several prizes
in sports at university level. In B.Sc., M.Sc. and written exam,
he has obtained 52%, 55% and 68% marks, respectively. He has
6 yr experience in sales.
Solutions (Ex. no. 1-5) : First of all, let us understand
the contents of the questions.
l Basic conditions A , B ,C , and D
l Additional conditions (i) and (ii)
To solve these questions, we have to follow a step by step
approach. Steps involved in the process are as follows :
Step I : Read carefully the basic conditions A , B ,C and D.
Step II : Read carefully the additional conditions (i) and
(ii) and find out with which basic conditions (A , B ,C , or D )
these additional conditions (i) and (ii) are attached.
l We find that additional condition (i) attached with
basic condition D.
102 Fundamentals of Reasoning

l Additional condition (ii) is attached with basic


condition A .
There is no any other additional condition. It means
apart from A and D, other basic conditions (B ,C and E ) are
independent.
Step III : Make table for the questions as given below :
Step IV : Read information about each candidate and
then put following symbols in the table made in step III.
For basic condition fulfilled.
For unfulfilled basic condition.
For basic condition unfulfilled but additional condition
attached to the basic condition is fulfilled.
For the violation of both basic and additional condition.
For data inadequate.
Table14.1
S.No. Candidate A (ii) B C D (i) Answers.
1. Deepa ü (c)
2. Gaurav ü (d)
3. Akshit ü (a)
4. Shikha ü (b)
5. Vishal ü (e)

Condition A : Shikha & Vishal does not fulfill basic


condition. A but fulfil additional condition (ii) attached to A.
Hence ( ) is put for them. All other fulfill basic condition A.
Therefore, symbol ( ) was put for them.
Condition B : This condition is fulfilled by all
candidates, hence ( ) is put for them.
Condition C : Except Shikha this condition is fulfilled
by all as Shikha has participated in inter college debate.
But condition C demands university level participation.
Reasoning-Decision Making 103

Therefore, for Deepali symbol ( ) is put for all others symbol ( )


is put.
Remember, Shikha does not fulfill basic condition C.
Further, basic condition C is not attached with any additional
condition. Hence, putting (x) for Shikha means, declaring
her disqualified for required post. Therefore, for Shikha,
we will stop our work here. It means condition D and E
will not be checked for her because we have reached our
answer for as option (b) not to be selected.
Condition D : Akshit & vishal fulfill basic condition D.
Hence, ( ) is put for them.
Gaurav violate basic condition D but fulfill additional (i)
attached to D. Hence. ( ) is put for them.
Biodata of Deepa does not give any information about
her percentage of marks in written examination.
Hence, we are not sure about her fulfilling of condition D. As
we lack definite information in case of Deepa. we put '?' for
her.
Now, we have our table ready to answer the given
question. Let us answer the questions as follows.
1. Deepa : Except condition D, all basic conditions are
fulfilled. But there is no information about his percentage of
marks in the written exam. It means data is inadequate.
2. Gaurav : Basic conditions A, B and C are fulfilled.
Further, basic condition D is violated. But additional
condition (i) attached with D is fulfilled. Therefore, case will
be referred to Chief Manager (CR).
3. Akshit : All basic conditions are fulfilled. Hence, he
will be selected.
4. Shikha : Basic condition C is violated. Hence, she will
be rejected.
5. Vishal : All basic conditions, except. A are fulfilled.
But additional condition (ii) attached to A is fulfilled.
Therefore, case will be referred to DGM (CR).
104 Fundamentals of Reasoning

Directions (Ex. no. 6-10) : Study the following


information carefully and answer the questions based on it.
The following are the conditions for the requirement of
person as faculty members in a computer education centre.
The candidate must.
(A) Be in the range of 23 years to 28 years as on 1
November, 1993.
(B) Have work experience : or programming experience.
(C) Have a postgraduate degree in computer
application. i. e., MCA, M.Tech etc. with at least 60%
of marks.
(D) Obtain at least 25 marks in the interview out of a
total of 50 marks.
In case of a candidate who fulfils all these criteria except.
(i) Has a post graduate degree in computer application
with less than 60% marks but 50% marks, he/she
should be appointed as junior faculty member.
(ii) Is of age more than 28 years but less than 32 years as
on 1 Nov., 1993, in case may be referred to director of
centre.
On the above condition and the information provided, we
have to decide which of the following courses of action should
be taken against each candidate. Don't assume anything. The
decisions will be made only on the information provided.
Mark answer :
(a) If the candidate is to be selected as junior faculty
member.
(b) If the candidate is to be selected as senior faculty
member.
(c) If the case is to be referred to the director of the
centre.
(d) If data is inadequate.
(e) If candidate is not to be selected.
Reasoning-Decision Making 105

6. Manisha Punjabi is M.Sc. in computer science with 65% of


marks. Her date of birth is 31 st July, 1965. She has been
working in an institution as a programmer for last 4 years.
7. Kishore Garg is MCA with 61% marks. His date of birth is 14
August, 1970. He has worked as a computer teacher for 3 years.
He has obtained 60% marks in the interview.
8. Prerna Tyagi is M.Tech with 58% marks. Her date of birth is
31 st December, 1971. She has been working as a programmer
for last two-and-half years. She has obtained 40 marks in an
interview.
9. Suresh Kumar is M.Sc. with 52% marks. His date of birth is 10
July, 1968. He has been working as computer teacher for 4
years. He has obtained 40% marks in interview.
10. Sudha Ranjan is MCA with 56% marks. She has been working
as a programmer for 3 years. Her date of birth is 12 February,
1969. She has obtained 55% mark in interview.
Solution :
Step-I : There are 4 primary conditions 'A', 'B', 'C' and 'D',
and two additional conditions are (i) and (ii).

Q.No. A/(ii) B C/(i) D


Manisha
Kishore
Prerna
Suresh
Sudha

Step-II : Read each question carefully and compare facts


given in it with the various conditions and put appropriate
mark below.

Q.No. A/(ii) B C/(i) D


Manisha
Kishore
Prerna
106 Fundamentals of Reasoning

Suresh
Sudha

Manisha : Manisha is a M.Sc. student with 65% marks.


This fulfils condition (c). Condition (A) is violated though on
31st July, 1993, she is more than 28 years. Additional
condition of (A) is (ii). (B) is satisfied, that's why we put ( )
mark here. Manisha has more than 2 years of experience. So
we put ( ) mark below condition (B). Information about
Manisha's mark in interview is missing so we put '?' mark
there.
Kishore : Kishore Garg is an MCA with 61% marks. It
fulfils condition C, So we put A ( ) mark below C. According to
his date of birth, he will be slightly more than 23 years. So it
fulfils A. He has a work experience of more than 3 years. So it
fulfis D. He acquired 60% marks in interview that fulfils D.
Prerna : Prerna Tyagi is certainly below 23 as her date of
birth is in 1971. This violates condition A. The additional
condition attached to A is (ii) is also violated. We put a × mark
there.
Suresh : Suresh is M.Sc. with 52% marks. So it violates C
but fulfils the additional condition (i) attached to C. We put ( )
mark there. He is born in 1968, so he is nearly 25 years. Hence
A is satisfied. We put ( ) mark below A. He worked as
computer teacher for 4 years. So it fulfils condition D. Marks
obtained in the interview is 40%. It violates D. So we put ×
mark below D.
Sudha : Sudha is an MCA with 56% marks. This violates
condition C but fulfils (i). We put A ( ) mark there. She has an
experience of 3 years. It fulfils condition B. She was born in
1969, so she is 24 years old. So, condition A is satisfied. She
has obtained more than 50% marks. So condition D is
satisfied.
6. Ans. (d) Manisha : No information about her interview
marks. It means data is inadequate.
Reasoning-Decision Making 107

7. Ans. (b) Kishore : All basic conditions are fulfilled.


Hence, selected as senior faculty member.
8. Ans. (e) Prerna Basic condition A is violated. Hence
she will be rejected.
9. Ans. (e) Suresh Basic condition D is violated. Hence
she will be rejected.
10. Ans. (a) Sudha : Basic condition A,B and D are
fulfilled. Further, basic condition C is violated. But additional
condition (i) attached with C is fulfilled. Therefore, Selected
as junior faculty member.
PRACTICE SET
Directions (Q. n. 1-5) : Read the following information
carefully and then answer the questions given below it.
Following are the conditions for selecting a'research officer'
for a reputed research institution.
The candidate must :
A. Be a post-graduate with minimum 60% marks.
B. Have Ph.D. degree.
C. Have research experience of atleast 3 yr.
D. Have fluency in Hindi and English.
E. Have published atleast 5 research papers.
F. Not be less than 25yr and more than 35 yr as on 1st
August, 2016
In the case of a candidate who :
(i) Satisfies all other criteria except A above but has
post-graduate degree with more than 52% marks,
his case will be referred to the director of the
institute.
(ii) Satisfies all other criteria except C above will be
referred to the join director.
Now, read the information provided in the case of the
candidates in each of the questions given below and decide on
108 Fundamentals of Reasoning

the basis of the information provided and above conditions,


which of the following courses of action is to be taken.
Give answer :
(a) If candidate is to be referred to the director.
(b) If the data provided are inadequate to decide the
courses of action.
(c) If the candidate is to be selected.
(d) If the candidate is to be referred to the joint director.
(e) If the candidate is not to be selected.
1. Naman is a post-graduate with 63% marks. He has obtained
Ph.D. degree. He has research experience of 5 yr and has got 7
research papers published.
Ans. (b)
2. 31 yr old sumit is a post-graduate with 64% marks and has
obtained Ph.D. degree. He is fluent in Hindi and English. He
has got 5 research papers published and has research
experience of 4 yr.
Ans. (c)
3. Preeti is post-graduate with 68% marks. She has obtained
Ph.D. degree and has published 5 research papers. She has
1
research experience of 3 yr and fluent in Hindi and English.
2
Her date of birth is 4th october, 1991.
Ans. (e)
4. Himanshu is post-graduate with 57% marks and has obtained
Ph.D. degree. He is fluent in Hindi and English. He got 6
research papers published and has research experience of 4 yr.
His date of birth is 1st 1982, November.
Ans. (a)
5. 29 yr old Sakshi has obtained his Ph. D. degree and is a
post-graduate with 72% marks. She has research experience of
1
2 yr. She has got published 7 research papers and is fluent in
2
English and Hindi.
Ans. (d)
Reasoning-Decision Making 109

Directions (Q. n. 6-10) : Following are the criteria for


selection of chemistry lecturer in an organization. The
candidate must :
A. Have passed +2 examination in first class with at
least 60% marks.
B. Have passed graduation with chemistry honours or
pass degree with at least 55% marks.
C. Have experience in chemistry teaching for a
minimum of 1 year.
D. Be not less than 23 years and not more than 28 years
of age as on 1.1.2014.
If a candidate fulfils all the above mentioned criteria
except.
E. Above but has a post-graduate degree in chemistry,
may be referred to the executive director ( ED ).
F. Above but has experience of marketing and can do
marketing, may be referred to the vice president
(VP ).
Based on these criteria and information provided below,
decide the courses of action in each case. You are not to
assume anything. If the data provided is not adequate to
decide the given course of action, your answer will be data
inadequate. All the cases are given to you as on 1.1.2014.
(a) If the candidate is to be selected.
(b) If the case is to be referred to ED .
(c) If the case is to be referred to VP.
(d) If the information given is inadequate to answer.
(e) If the candidate is not to be selected.
6. 27 year old Laxmi Rattan Shukla has passed +2 with 70%
marks and is a chemistry graduate with 50% and he has also
done M.Sc. In chemistry. He has experience of teaching for
more than 1 year.
Ans. (b)
110 Fundamentals of Reasoning

7. Mamta Kulkarni has secured 62% marks in B.Sc. She has


secured 68% in the +2 but has teaching experience of 4 years.
She was born on 10 July, 1984.
Ans. (e)
8. Ritu Chandra has secured 57% in B.Sc. and 64% in +2
examination. She has no experience in teaching but are can do
marketing for the institute. She has experience in marketing.
Her age is around 26 years.
Ans. (c)
9. Mansi Malhotra is a B.Sc. graduate in chemistry and has
scored 65% in her graduation. She has teaching experience of 3
years and her age is not more than 26 years.
Ans. (d)
10. Manan Taneja has completed +2 from Sitaram Junior College
with 75% marks and graduated with chemistry hons. from BM
college with 70%. He has teaching experience of more than 4
years and his date of birth is 2nd February, 1987.
Ans. (a)
qqq
15 REASONING-DIRECTION
AND DISTANCE

INTRODUCTION
Direction is a measurement of position of one thing with
respect to another thing or a reference point. Distance
between two points is a measurement of the shortest
distance, i. e., the displacement between the two points.
The problems based on direction and distance have
instructions regarding the movement of a person or an object
from a starting point upto an end point.
These instructions generally provide magnitude as well
as direction of the movement. These Questions on direction
and distance are simpler than other questions, if the student
possesses the right knowledge of the directions.
1. North 2. South 3. East
4. West are four prime directions.
The angle between any two prime directions is 90°.
A direction between two main or prime directions is
known as a subdirection/cardinal direction.
There are four subdirections as given below:
1. North-East (NE) 2. South-East (SE)
3. South-West (SW) 4. South-West (NW)
This various directions discussed above are represented on
paper as shown below :
S

NW NW

90°
W E
90°
45°

SW SW

(111)
112 Fundamentals of Reasoning

Angle formed between a prime direction and a


subdirection is 45°.

S.N. Direction before Direction in which the person or


taking the turn vehicle will be moving after
taking the turn
Right Left
(i) North East West
(ii) South West East
(iii) East South North
(iv) West North South
(v) North-West North-East South-West
(vi) South-West North-West South-East
(vii) South-West South-West North-East
(viii) North-East South-East North-West
Note :
If the angle at which a turn takes place is not given, then
the angle is assumed to be right angle. i. e., 90°
1. A man walks 8 km South, turns left and walks 5 km, again
turns left and walks 6 km. Which direction is he facing now :
(a) West (b) South (c) North (d) East
Ans. (c)
Explanation :
A N
D

8 km 6 km
W E

B C
5 km S

2. Naresh walks 2 m South from his house, turns left and walks
30m, again turns left and walks 45 m, then turns right and
Reasoning-Direction and Distance 113

walks 5m to reach the market. In which direction is the market


from his house :
(a) South-West (b) East
(c) North-East (d) North
Ans. (c)
Explanation :
5m
Market
N

NW NE

45 m
W E
House

20 m SW SE

S
25 m
So, it is clear from the diagram that market is in North-East
direction from Naresh's house.
3. Ram started walking from his in eastward direction. After
walking some distance he turned to his right and walked, after
walking some distance he turned to his left and walked. After
some time he takes two left-hand side turns simultaneously.
which direction is he facing now ?
(a) East (b) West (c) North (d) South
Ans. (a)
Explanation : First Ram started walking to east then he took
a right-hand side turn. Now he is facing south. After walking
some distance, he turned left-hand side so after turning now he
is facing east. After this, he has taken two left-hand side
movement simultaneously so now he is facing towards west
direction. And finally after walking some distance he has again
moved to his left-hand side twice so finally after his all
movements, he is currently facing towards east direction.
4. Priya goes 7 km straight from her school. She turns to her right
and walks 3 km. She again moves 3 km after turning right to
reach her hose. If her house is located in South-East from her
school, then in which direction Priya started moving initially
from her school :
114 Fundamentals of Reasoning

(a) North-East (b) West


(c) East (d) North-west
Explanation :
School 7m N

NW NE
Right
So
ut

W E
3 km
h-
Ea

Right
st

SW SE

House 3 km S

Clearly, he started moving towards East initially.


5. Sachin started from point K and walked straight 4 km in West,
then turned left and walked straight 3 km and again turned
left and walked straight 9 km. In which direction is he from the
point K :
(a) North-West (b) South-East
(c) South-West (d) North-East
Ans. (b)
Explanation :
4 km West Starting N

A point NW NE
K
Left South-East W E
2 km
Left SW SE
S
B C
9 km Final
point
Clearly, point C is to the South-East of X.
6. Sohan walked 4 m towards South, took a left turn and walked
15 m. He, then took a right turn and walked 25 m. He again
took a right turn and walked 15 m. How far is he from the
starting point :
(a) 95 m (b) 65 m (c) 70m (d) 80m
Ans. (b)
Explanation :
Reasoning-Direction and Distance 115

P Starting
position

40 m
South N

Left 15 m
W E

Q R
S
Right
25 m
25 m
Right

T S
15 m
Final
point
PQ = 40m
QR = TS = 15m
QT = RS = 25m
∴ Required distance, PT = PQ + QT = 40 + 25 = 65m.
7. Zaheer walks Northward upto 10 m. He turns left and walks
30m. Finally, he turns left and walks 50m. At what distance
Zaheer is now from his starting position :
(a) 20m (b) 90m (c) 10m (d) 50 m
Ans. (d)
Explanation :
IO = 10m,OP = 30 m, PF = 50m
QI = OP = 30m, PQ = IQ = 10m
QF = PF − PQ = 50 − 10 = 40m
F represents the final position.
∴ Required P
30 m
O
distance,
O
50 m
Q Initial
position
40 m

F
Final
position
116 Fundamentals of Reasoning

IF = (QI ) 2 + (QF ) 2

= ( 30) 2 + ( 40) 2 = 900 + 1600 = 2500 = 50 m


8. Krishna is going to a local market place for grocery. He has
taken his cycle from home and starts moving to north. He went
for 10 km. After that he turned his left and cycled for 6 km.
After that he again to his left and cycled for 18 km. Now he has
reached the market place. What is distance between Krishna's
home and market place and he is in which direction from his
home ?
(a) North-east, 15 km (b) North-west, 12 km
(c) South-west, 10 km (d) South-east, 10 km
Ans. (c)
Explanation :
IC = 6 km
CD = BD − IA = 18 − 10 = 8 km
ID = 82 + 62 = 64 + 36 = 100
= 10 km
6 km
N 8 A
NW NE
18 km 10 km
Home
W E
C I

SW SE
S

D
Clearly, Krishna travel 10 km in south-west direction.
9. Jyoti goes 12 km North, then goes 8 km South, again she moves
on 3 km west. At what distance is she now from original
position and in which direction ?
(a) 5 km North-West (b) 5km South-East
(c) 4 km North-East (d) 10 km South-East
Ans. (c)
Explanation :
AD = 42 + 32
= 16 + 9
Reasoning-Direction and Distance 117

= 5 km
C

8 km

3 km 12 km
D B

4 km
Original
A Position
Hence, Jyoti is at a distance of 5 km in North-West direction.
10. Salim started to move in the direction of west in an open field
and stopped after moving 5 km. Now he turned to his left and
moved 12 km. How much minimum distance he has to cover to
reach his starting point ?
(a) 10 km (b) 7 km (c) 15 km (d) 13 km
Ans. (d)
Explanation : AC = 5 2 + 12 2
= 25 + 144
= 169
= 13 km
11. Swagat walks towards North for 15 m and from there he goes
right-hand side for 10 m and again turns right-hand side for 25
m. He is in which direction?
(a) North-West (b) North-East
(b) East (d) South
Ans. (b)
Explanation :
10 m N
8 C NE
NW
15m
W E
Original A
SW SE
Position 25 m S

D Final
Position
118 Fundamentals of Reasoning

Clearly, paint D is to the South-east of A.


12. Sudhir starts from his house and travels 4 km in East direction,
after that he turns towards left and moves 4 km. Finally, he
turns towards left and moves 4 km. At what distance and in
which direction he finally stands from his starting point ?
(a) North, 4 km (b) North-East, 4 km
(c) South, 12 km (d) West, 4km
Finishing
point
4 km N
D C
Left

4 km 4 km W B
North
Left

A B S
4 km East
Starting
point
Clearly, at finishing point, he is 4 km North from the starting
point.
13. Town D is 12 km towards the North of town A. Town C is 25 km
towards the West of town D. Town B is 25 km towards the West
of town A. How far and in which direction is town B from town
C?
(a) 3 km towards South (b) 15 km towards North
(c) 12 km towards South (d) 12 km towards North
Explanation :
25 km N
C D

12 km 12 km W E

B A S
25 km
Hence, town B is 12 km South of town C.
14. Yash drives a car from Delhi to Agra. After 100 km he turns
right and goes 35 km. After that he again turns to his right and
moves on 65 km. Finally, he turns to his right and stops after
moving a distance of 35 km. At what distance is Yash now from
Delhi.
(a) 35 km (b) 65 km (c) 95 km (d) 30 km
Reasoning-Direction and Distance 119

Ans. (a)
Explanation :
10 km
F C
B
Delhi
Right

35 km
Right Right

E 65 km D
BC = 100 km, CD = FE = 35
DE = FC = 65 km
∴ Required distance, BF = BC − FC = 100 − 65 = 35 km
15. Madhuri travels 29 km Westwards and then turns left and
travels 6 km and further turns left and travels 41 km. How far
is Madhuri now from the starting point?
(a) 100 km (b) 221 km (c) 80 km (d) 180 km
Ans. (d)
Explanation : AB = CE = 29
BC = AE = 6 km
CD = 41 km
ED = CD − CE = 41 − 29 = 12 km
∴ Required distance, AD = ( AE ) 2 + ( ED ) 2
(by Pythagoras theorem)
= 6 + 12 2 = 180 km
2

29 km
B A Starting point
Left Right

6 km 6 km
Left

C 41 km D

PRACTICE SET
1. Ajay walked 20 m towards South, took a left turn and walked
25 m. He, then took a right turn and walked 35 m. He again
took a right turn a right turn and walked 25 m. How far is he
from the starting point?
120 Fundamentals of Reasoning

(a) 95 m (b) 80 m
(c) 55 m (d) Cannot be determined
Ans. (c)
2. Town D is to the West of town M. Town R is to the South of town
D. Town k is to the East of town R. Town K is towards which
direction of town D?
(a) North-west (b) South-west
(c) North-East (d) South-East
Ans. (d)
3. Shyam walked 50 m toward West, took a left turn and walked
25 m. He, then took a right turn and walked 30 m. He again
took a right turn and walked 25 m. How far was he from the
starting point?
(a) 80m (b) 60m (c) 75 m (d) 95 m
Ans. (a)
4. Shiv walked 35 m towards East took a left turn and walked 15
m. He again took a left turn and walked 35 m. How far and in
which direction is he from his starting point ?
(a) 15 m, North (b) 85 m, North
(c) 15 m, South (d) 85 m, South
Ans. (a)
5. Starting from a point X, Neha walked 15 m towards South. He
turned to his left and walked 50 m. He, then again turned to his
left and walked 15 m. He again turned to his left and walked 75
m, and reached a point 4. How far is Neha from point X and in
which direction ?
(a) 10 m, West (b) 25 m, North
(c) 10 m, East (d) 25 m, West
Ans. (d)
6. A man travels 2 m towards North, then he turns towards East
and travels 6 m. Finally, he travels 10 m in South direction.
How far is he from his starting point?
(a) 14m (b) 10 m (c) 4 m (d) 6 m
Ans. (b)
7. A person moves 15 km in East direction, then turns toward
North and moves 4 km. From here he turns towards West and
travels 12 km. How far and in which direction is he from his
starting point?
(a) 31 km, South-West (b) 5 km, North-East
Reasoning-Direction and Distance 121

(c) 19 km, North-East (d) 27 km, South-West


Ans. (c)
8. Starting from a point, Vasu walked 19 m North, he turned right
and walked 12 m, he again turned right and walked 19 m, then
the turned left and walked 8 m. How far is he now and in which
direction from the starting point?
(a) 30 m towards East (b) 4 m towards West
(c) 27 m towards West (d) 20 m towards East
Ans. (d)
9. Subhi is going towards East. She turns left, moves on some
distance and again turns to her left. After walking some
distance she turns to her right and moves on. In which
direction she is going now?
(a) North (b) South
(c) North-West (d) West
Ans. (c)
10. Two cars start from the opposite points of a main road, 150 km
apart. The first car runs for 25 km and takes a right turn and
then runs for 15 km. It then turns left and runs for another 25
km and takes the direction back to reach down the other car has
time, due to the minor break down the other car has run only 35
km along the main road. What would be the distance between
the two cars at this point ?
(a) 85 km (b) 65 km (c) 75 km (d) 80 km
Ans. (b)
qqq
16 REASONING-LOGICAL
SEQUENCE OR WORDS

INTRODUCTION
Logical sequence of words as the name implies is the type
of reasoning which consists of words and we have to find out a
sequence which is logical in that context. Normally, in these
questions, the words are mentioned in serial numbers 1, 2, 3
etc. There should be a minimum of four words to ensure
complexity of the question. There are is no limit on maximum
number of words. There are some questions in which
numbers even range to eight or nine options. But such type os
questions are very rare. Because if you will give nine options
on technical issues then it may be next to impossible for some
candidates to solve the question and find out the logical
sequence of the question. So, generally maximum six options
are provided in this type of problems.
For Example :
1. USA 2. Vatican City 3. England 4. India
Here four numbers are provided and in each of the four
numbers, names of four countries are given. So, now how we
can make a logical sequence of words from such a question.
We all know that according to area Vatican City is world's
smallest country and USA is one of the biggest countries in
the world. The other two countries are England and India and
India is bigger than England. So, we can arrange them either
in ascending or descending order. They can be arranged in
following way :
USA, India, England, Vatican City
or Vatican City, England, Indian, USA.
To get t he answer we have to look at the options and find
out which one is provided. One thing is sure that both the

(122)
Reasoning-Logical Sequence and Words 123

options will not be there so we have to find out any one of them
which will be our answer.
The Examples given below will give you a better idea
about this chapter.
1. Arrange the following words in a meaningful order.
1. World 2. Uttar Pradesh
3. Universe 4. Jhansi 5. India
(a) 4,2,5,3,1 (b) 4,2,5,1,3 (c) 3,1,5,4,2 (d) 4,1,2,3,5
Ans. (b)
Explanation : Jhansi is a city situated in the Uttar Pradesh
state of India. India is a part of the world and world in turn, is a
part of the universe.
So, the correct sequence of part to whole is given as
Jhansi Uttar pradesh India World Universe
and the correct option showing this sequence is (c) i. e.,
4 2 5 1 3
2. Arrange the following words in a logical sequence.
1. Hundred 2. Crore
3. Thousand 4. Lakh
5. Ten
(a) 2,3,4,1,2 (b) 2,4,5,1,3 (c) 5,1,3,4,2 (d) 5,1,3,2,4
Ans. (c)
Explanation : All the words represent the counting numbers
and their increasing order is given as below :
Ten Hundred Thousand Lakh Crore
This order is given in option (c) i. e., 5,1,3,4,2
3. Arrange the following words in a logical sequence.
1. Application 2. Selection
3. Exam 4. Interview 5. Advertisement
(a) 3,5,4,2,1 (b) 5,4,1,2,3 (c) 3,2,5,4,1 (d) 4,5,1,2,3
Ans. (c)
Explanation : For a job,
(i) Advertisement is the 1st stage – 3
(ii) Application is the 2nd stage – 2
(iii) Exam is the 3rd stage – 5
(iv) Interview is the 4th stage – 4
124 Fundamentals of Reasoning

(v) Selection is the final stage – 1


∴ Correct sequence = 3,2,5,4,1
4. Arrange the following words in a meaningful order.
1. China 2. Germany
3. Brazil 4. Russia 5. Maldives
(a) 4,1,3,2,5 (b) 5,2,1,3,4 (c) 4,3,2,1,5 (d) 5,3,2,1,4
Ans. (a)
Explanation : We can arrange it both in ascending or
descending order of their area. Only option (a) is correct.
Because it represents the sequence in descending.
5. Arrange the following words in a logical sequence.
(a) 2,1,3,4,5 (b) 5,1,2,3,4 (c) 4,1,2,3,5 (d) 1,3,2,4,5
Ans. (c)
Explanation : There should be first a goal keeper then
defender then midfielder after that striker and all of these
constitutes a football team.
Directions (Ex. n. 6-8) : In each of the following
questions, arrange the given words in meaningful sequence.
6. 1. Town 2. Street 3. District 4. District 5. House
(a) 3,5,1,2,4 (b) 3,5,2,1,4 (c) 3,1,2,4,5 (d) 3,1,2,5,4
Ans. (b)
Explanation : As we know that, a district has towns, a town
has streets, a street has house and a house has rooms. Hence,
correct sequence is (b)
7. 1. Apply 2. Study 3. Examination 4. Earn 5. Job
(a) 2,3,5,4,1 (b) 1,5,2,3,4 (c) 3,2,1,5,4 (d) 2,3,1,4,5
Ans. (d)
Explanation : First we study to clear our exams, then we
apply for a job and start earning.
8. 1. Noon 2. Night 3. Dawn 4. Twilight
(a) 3,1,2,4 (b) 2,3,1,4 (c) 3,1,4,2 (d) 1,3,2,4
Ans. (c)
Explanation : From sunrise to sunset, dawn is followed by
noon, noon is followed by twilight and twilight is followed by
night.
Directions (Ex. n. 9-10) : Arrange the following words
as per order in the dictionary.
Reasoning-Logical Sequence and Words 125

9. 1. Animalization 2. Ambiguous 3. Ambassador


4. Animation 5. Ambience
(a) 3,5,2,1,4 (b) 3,2,5,4,1 (c) 3,2,1,5,4 (d) 3,5,4,2,1
Ans. (a)
Explanation : According to English dictionary, correct
sequence of words is :
Ambassador Ambience Ambiguous
Animalization Animation
10. 1. Necropsy 2. Necromancy 3. Necropolis 4. Necrophagous.
(a) 2,1,4,3 (b) 1,2,3,4 (b) 2,4,1,3, (c) 2,1,3,4
Ans. (c)
Explanation : According to English dictionary, correct
sequence of words is :
Necromancy Necrophagous Necropolis Necropsy

PRACTICE SET
Directions (Q. n. 1-7) : In each of the following
questions, arrange the given words in meaningful sequence.
1. 1. Journalist 2. Incident 3. Newspaper 4. Article
5. Public
(a) 2,1,4,3,5 (b) 2,1,3,4,5 (c) 5,3,4,2,1 (d) 1,3,2,4,5
Ans. (a)
2. 1. Fruits 2. Vitamins 3. Healthy 4. Body
(a) 4,3,2,1 (b) 2,1,3,4 (c) 1,2,4,3 (d) 1,4,3,2
Ans. (c)
3. 1. Night 2. Darkness 3. Lamp 4. Light 5. Vision
(a) 4,3,5,2,1 (b) 3,2,1,4,5 (c) 2,1,3,5,4 (d) 1,1,3,4,5
Ans. (d)
4. 1. Birds 2. Winter 3. Migration 4. India 5. Siberia
(a) 4,1,2,3,5 (b) 3,1,2,4,5 (c) 4,1,2,5,3 (d) 5,2,1,3,4
Ans. (b)
5. 1. Phrase 2. Letter 3. Word 4. Sentence
(a) 1,3,2,4 (b) 2,3,1,4 (c) 1,2,3,4 (d) 2,3,4,1
Ans. (b)
6. 1. Country 2. District 3. State 4. Village 5. Continent
(a) 5,4,3,2,1 (b) 1,3,2,5,4 (c) 4,2,3,1,5 (d) 2,1,3,5,4
Ans. (c)
126 Fundamentals of Reasoning

7. 1. Funeral 2. Death 3. Birth 4. Marriage


5. Education.
(a) 3,5,4,2,1, (b) 5,4,3,2,1 (c) 3,4,5,1,2 (d) 3,5,4,1,2
Ans. (a)
8. 1. Epithelium 2. Epicenter 3. Epigene 4. Epitome
5. Epimysium
(a) 1,2,3,4,5 (b) 2,3,1,5,4 (c) 2,5,3,1,4 (d) 2,3,5,1,4
Ans. (d)
9. 1. Bonfire 2. Bonus 3. Bouncer 4. Board
(a) 4,1,2,3 (b) 2,4,3,1 (c) 4,2,1,3 (d) 4,3,2,1
Ans. (a)
10. 1. Important 2. Importable 3. Improvise
4. Importance
(a) 2,3,4,1 (b) 2,4,1,3 (c) 3,4,1,2 (d) 2,1,3,4
qqq
17 REASONING-LOGICAL
VENN DIAGRAMS

INTRODUCTION
It is a process of showing complex relationship between
2-3 categories diagrammatically through various geometric
figures such as square, circle, triangle, rectangle etc.
Intersection between two geometric structures indicate that
they have something in common and total isolation indicates
just opposite of that. Though the theory seems to be complex
from above but a basic understanding of various types of
sample cases will help in solving the coming questions more
easily.
The most common type of questions in logical Venn
diagram are based on circular Venn diagrams. Here, a
candidate is required to find out the diagram which classify
the given group correctly or illustrates the relation between
them.
The main motive of asking these type of questions is to
analyse the candidate's ability to relate a certain given group
of items and illustrate it diagrammatically.
Different types of relationships can be established
between three objects/things/places/persons etc., which are
given as follows :

Venn Applicable cases Example


diagram
There will be a series India, Maharashtra, Pune,
of sub cases one Pune is a part (City) of
under another. Maharashtra and
Maharashtra is a part
(state) of India and both
belongs to India.

(127)
128 Fundamentals of Reasoning

One main category, Liquids, Petrol, diesel. Both


under it two sub petrol and diesel are liquids
categories and both and both are flammable,
bear some similarities thus bear similarity.
among them.
One category may Vegetable, Capsicum, Red.
have one sub category. All capsicum are vegetables.
They both partially Also, some vegetables and
satisfy conditions not some capsicum can be red.
always.
There will be two Furniture, Chair, Bed. No
separate categories chair can be chair. But both
which belong to the come under the group of
third category. furniture.
Three sections having Pencil, Book, pen. All three
no common feature. belong to stationery item
but they are entirely
independent of are another.
Three separate Scientists, Authors, Men.
sections partly related Some men are authors.
to one another. Some men scientists are
authors.
Out of three sections, Tree, banana tree, Angry.
one section belongs to We all know banana tree is
the second section coming under tree category,
while the third section and emotion "Angry" is
is entirely different entirely different from tree
from the two. and banana tree.

The following examples will give you a better idea about


the type of questions asked in various competitive exams.
1. Which of the following diagrams best represents a
relationship among Criminal, Thief and Pick pocket :

(a) (b) (c) (d)


Ans. (a)
Reasoning-Logical Venn Diagrams 129

Explanation : Some thieves are pick pockets and they


both come under the category of criminals. This can be
expressed as given in option (a)
Criminals
Pick pocket

Thief
2. Which of the following diagrams best represents a
relationship among Birds, Parrots, Giraffes :

(a) (b) (c) (d)


Ans. (d)
Explanation : Parrots completely belongs to the group
of birds while giraffe is entirely different from both. This
can be expressed as given in option (d)
Birds
Parrots

Giraffes

3. Which of the following diagrams best represents a


relationship among House, Kitchen, Bathroom :

(a) (b) (c) (d)


And. (b)
Explanation : Both kitchen and bathroom are entirely
different but both are parts of a house.
This can be expressed as given in option.
House
Teacher

kitchen
4. Which of the following diagrams represents the
relationship among orange, Peach and Mango :
130 Fundamentals of Reasoning

(a) (b) (c) (d)


Ans. (d)
Explanation : Orange, Peach and mango all are fruits
but they are entirely different from one another.
Orange

Peach
Mango
5. Which diagram correctly represents the relationship
between Scientists, Authors and men.

(a) (b) (c) (d)


Ans. (c)
Explanation : Some men are authors. Some men are
scientist. Some men scientists authors. This can be
expressed as given in option (c).
Authors Men

Scientists
6. Which of the following diagrams best represents a
relationship among Berlin, Europe, Germany :

(a) (b) (c) (d)


Ans. (b)
Explanation : Germany is completely inside Europe
while Berlin is completely inside Germany. This can be
expressed as given in option (b).
Europe
Berlin
Germany
Reasoning-Logical Venn Diagrams 131

7. Which one of the following diagrams represents the


correct relationship among Teachers, Educated,
Employed :

(a) (b) (c) (d)


Ans. (a)
Explanation :
Another type of question that often comes under this
chapter is intersection of various geometric figures. Each
geometrical figure in the diagram represents a certain class.
The candidate is required to study and analyse the figure
carefully and then answer the given questions based on it.
Directions (Ex. N. 8-10) : In the given Venn diagram,
the square represents the who are cute, the circle represents
persons who are smart and the arrow represents polite
persons. Study the diagram and answer the questions that
follow.
Smart
Persons 4

6 3 5
2
1 Polite
Persons
Cute persons
8. Which number represents persons who are smart as well
as cute ;
(a)4 (b)2 (c) 6 (d)3
Ans. (b)
Explanation : Number 2 lies in the common area of
square and circle thus bears the characteristics of both
structures.
9. Which number represents persons who are smart but
neither polite nor cute :
(a)3 (b)4 (c) 1 (d)5
Ans. (d)
132 Fundamentals of Reasoning

Explanation : Number 4 lies in the circle area thus bear


only the characteristics of being smart.
10. Which number represents person who are smart, polite
and cute :
(a)5 (b)2 (c) 3 (d)6
Ans. (d)
Intelligent
Explanation : Number
6 lies in the common
area of square, circle Brave
R
and arrow, thus bears
the characteristics of T S P U
all.
Directions (1-4) : V Handsome
Study the following diagram
and answer the question accordingly.
1. Which letter among the following is intelligent but not
brave or handsome :
(a)T (b)R (c) V (d)U
2. Which letter among the following is intelligent,
handsome brave :
(a)V (b)U (c) S (d)R
3. R is :
(a)Brave and Handsome (b)Handsome only
(c) Brave only (d)Brave and intelligent
Ans. (b)
4. Who among the following is brave but neither intelligent
nor handsome :
(a)U (b)R (c) S (d)P
Ans. (a)
5. Which of the following diagrams best represents a
relationship among sweets, malpua and Tomato :

(a) (b) (c) (d)


Ans. (b)
Reasoning-Logical Venn Diagrams 133

6. Which of the following diagrams indicates the best


relation between ship, ocean and sailor :

(a) (b) (c) (d)


Ans. (c)
7. Which of the following diagrams indicates the best
relation between Professors, Men and Doctors :

(a) (b) (c) (d)


Ans. (c)
8. Magazine, French Magazing, PDF

(a) (b) (c) (d)


Ans. (d)
9. Which of the following diagrams indicates the best
relation between Aluminium, Silicon and carbon :

(a) (b) (c) (d)


Ans. (a)
10. Which of the following diagrams indicates the best
relation between candle, Torch and light :

(a) (b) (c) (d)


Ans. (b)
qqq
18 REASONING-LOGICAL
DEDUCTION

INTRODUCTION
Logical deduction is an important chapter as many easy
and mark fetching questions from this chapter are included
in various competitive exams. In this chapter, we will discuss
two methods to solve such problems :
l Syllogism method
l Venn diagram method
If there are two statements will be the best way to solve
the problem but if number of statements are more than two,
then go for Venn diagram method. Sometimes statements
and conclusions are derived from general accepted facts but
they can also be non logical. For example :
(a) Some chairs are doors.
(b) All mans are woman
(c) No milk is white
To arrive at the conclusion, abstract thinking is required.
One need to understand the logical sense of the sentence then
have to answer the conclusions accordingly. Each statement
needs to thought as true then we have to verify whether
conclusions are logically following the statements.
Qualifiers :
There are some elementary words that are used to
describe how much one thing is similar or different from
other. Example of some of the qualifiers are "All", "Some", No
etc.
Venn Diagram Concept :
On the other hand, Venn diagram is a process where we
can represent a sentence or statement in the form of
geometrical figures. All the given statements are plotted in

(134)
Reasoning-Logical Deduction 135

possible Venn diagrams. Then all conclusions are verified


with those diagrams. Any conclusion that satisfies all the
Venn diagrams will be considered as a valid conclusion.

Statements Corresponding Venn Diagrams


All A are B B

Some A are not B


A B

No A is B
A B

Some A are B
A B

Let us solve some examples :


Directions (Ex. No. 1-6) : In the question below are
given two statements followed by two conclusions. You have
to take t he two given statements to be truce even if they seem
to be at variance form commonly known facts and decide
which of the conclusions (s) logically follow (s) from the two
given statements.
Give answer :
(a) If only conclusion I follows
(b) If only Conclusion II follows
(c) If either Conclusion I or II follows
(d) If both Conclusion I and II follow
(e) If neither I nor II follows
136 Fundamentals of Reasoning

1. Statements : All cats are Lions


Conclusions : I. Some Tigers are Lions
II. Some Tigers are Cats
Ans. (d)
Explanation :
Tigers

Lions

Cats
Conclusions : I. Some Tigers are Lions. ( )
II. Some Tigers are cats. ( )
2. Statements : All writers are doctors.
All readers are doctors.
Conclusions : I. Some doctors are readers.
II. Some readers are writers.
Ans. (a)
Explanation :
Doctors

Writers Readers

Conclusions : I. Some doctors are readers.


II. Some readers are writers.
3. Statements : Some players are actors.
All actors are tall.
Conclusions : I. All players are tall.
II. Some players are tall.
Ans. (b)
Explanation :
Tall

Players Actors
Reasoning-Logical Deduction 137

Conclusions : I. All players are tall. (×)


II. Some players are tall. ( )
4. Statements : All pencils are crayons.
No crayon is pencil a stapler.
Conclusions : I. No pencil a stapler.
II. Some staplers are pencils.
Ans. (a)
Explanation :
Crayon

Stapler
Pencils
Conclusions : I. No Pencil is a crayon. ( )
II. Some staplers are pencils. ( )
5. Statements : No colour is a
brush.
No paint is a brush.
Conclusions :
I. All brushes are colours. Colour Paint Brush
II. No colour is a brush.
Ans. (e)
Explanation :
Conclusions :
I. All brushes are colours. ( ) Brush Colour Paint

II. No colour is a brush. ( )


6. Statements : Some cows are horses All horses are
donkeys.
138 Fundamentals of Reasoning

Donkeys
Conclusions : I. Some horses are cows
Cows
II. Some cows are donkeys
Ans. (d)
Explanation : According to the A
question.
Clearly,
I. Some Horses are cows ( )
II. Some Cows are donkeys (×)
Horses
Directions (Ex. no. 7) : In the question
below are given three statements followed by two conclusions
numbered I and II. You have to take the two given statements
to be true even if they seem to be at variance from commonly
known facts and decide which of the given conclusions (s)
logically follow (s) from the three given statements.
7. Statements : All engineers are fools. All fools are
doctors. All doctors are poor.
Conclusions : I. Some poor are fools.
II. Some poor are engineers.
(a)Only conclusion I is valid
(b)Only conclusion II is valid
(c) Both the conclusion are valid
(d)None of the conclusions are valid
Ans. (c)
Explanation :
Poor
Doctors
Engineers
Fools

Here fools are subset of poor. So it is an obvious fact that


some poor will be fools. Hence, conclusion I is valid.
Similarly conclusion II is valid as engineers are also a
subset of poor. Hence, both the conclusions are valid.
Directions (Ex. no. ) : In the questions below are given
three statements followed by three conclusions numbered I,
II and III. You have to take the given statements to be true
Reasoning-Logical Deduction 139

even. if they seems to be at variance with commonly known


facts. Read all the conclusions and then decide which of the
given conclusions logically follows from the given statements
disregarding commonly known facts.
8. Statements : All dams are rivers.
All rivers are cars.
Some cars are trucks
Conclusions : I. Some trucks are dams.
II. Some cars are dams.
III. Some trucks are rivers.
(a)None follows (b)Only I follows
(c) Only II follows (d)Only III follows
(e) II and III follow
Ans. (c)
Explanation :
Dams Trucks

Rivers

Cars

Conclusions : I. Some Trucks are dams. ( × )


II. Some cars are dams.
III. Some Trucks are rivers. ( × )
9. Statements : Some keyboards are Mouse. Some Mouse
are radios.
Conclusions : I. Some keyboards are radios.
II. Some radios are keyboards.
III. All radios are Mouse.
IV. All Mouse are keyboards.
(a)Only conclusion I is valid
(b)Only conclusion II is valid
(c) Either I or II is valid
140 Fundamentals of Reasoning

(d)None of the conclusions is valid


(e) Both I and II are valid
Ans. (d)
Explanation :
Keyboards

Radios

Mouse
Some keyboards are radios (×)
Some radios are keyboards (×)
All radios are mouse (×)
All mouse are keyboards (×)
10. Statements : Some boys are trees.
No men are flowers
All trees are men.
Conclusions : I. Some boys are men.
II. Some boys are flowers.
III. Some boys are not flowers.
IV. No flower is tree.
(a)I and III follow (b)III and IV follow
(c) II and IV follow (d)II, III and IV follow
(d)I, III and IV follow
Ans. (d)
Explanation : Trees
Flowers

Boys
Men
Reasoning-Logical Deduction 141

Conclusions : I. Some boys are men. ( )


II. Some boys are flowers. (×)
III. Some boys are not flowers. ( )
IV. No flowers is tree.( )
PRACTICE SET
Directions (Q. No. 1-10) : In each of the following
question, two or three statements are given and after that two
conclusions are provided. You have to take the statements to
be true thought it shows difference from the known facts. or
the Read all the conclusions and then decide which of the
given conclusions logically follows from the given statements
disregarding common known facts.
1. Statements : All liquids are waters. Some waters are
boats.
Conclusions : I. Some boats are liquids.
II. All waters are boats.
(a)Only conclusion I is correct.
(b)Only conclusion II is correct.
(c) Either I or II is correct.
(d)Neither I nor II is correct.
(e) Both I and II are correct.
Ans. (d)
2. Statements : No matches are exams. Some matches are
cricket.
Conclusions : I. All cricket are matches.
II. Some cricket are matches.
(a)Only conclusion I is correct.
(b)Only conclusion II is correct.
(c) Either I or II is correct.
(d)Neither I nor II is correct.
(e) Both I and II are correct.
Ans. (b)
3. Statements : Some actors are fools.
Conclusions : I. Some actors re fools.
142 Fundamentals of Reasoning

II. Rajeev is an actor.


(a)Only conclusion I is correct.
(b)Only conclusion II is correct.
(c) Either I or II is correct.
(d)Neither I nor II is correct.
(e) Both I and II are correct.
Ans. (a)
4. Statements : I. All authors are literates.
II. All literates are smart.
Conclusions : I. All authors are smart.
II. All smart are authors.
III. All smart are literates.
IV. Some smart would be authors.
(a)Only IV follows (b)Only II follows
(c) III and IV follows (d)I and IV follows
(e) No solution.
Ans. (d)
5. Statements : Some bikes are cars.
All cars are buses.
Conclusions : I. Some bikes are buses.
II. No car is a bike.
(a)Only conclusion I is correct.
(b)Only conclusion II is correct.
(c) Neither I or II is correct.
(d)Both I and II are correct.
Ans. (a)
6. Statements : Some staplers are erasers.
Some scales are erasers.
Conclusions : I. Some erasers are scales.
II. Some erasers are staplers.
(a)Only conclusion I is correct.
(b)Only conclusion II is correct.
(c) Neither I or II is correct.
Reasoning-Logical Deduction 143

(d)Both I and II are correct.


Ans. (c)
7. Statements : Some books are tyres.
Some tyres are cars.
Conclusions : I. Some buttons are tyres.
II. Some cars are books.
(a)Only conclusion I is correct.
(b)Only conclusion II is correct.
(c) Neither I or II is correct.
(d) Both I and II are correct.
Ans. (b)
8. Statements : All owls are ducks.
Some parrots are owls.
Conclusions : I. All owls are parrots.
II. All parrots are ducks.
III. Some parrots re ducks.
IV. Some ducks are not parrots.
(a)Only IV follows (b)Only III follows
(c) Only I follows (d)Only II follows
Ans. (d)
10. Statements : All lawyers are actors.
Some actors are painters.
Conclusions : I. Some actors are lawyers as well as
painters.
II. All painters are actors.
(a)Only conclusion I is correct.
(b)Only conclusion II is correct.
(c) Neither I or II is correct.
(d) Both I and II are correct.
Ans. (a)
qqq
19 REASONING-MACHINE
INPUT OUTPUT

INTRODUCTION
In input-output questions, a sequence of words, letters or
both is considered as an input and then this input is passed
through a processing machine or reorganized to give the
sequential outputs.
A candidate is required to trace the pattern used in
reorganizing the different elements of the input and
subsequently determine the required final output or last step,
elements' positions in some specified step etc.
Some of the specific types of patterns are as follow :
Type-1 Problems Based on Shifting
Shifting problem consists of sequence in which the
elements are shifted from one position to other according to a
certain set of rules/patterns.
To understand the concept of shifting, lets study some
examples.
Directions (Ex. No. 1-5) : Read the information
carefully and answer the questions given below. A word
arrangement machine, when given an input line of words,
rearranges them following a particular rule in each step. The
following is the illustration of the input and the steps of
arrangement.
Input : Gambhir retired but feel sorry for him.
Step I : But retired Gambhir sorry feel him for.
Step II : Retired feel sorry Gambhir for him for.
Step IV : Sorry retired but him feel Gambhir for.
And so on for subsequent steps. You have to find out the
logic and answer the questions given below.
(144)
Reasoning-Machine Input Output 145

1. If Step III reads ‘‘Good things come to those who wait’’,


then what will be the arrangement of the input :
(a)To who things good come wait those.
(b)Wait things good to come who those.
(c) Things good wait to come who those.
(d)None of the above.
Ans. (b)
Explanation : Given, Step III

Good Things Come To Those Who Wait


↓ ↓ ↓ ↓ ↓ ↓ ↓
3 2 5 4 7 6 1

∴ Input
1 2 3 4 5 6 7
↓ ↓ ↓ ↓ ↓ ↓ ↓
Wait Things Good to Come Who Those

2. If the given input is ‘‘sports give lots money & fame’’, then
what will be Step IV :
(a)And lots sports give of money fame.
(b)Of give lots money sports fame and
(c) Lots money of sports and give fame
(d)Money give lots fame of sports and
Ans. (d)
Explanation : Give, Input

Sports Give Lots Of Money And Fame


↓ ↓ ↓ ↓ ↓ ↓ ↓
1 2 3 4 5 6 7

∴ Step IV
5 2 3 7 4 1 6
↓ ↓ ↓ ↓ ↓ ↓ ↓
Money Give Lots Fame Of Sports And
146 Fundamentals of Reasoning

3. If Step I read ‘‘It was the name written on board’’, then


what would be the arrangement for Step III :
(a)It was name written on board the .
(b)Was it name written the board on.
(c) On the board was it written name.
(d)None of the above.
Ans. (a)
Explanation : Let reference chart be

Gambhir Retired But = 3 Feel = 4


Input 1 2 3 4 5 6 7
Step I 3 2 1 5 4 7 6
Step II 2 3 4 5 1 6 7
Step III 3 2 5 4 7 6 1
Step IV 5 2 3 7 4 1 6

Give Step I.

It Was The Name Writte On Board


n
↓ ↓ ↓ ↓ ↓ ↓ ↓
3 2 1 5 4 7 6

∴ Step III
3 2 5 4 7 6 1
↓ ↓ ↓ ↓ ↓ ↓ ↓
It Was Name Written On Board The
4. If Step IV read ‘‘weeks of little enthusiasm and weak
ideas’’, then what would Step III read :
(a)Ideas weeks and Little of weak enthusiasm.
(b)Of weeks and enthusiasm little ideas weak.
(c) Of little enthusiasm ideas weak and weeks.
(d)Little of weeks and enthusiasm ideas weak.
Ans. (d)
Explanation : Given, Step IV
Reasoning-Machine Input Output 147

Weeks Of Little Enthusiasm And Weak Ideas


↓ ↓ ↓ ↓ ↓ ↓ ↓
5 2 3 7 4 1 6

∴ Step III
3 2 5 4 7 6 1
↓ ↓ ↓ ↓ ↓ ↓ ↓
Little Of Weeks And Enthusiasm Ideas Weak
5. If Steep input is ‘‘workers must take a stand against
working’’, then what will be the last word of step III.
(a)Must (b)A (c) Workers (d)Take
Ans. (c)
Explanation :

∴ Step III
3 2 5 4 7 6 1
↓ ↓ ↓ ↓ ↓ ↓ ↓
Take Must Stand Working Against Workers Last word
Directions (Ex. no. 6-10) : Read the information
carefully and answer the questions given below. A word and
number arrangement machine when given a particular
input, rearranges it following a particular rule. The following
is the illustration of input and the steps of arrangement.
Input : 13 give not rat 45 33 nine
Step I : Nine give not 45 33 13
Step II : Nine not give rot 33 45 13
Step III : 13 not give rot 33 45 nine
Step IV : 13 give not rot 45 33 nine
Step V : Nine give not rot 45 33 13.
6. If input is 18 nine 36 ten two seven eight, then find Step
V.
(a)36 ten two seven 18 nine eight
(b)Eight nine 36 ten two seven 18
(c) 18 seven two ten 36 nine eight
(d)None of these
148 Fundamentals of Reasoning

Ans. (b)
Explanation : Given, Input

18 Nine 36 Ten Two Seven Eight


↓ ↓ ↓ ↓ ↓ ↓ ↓
1 2 3 4 5 6 7

7. Step III ‘29 39 49 59 95 tap map’. Find the input.


(a)Map 49 39 59 tap 95 29 (b)39 95 tap 59 49 29 map
(c) 29 49 39 59 tap 95 map (d)tap map 29 39 49 59 95
Ans. (c)
Explanation : Given, Step III

29 39 49 59 95 tap map
↓ ↓ ↓ ↓ ↓ ↓ ↓
1 3 2 4 6 5 7

∴ Input
1 2 3 4 5 6 7
↓ ↓ ↓ ↓ ↓ ↓ ↓
29 49 39 59 tap 95 map
8. We have Step V as ‘11 99 100 121 fine wine dine’, then
what would be the Step II :
(a)Wine fine dine 11 100 99 121
(b)Dine fine wine 11 100 99 121
(c) 11 100 99 121 wine fine dine
(d)121 99 100 11 dine fine wine.
Ans. (c)
Explanation : Given, Step V

11 99 100 121 Fine Wine Dine


↓ ↓ ↓ ↓ ↓ ↓ ↓
7 2 3 4 5 6 1

∴ Step II
7 3 2 4 6 5 1
↓ ↓ ↓ ↓ ↓ ↓ ↓
11 100 99 121 Wine Fine Dine
Reasoning-Machine Input Output 149

9. If ‘17 33 43 50 11 22 25’ is the input, then find out the Step


IV.
(a)43 17 11 50 33 22 25 (b)25 22 33 50 43 11 17
(c) 50 11 22 25 43 17 33 (d)17 33 43 50 11 22 25
Ans. (d)
Explanation : Given, Input

17 33 43 50 11 22 25
↓ ↓ ↓ ↓ ↓ ↓ ↓
1 2 3 4 5 6 7

Now, from reference chart, taking the digits for Step IV


and replacing them with numbers.

Step IV
1 2 3 4 5 6 7
↓ ↓ ↓ ↓ ↓ ↓ ↓
17 33 43 50 11 22 25
10. If ‘rat cat fat chat that hat mat’ is the Step II, then what
would be the input :
(a)Mat fat cat chat hat that rat
(b)Chat mat fat cat rat that hat
(c) Fat mat chat rat hat that
(d)That hat rat cat chat mat fat
Ans. (a)
Explanation : Given, Step II

29 39 49 59 95 tap map
↓ ↓ ↓ ↓ ↓ ↓ ↓
1 3 2 4 6 5 7

∴ Input
1 2 3 4 5 6 7
↓ ↓ ↓ ↓ ↓ ↓ ↓
29 49 39 59 tap 95 map

Directions (Ex. no. 11-15) : Study the following


information carefully and answer the given questions. A word
and rule in each step. The following is an illustration of input
150 Fundamentals of Reasoning

and rearrangement. (All the numbers given in the


arrangement are two digit numbers)
Input : ‘gone over 38 65 test 74 park 21
Step I : 21 gone over 38 65 test 74 park
Step II : 21 test gone over 38 65 74 park
Step III : 21 test 38 gone over 65 74 park
Step IV : 21 test 38 park gone over 65 74
Step V : 21 test 38 park 65 gone over 74
Step VI : 21 test 38 park 65 over gone 74
Step VII : 21 test 38 65 over 74 gone
and Step VII is the last step of the rearrangement of the above
input as the desired arrangement is obtained.
As per the rules followed in the above step, find out in
each of the following questions the following questions the
appropriate step for the given input.
11. Input ‘where 55 68 17 are they going 37’ which of the
following steps will be the last but one:
(a)VII (b)IV (c) V (d)III
Ans. (III)
Explanation : Input : Where 55 68 17 are they going 37
Step I : 17 where 55 68 are they going 37
Step II : 17 where 55 68 are they going 37
Step III : 17 where 37 they 55 68 are going
Step IV : 17 where 37 they 55 going 68 are
∴ Last but one step = Step III
12. Input ‘33 store 81 75 full of goods 52’ which of the
following will be Step III :
(a)33 store 52 81 75 full of goods
(b)33 store 52 of 75 81 full of goods
(c) 33 store 52 of 75 goods 81 full
(d)33 store 52 of 75 full 81 goods
Ans. (b)
Explanation : Input : 33 store 81 75 full of goods 52
Reasoning-Machine Input Output 151

Step I : 33 store 52 81 75 full of goods


Step II : 33 store 52 of 81 75 full goods
Step III : 33 store 52 of 75 81 full goods
13. Input ‘86 open shut door 31 49 always 45’. How many
steps will be required to complete the rearrangement :
(a)Seven (b)Six (c) Five (d)Four
Ans. (b)
Explanation : Input : 86 open shut down 31 49 always
45
Step I : 31 86 open shut door 49 always 45
Step II : 31 shut 86 open door 49 always 45
Step III : 31 shut 45 86 open door 49 always 45
Step IV : 31 shut 45 open 86 door 49 always
Step V : 31 shut 45 open 86 door 49 always
Step VI : 31shut 45 open 49 door 86 always
∴ Total steps = Six
14. Step II of an Input is 18 win 71 34 now if victory 61 How
many more steps will be required to complete the
rearrangement :
(a)Four (b)Six (c) Five (d)Three
Ans. (a)
Explanation : Step II : 18 win 71 34 now if victory 61
Step III : 18 win 34 71 now if victory 61
Step IV : 18 win 34 victory 71 now if 61
Step V : 18 win 34 victory 61 71 now if
Step VI : 18 win 34 victory 61 now 71 if
15. Step III of an input is 21 full 39 water joy defeat 65 46
Which of the following is definitely the input :
(a)Water 21 joy defeat full 65 39 46
(b)39 Water 21 joy full defeat 65 46
(c) Water joy defeat 21 65 full 46 39
(d)Cannot be determined
Ans. (d)
152 Fundamentals of Reasoning

Explanation : (b) In the arrangement problems,


previous steps/Input cannot be determined.
PRACTICE SET
Direction (Q. n. 1-5) : Read the information carefully
and answer the questions given below. A word
rearrangement machine when given an input line of words,
rearranges them following a particular rule in every step.
The following is an illustration of input and the steps of
rearrangement.
Input : Gavaskar's shots are really good in cricket
Step I : Gavaskar's in are really good shots cricket
Step II : Really are in Gavaskar's cricket shots good
Step III : In are really good shots cricket Gavaskar's
Step IV : In cricket really good shots are Gavaskar's
Step V : Good really cricket in Gavaskar's are shots.
As per the pattern followed in the above steps. Find the
appropriate step for the given input or vice-versa in questions
given below.
1. Input ‘team sudden now to act police staff’. Which of the
following steps would be ‘police staff to act sudden now
team :
(a)Step III (b)Step II (c) Step IV (d)Step I
Ans. (c)
2. Input ‘Most young people are interested in politics’.
Which of the following will be the step IV for the given
input :
(a)Interested are young the politics people in .
(b)People interested young most are politics in.
(c) In politics are interested young people most.
(d)None of the above.
Ans. (c)
3. If Step IV of an input is ‘none of the musicians have
toured Nepal’, find the Step II.
(a)Nepal have musicians none of the toured
Reasoning-Machine Input Output 153

(b)Musicians the of none Nepal toured have


(c) Have the none musicians toured Nepal of
(d)Have musicians none the toured Nepal of
Ans. (b)
4. If Step I of an input is ‘right do a two make not wrong’ find
out the step of that input
(a)Make two wrongs do right a not
(b)Make a wrongs do not night two
(c) Wrongs two a not right make fo
(d)None of the above
Ans. (a)
5. If Step III of an input is none of them is happy at all’, then
which of the following will definitely be the input :
(a)Happy all at of non is them
(b)All happy none is them at
(c) Them is none of a t all happy
(d)None of these
Ans. (d)
Directions (Q. no. 6-10) : Study the following
information carefully and answer the given questions. A word
and number arrangement machine when given an input line
of words and numbers rearranges them following a particular
rule in each step. The following is an illustration of input and
rearrangement.
Input : ‘gone 87 over 51 73 now for 25
Step I : 25 gone 87 over 51 73 now for
Step II : 25 over gone 87 51 73 now for
Step III : 25 over 51 gone 87 73 now for
Step IV : 25 over 51 now gone 87 73 for
Step V : 25 over 51 now 73 gone 87 for
and Step V is the last step of the rearrangement of the above
input.
154 Fundamentals of Reasoning

As per the rules followed in the above steps. Find out in


each of the following questions the appropriate step for the
given input.
6. Input ‘station hurry 39 67 all men 86 59’ How many steps
will be required to complete the rearrangement :
(a)Three (b)Five (c) Four (d)Six
Ans. (b)
7. Input ‘news 79 53 glory for 46 29 task’ Which of th
following will be step III :
(a)29 task 46 news 79 53 glory for
(b)29 news 79 53 glory for 46 task
(c) 29 task 46 news 53 glory 79 for
(d)29 task 46 news 53 79 glory for
Ans. (a)
8. Step III of an input 15 window 29 93 86 sail tower buy
Which of the following will be Step VI :
(a)15 window 29 tower 86 93 sail buy
(b)15 window 29 tower 93 86 sail buy
(c) 15 window 29 tower 86 sail 93 buy
(d)There will be no such step
Ans. (c)
9. Step II of an input is 19 years 85 74 near gone 26 store
How many more steps will be required to complete the
rearrangement :
(a)Three (b)Five (c) Two (d)Four
Ans. (d)
10. Step III of an input is 27 tube 34 gas chamber row 74 53
Which of the following steps will be the last but one :
(a)V (b)VI (c) VIII (d)VII
Ans. (a)
qqq
20 REASONING-
MATHEMATICALOPERATIONS
Type-2 Problems Based on Arrangement
In such problems, words or numbers or both are
arranged. Words are arranged alphabetically while numbers
are arranged in their increasing or decreasing order.
Under this segment of reasoning, the four mathematical
operations viz. addition, subtraction, multiplication and
division are represented by unusual symbols. You may say
such symbols as artificial ones. The candidate is required to
substitute the real signs in place of artificial symbols to solve
the questions.
VBODMAS RULE
While simplifying a mathematical problem, one must
follow ‘VBODMAS’ rule. Order of various operations is as
same as the order of letters in the ‘VBODMAS’ from left to
right.
V B O D M A S

Left to right

In this chapter, a candidate is provided with substitutes


for various mathematical symbols, followed by a question
involving calculation of an expression or choosing the
correct/incorrect equation. The candidate is required to put in
the real signs in the given equation and them solve the
questions as required.
1. If ‘+’ means ‘×’, ‘–’ means ‘+’, ‘×’ means ‘÷ ’ ‘÷’ means ‘–’, then
23 – 3 ÷ 1 + 15 × 5 is equal to
(a)75 (b)23 (c) 17 (d)58
Ans. (b)

(155)
156 Fundamentals of Reasoning

Explanation : If the actual operators will be replaced as


per the direction of the question, then the series will be–
= 23 + 3 − 1 × 15 ÷ 5 (using VBODMAS rule)
= 23 + 3 − 1 × 3
= 23
2. If + means –, × means ÷, – means + and ÷ means × then = ?
(a)87 (b)726 (c) 111 (d)129
Ans. (c)
Explanation :
= 123 − 8 ÷ 4 × 6
= 123 − 2 × 6
= 123 − 12
= 111
So, option (c) is correct.
3. If + means–, × means ÷, – means + and ÷ means × then
45 × 5 ÷ 3 − 15 = ?
(a)30 (b)42 (c) 18 (d)19
Ans. (b)
Explanation : 45 ÷ 5 × 3 + 15 = 9 × 3 + 15 = 27 + 15 = 42.
4. If + means –, × means ÷, – means + and ÷ means ÷ then
88 × 8 ÷ 3 − 24
(a)88 (b)35 (c) 57 (d)30
Ans. (c)
Explanation : = 88 ÷ 8 × 3 + 24
= 11 × 3 + 24 = 33 + 24 = 57
5. If + means –, × means ÷, – means + and ÷ means × then
167 + 45 − 4 + 7 ÷ 2
(a)112 (b)171 (c) 18 (d)238
Ans. (a)
Explanation : = 167 − 45 + 4 − 7 × 2
= 167 − 45 + 4 − 14
= 171 − 59 = 112
Reasoning-Mathematical Operations 157

6. If ‘×’ stands for ‘+’, + stands for ‘–’ means ‘×’ means ‘+’,
then 12 + 6 ÷ 3 − 2 × 8 = ?
(a)–2 (b)4 (c) 2 (d)8
Ans. (d)
Explanation : = 54 ÷ 16 − 3 × 6 + 2
= 54 − 16 × 3 + 6 ÷ 2
= 54 − 16 × 3 + 3
= 54 − 48 + 3
= 6+ 3 = 9
7. If A means ‘+’, B means ‘–’ C means ‘×’ and D means ÷,
then 13 C 14 A 6 B 24 D 8 = ?
(a)188 (b)164 (c) 166 (d)185
Ans. (d)
Explanation : = 13 × 14 + 6 − 24 ÷ 8
= 13 × 14 + 6 − 3
= 182 + 6 − 3
= 188 − 3
= 185
8. If ‘+’ means subtraction, ‘÷’ means addition, ‘–’ means
multiplication and ‘×’ means division, then which of the
following equation is correct :
(a)9 ÷ 64 − 6 × 2 = 54 (b)132 × 44 − 12 + 10 = 46
(c) 8 ÷ 44 − 5 + 25 = 203 (d)56 + 12 × 34 − 12 = 102
Ans. (c)
Explanation :
(a)= 9 + 64 × 6 ÷ 2 = 9 + 64 × 3 = 201 = 201 ≠ 54
(b)= 132 ÷ 44 × 12 − 10 = 3 × 12 − 10 = 36 − 10 = 26
= 26 ≠ 46
(c) = 8 + 44 × 5 − 25 = 8 + 220 + 25 = 228 − 25 = 203
= 203 = 203
So, option (c) is correct and there is no need to check
remaining options.
158 Fundamentals of Reasoning

9. If + means ÷, × means +, – means × and ÷ means – then


182 + 13 × 45 ÷ 65 − 2 × 95
(a)284 (b)24 (c) 405 (d)35
Ans. (b)
Explanation : = 182 ÷ 13 + 45 − 65 × 2 + 95
= 14 + 45 − 65 × 2 + 95
= 14 + 45 − 130 + 95 = 24
10. If ‘@’ means ‘×’, ‘c’ means ‘+’, % means (+) and ‘$’ means ‘–’,
then 9% 18 C 3@ 7 $ 15 = ?
(a)36 (b)35 (c) 39 (d)33
Explanation : = 9 + 18 ÷ 3 × 7 − 15
= 9 + 6 × 7 − 15
= 9 + 42 − 15 = 51 − 15 = 36
PRACTICE SET
Directions (Q. no. 1-4) : If ‘+’ is ‘×’ is ‘÷’ and ‘+’ is ‘–’, then
answer the following questions based on this information.
1. {6 + 7 × 3} – 8 ÷ 20 = ?
(a)–3 (b)6 (c) 2 (d)1
Ans. (b)
2. 11 – 3 + 2 ÷ 16 × 2 = ?
(a)7 (b)5 (c) 9 (d)6
Ans. (c)
3. 30 ÷ 8 + 3 – 12 × 6 = ?
(a)14 (b)9 (c) 8 (d)11
Ans. (c)
4. 6 – 9 + 8 × 3 ÷ 20 = ?
(a)10 (b)6 (c) 12 (d)–2
Ans. (a)
5. If ‘+’ means ‘divided by’, ‘–’ means ‘added to’, ‘×’ means
‘subtracted from’ and ‘÷’ means ‘multiplied by’, then
19 × 20 + 5 − 18 ÷ 2 = ?
(a)216 (b)31 (c) 21 (d)51
Ans. (d)
Reasoning-Mathematical Operations 159

6. If ‘P’ denotes ‘–’, ‘Q’ denotes ‘÷’, ‘R’ denotes ‘×’ and ‘W’
denotes ‘+’, then 48 Q 12 R 10 P P 8 W 4 = ?
(a)56 (b)44 (c) 52 (d)36
Ans. (d)
7. If ‘+’ means ‘–’, ‘+’ means ‘+’, ‘–’ means ‘×’ and ‘×’ means ‘+’,
then which of the following equation is correct :
(a)4 − 10 + 10 × 5 = 38 (b)65 + 11 − 56 × 14 = 22
(c) 66 × 3 − 11 + 12 = 230 (c) 2 − 14 × 4 ÷ 11 = 16
Ans. (c)
8. If + means –, × means ÷, – means + and ÷ means × then
108 × 9 − 69 + 27 ÷ 2
(a)27 (b)849 (c) 3 (d)231
Ans. (a)
9. If K denotes ‘×’, ‘B’ denotes ‘÷’, ‘T’ denotes ‘–’ and ‘M’
denotes ‘+’, then 40 B 8 T 6 M 3 K 4 = ?
(a)–7 (b)11 (c) 19 (d)23
Ans. (b)
10. If ‘–’ stands for ‘÷’ ‘+’ stands for ‘×’, ‘+’ for ‘–’ for ‘×’ for ‘+’,
which one of the following equations is correct :
(a)30 ÷ 6 × 5 + 4 − 2 = 36 (b)30 + 6 − 5 ÷ 4 × 2 = 38
(c) 30 × 6 ÷ 5 − 4 + 2 = 32 (d)30 − 6 + 5 × 4 ÷ 2 = 27
Ans. (a)
qqq
21
REASONING-CLASSIFICATION

INTRODUCTION
In this chapter, the questions contain a set of different
items and it is asked to find the odd item out of the given ones.
All the items except one, follow a certain rule or pattern
or they possesses some common quality or characteristics
between them and one which is odd does not possesses the
common quality or characteristics.
These types of problems are classified into 3 categories.
That are-
l Alphabet Classification
l Word Classification
l Number Classification
Let's discuss about the above types of classifications.
Alphabet Classification : A single letter or group/pair
of letters are given in each of four options or five options.
Three or four of them are similar to each other in same
manner while one is different and this is to be chosen by the
candidate as the answer.
1. Find the odd letter.
(a)m (b)s (c) e (d) y
Ans. (c)
Explanation : Except ' e' all are consonants.
2. Find the odd letter.
(a)z (b)R (c) J (d)P
Ans. (a)
Explanation : All except ' z' are made with combination
of straight and curvex lines.
3. Find out the odd pair.
(a)JG (b)TQ (c) NL (d)ZW
(160)
Reasoning-Classification 161

Ans. (c)
Explanation : So the answer will be NL, which is choice
(c)
J T N z
↓ −3 ↓ −3 ↓ −3 ↓ −3
G Q K w
4. Which of the following is the odd letters group?
(a)DFH (b)JLN (c) PRT (d)VWX
Ans. (d)
Explanation : All except 'VWX ' have a difference of two
positions between first and second and second and third
letters.
E F HJ L N P R T V W X

+2 +2 +2 +2 +2 +2 +1 +1
5. Find the odd one.
(a)FDH (b)KGM (c) MOQ (d)YWA
Ans. (b)
+2 −2
Explanation : FDH → F → H , F → H
+2 −2
KGM → K → M , K → I
+2 −2
MOQ → O → Q , O → M
+2 −2
YWA → W → A , W → Y
2. Word Classification : In this type, different items
which belong to common properties like places, area of usage,
number, etc. are present. The one which doesn't match to that
category will be the odd one.
6. Find the odd word
(a)Venus (b)Moon (c) Jupiter (d)Saturn
Ans. (b)
Explanation : All except moon are planets.
162 Fundamentals of Reasoning

7. Three of the following four are same in a certain way and


hence form a group. Find out the one which does not
belong to that group.
(a)Red (b)Violet (c) Green (d)Black
Ans. (d)
Explanation : Except black, all others are colours of
rainbow.
8. Find the odd word.
(a)Cuboid (b)Cylinder (c) Triangle (d)Sphere
Ans. (c)
Explanation : All except Triangle are
three-dimensional figures.
9. Find the odd word.
(a)Zurich (b)Ngultrum (c) Ruble (d)Dinar
Ans. (a)
Explanation : (a) All others are Currencies, while
Zurich is a city in Switzerland.
10. Find the odd word.
(a)Apartment (b)House
(c) Cottage (d)Factory
Ans. (d)
Explanation : All others are dwelling places of human
being, while 'Factory' is a working place.
3. Number Classification : Number classification
means a group of numbers which follows the same pattern. In
this case we will see that there are numbers given in question
from which we have to find the odd one. The numbers may
belong to a particular set i.e., they may be odd, even, prime,
rational, cubes, squares, coded binary digits etc. One choice
will not follow the rule and that will be our answer.
11. Find the number different from others.
(a)476 (b)121 (c) 181 (d)315
Ans. (c)
Explanation : All except 181 are non-prime numbers.
Reasoning-Classification 163

12. Find the odd one.


(a)169 (b)343 (c) 64 (d)512
Ans. (a)
Explanation : All except 169 are cubes of natural
numbers.
13. Find the odd among the following.
(a)1011 (b)1101 (c) 1010 (d)10001
Ans. (d)
Explanation : These numbers follow the binary coding.
Let's convert them into decimal.
(a) 10001 = 1 × 2 4 + 0 × 2 3 + 0 × 2 2 + 0 × 21 + 1 × 2 0
= 16 + 0 + 0 + 0 + 1 = 17
(b) 1011 = 1 × 2 3 + 0 × 2 2 + 1 × 21 + 1 × 2 0
= 8 + 0 + 2 + 1 = 13
(c) 1101 = 1 × 2 3 + 1 × 2 2 + 0 × 21 + 1 × 2 0
= 8 + 0 + 2 + 0 = 11
(d) 1111 = 1 × 2 3 + 0 × 2 3 + 1 × 21 + 1 × 2 0
= 8+ 0+ 2+ 1
= 11
Here, (d) will be the answer because 10 is not a prime
number but all others are prime numbers.
14. Find the odd group.
(a)4, 16, 64 (b)5, 25, 100
(c) 7, 49, 343 (d)3, 9, 27
Ans. (b)
Explanation : In all options except (b), second & third
numbers are the square and cube of first number
respectively.
15. Find the odd one
(a)706 (b)282 (c) 931 (d)445
Ans. (b)
Explanation : All except 282 have sum of digits = 13
164 Fundamentals of Reasoning

PRACTICE SET
Directions : In each of the following questions, four
options are given. Out of these, three are alike in particular
way but remaining one is different. Find the odd one from the
given options which is different from others.
1. (a)Bat (b)Eagle (c) Sparrow (d)Duck
Ans. (a)
2. (a)Adequate (b)Capacious
(c) Abundance (d)Ample
Ans. (c)
3. (a)U (b)C (c) K (d)S
Ans. (c)
4. (a)96 (b)74 (c) 48 (d)12
Ans. (b)
5. (a)5, 6, 26 (b)38, 39, 59
(c) 113, 114, 134 (d)21, 22, 12
Ans. (d)
6. (a)DI (b)KO (c) QV (d)SX
Ans. (b)
7. (a)FJL (b)JMP (c) TWZ (d)PSV
Ans. (a)
8. (a)Cement (b)Bricks (c) Sand (d)Bungalow
Ans. (d)
9. (a)TEETH (b)CHEEK (c) SPEED (d)WHEEL
Ans. (a)
10. (a)2019 (b)3512 (c) 2604 (d)3341
Ans. (c)
qqq
22 REASONING-PASSAGE
AND CONCLUSIONS

INTRODUCTION
This particular section of reasoning demands a thorough
reading of a passage with a clear understanding of the subject
and then judging the truth of certain pre-given statements
(conclusions) based upon your vision of understanding. The
passage based conclusions have scope for probability of
truthfulness or falsity. This is the reason why these
conclusions may be definitely true or definitely false or
probably true or probably false.
Explanation of all these four types of conclusions will
give a better idea about how to draw such inference.
Passage-1
Doordarshan has undoubtedly helped popularize yoga
among the people especially city dwellers but performing
various 'asanas' without adequate instructions could be
hazardous, according to a renowned 'yoga expert' who has
conducted hundreds of yoga shivirs (camps).
He says several people, who tried to learn yoga through
the programme of Doordarshan, have come to him
complaining of pain in different parts of their body. He
explains that the asanas involve a complicated but scientific
technique of breathing which controls the flow of oxygen in
various parts of the body under calculated stress.
1. Uncontrolled flow of oxygen in parts of the body causes
pain in those parts.
Ans. (b)
Explanation : Conclusion is probably true because
uncontrolled flow of oxygen in various parts of body
causes pain in these parts.
(165)
166 Fundamentals of Reasoning

2. Yoga exercises (asanas) are quite safe for any person.


Ans. (e)
Explanation : It is given in the passage that without
adequate instructions, yoga could be hazardous. Hence,
the conclusion is definitely false.
3. Quite a large number of 'yoga experts' are available.
Ans. (c)
Explanation : Data is inadequate, as it is not given in
the passage that quite a large number of yoga experts are
available.
4. TV programmes on yoga have successfully replaced
personal guidance by 'yoga experts'.
Ans. (e)
Explanation : Definitely false. It is given in the passage
that persons doing asanas complain of pains. Hence, it is
definitely false to say that TV programmes on yoga have
successfully replaced personal guidance by the yoga
expert.
5. Many TV viewers of yoga programmes are unable to
master the scientific technique of breathing while
performing 'asanas'.
Passage-2
In the present scenario, the idea of entrepreneurship is
getting wide spread enormously. This does not mean that
many people are getting attracted to start their own business
but it is due to the fact that the job market is getting narrow.
The number of unemployed engineering graduates are
getting multiplied every year. Now is the time to create a
start up and to do something new. But this requires a clear
visionary and a lot of patience followed by hard work. Prime
minister's Make in India and Made in India campaign are
also aiding fuel to the whole process. If such initiatives will
get right direction and acclaim, then soon India can eradicate
the black patch of unemployment from its head.
6. The present situation shows the number of people getting
jobs is more than that of previous years.
Reasoning-Passage and Conclusions 167

Ans. (e)
Explanation : The passage is clearly talking about the
problems of unemployment. This clearly indicates that
the number of jobless people has increased than before.
So, we can conclude that the given statement is definitely
false.
7. Engineering graduates are only the worst sufferers of
this situation.
Ans. (c)
Explanation : It is given in the statement that the
number of unemployed engineers is getting multiplied.
But the passage has not told anything specific that
whether only the engineers are the worst hit of the
situation or people with some other profession also facing
heat. So, this indicates that the statement may be true or
false. This is because it needs some extra statement to
clarify.
8. Entrepreneurship can be seen as an ultimate solution of
this problem.
Ans. (a)
Explanation : Yes; because as the job market is getting
narrow and the number of unemployed people is
increasing, entrepreneurship can solve both these
problems by creating enough job market. People can
generate job and other people can get jobs easily.
9. Make in India campaign will definitely influence all
people to start entrepreneurship.
Ans. (b)
Explanation : Make in India is a campaign that aims at
producing more entrepreneurs according to the passage
but it's success solely depends upon the way government
will promote and market this. If the exact message can be
successfully derived to each corner of the country then it
may help. So, it can be probably true but not definitely
true.
10. Clear vision and hard work are necessary to become a
successful entrepreneur.
168 Fundamentals of Reasoning

Ans. (a)
Explanation : Clearly these are the words that are
clearly mentioned in the passage. So, we can clearly
conclude that hard work and clear vision are necessary to
establish yourself as a successful entrepreneur.
PRACTICE SET
Directions : (Q1–10) Questions in the form of
inferences/conclusions are based on the passages given
below. Each passage is followed by five inferences. You are
required to examine each inference separately in the context
of the passage and decide upon its degree of truth or falsity.
Give answer :
(a) If the inference is definitely true,
(b) If the inference is probably true though not definitely
true.
(c) If the data are inadequate.
(d) If the inference is probably false though not
definitely false.
(e) If the inference is definitely false.
1. Meditation is the best way of getting closer to god.
Ans. (a)
2. Excess meditation can cause cancer.
Ans. (d)
3. Like school and universities, office also should make
meditation compulsory.
Ans. (c)
4. Control of breathing is not the only simple way of
meditation, there are other ways of doing simple
meditation too.
Ans. (b)
5. Homeopathy is not as good as allopathic.
Ans. (d)
Reasoning-Passage and Conclusions 169

Passage-2
Since ages there have been many debates over the
treatment of disease with homeopathy. For many MBBS
doctors, till now, homeopathy is just a taboo. They say, it is
only there to cure soft disease like cold and cough but cannot
even treat a deadly disease like cancer. However; many
doctors in homeopathy sector have claimed to cure blood
cancer patients through successful treatment. "It will take
some time but will eradicate the disease from its root level."
-says Dr. P Banerji, a renowned homeopathy doctor of
Kolkata. With changing technology many new drugs have
been developed in homeopathy that is acting towards the cure
of some disease those have no cure in allopathic.
6. Brain tumour treatment is not possible through
homeopathy.
Ans. (c)
7. People can be optimistic about homeopathy.
Ans. (a)
8. MBBS doctors admire the treatment process of
homeopathy.
Ans. (e)
Passage-3
Domestic steel industry has been going through
challenging times with raw material prices rising unabated
and Government trying to cap final product (steel) prices in
order to keep inflation under check. Notably, the Government
has taken several measures in the past six months to keep a
check on steel prices. Which contribute around 3.63% of WPI.
Now, after holding prices for three months the battle between
the Government and steel players has erupted again.
With the anticipation of players increasing prices very
soon, Government is trying to counter this with the
imposition of a price band on steel products. Imposition of
price band may unfairly treat the domestic steel industry as
global steel prices are ruling at 30% premium to domestic
170 Fundamentals of Reasoning

prices. Global prices have increased by 50% to 60% in 2008 as


compared to just 20% rise in the domestic market.
9. Price of steel is an integral part of the Wholesale Price
Index (WPI) of India.
Ans. (a)
10. There has been a decline in the rate of inflation in recent
months in India.
Ans. (d)
qqq
23
REASONING-PUZZLES

INTRODUCTION
In puzzles the candidates are provided with the
information in jumbled or haphazard format. It checks the
candidate's ability (both mental and analytical) to decipher,
sequence and analyze the given information into a
meaningful and judgmental form, so as to come to the final
decision or conclusion by following the systematic pattern of
linking and interlinking one or several informations with
each other.
This segment is considered to be the most difficult part of
reasoning as there exists no set pattern or formulae to solve
such problems.
In this chapter, the candidates re tested in the area of
classification type questions, comparison type questions,
sitting arrangement questions and many types of jumbled
pattern.
In some type of questions, instructions regarding
belongingness or non-belongingness of some objects or
persons with some other objects or persons is given and on the
basis of analysis of these instructions, the questions have to
be answered.
Following examples will give you a better idea about the
questions asked.
Directions : (Ex- 1 – 4) Study the following information
carefully and answer the questions given blow.
Group of girls gossip with each other. All are sitting
around a circular table facing the centre. The name of the
girls are Shikha, Radha, Megha, Jyoti and Rani. It is not
necessary that they are sitting in the order of the name as
(171)
172 Fundamentals of Reasoning

mentioned here. Radha is second to the right of Shikha does't


sit with Megha. Rani is second to the right of Radha. Radha is
an immediate neighbor of Jyoti.
1. Who is sitting to the left of Shikha?
(a)Megha (b)Radha (c) Jyoti (d)Rani
Ans. (d)
Explanation : After observation, we can conclude that
the sitting arrangement is like this-
Rani

Shikha Megha

Jyoti Radha

So, Rani sits to the left of Shikha.


2. If Radha and Jyoti change their places then who will be
second to the left of Rani?
(a)Radha (b)Jyoti (c) Shikha (d)None
Ans. (b)
Explanation : Second to the left of Rani will be Jyoti.
Hence, (b) is correct.
3. How many girls are there in between Shikha and Megha
if we count anti clockwise?
(a)2 (b)3 (c) 1 (d)None
Ans. (a)
Explanation : Only two girls are there in between
shikha and Megha if we count anti clockwise.
4. What is the position of Megha with respect to Jyoti ?
(a)Third to the right (b)Immediate right
(c) Second to the left (d)Third to the left
Ans. (d)
Following table can be drawn from the given
information.
Reasoning-Puzzles 173

Name Gulab Barfi Ladoo Peda


Sanjay P P O O
Charu O O P O
Lata P P O P
Tammay P O O P
Anuj P P O O

Directions : (Ex- 5 – 7) Read the information carefully


and then answer the questions given below. Sanjay, Charu,
Lata, Tanmay and Anuj help themselves to take some sweets
from bowl. Four of them each take a gulab jamun. Charu and
Tammy do not take a barfi as all the others do. Infact, Charu
takes only one sweet, which is a ladoo. Ladoo is not taken by
by others apart from Charu, only Sanjay and Anuj do not take
peda.
5. Who takes three sweets?
(a)Charu (b)Anuj (c) Sanjay (d)Lata
Ans. (d)
Explanation : Lata took gulab jamun, barfi and peda.
6. Who only had a peda and a gulab jamun?
(a)Charu (b)Anuj (c) Sanjay (d)Tanmay
Ans. (c)
Explanation : Tanmay took a peda and gulab janum.
7. Who are the two people taking the same number and
same type of sweets?
(a)Sanjay and Anuj (b)Sanjay and Tanmay
(c) Tanmay and Anuj (d)Anuj and Charu
Ans. (a)
Explanation : Sanjay and Anuj took gulab jamun and
barfi. From the above table, it clear that
Directions (ex. 8 – 10) Examine carefully the following
statements and answer the three items that follow.
Out of four friends P, Q, R and S
174 Fundamentals of Reasoning

(i) P and Q play football and chess


(ii) Q and R play chess and badminton.
(iii) P and S play basketball and football.
(iv) R and S play badminton and basketball.
Following table can be drawn with the help of given
information.
Foot ball Chess Badminton Basketball
P P P O P
Q P P P O
R O P P P
S P O P P
8. Who does not play Chess ?
(a)Q (b)S (c) R (d)P
Ans. (b)
Explanation : From the above table, it is clear that S
does not play Chess.
9. Who plays football, basketball and Chess?
(a)R (b)P (c) Q (d)S
Ans. (b)
Explanation : From the above table, it is clear that P
plays football, basketball & chess.
10. Which game do Q, R and S play?
(a)Chess (b)Football
(c) Basketball (d)Badminton
Ans. (d)
Explanation : From the above table, it is clear that Q, R
and S play badminton.
PRACTICE SET
1. A, B, C, D and E belong to five different cities P, Q, R, S
and T (not necessarily in that order). Each one of them
comes from a different city. Further it is given that
(I) B and C do not belong to Q.
Reasoning-Puzzles 175

(II)B and E do not belong to P and R.


(III) A and C do not belong to R, S and T.
(IV) D and E do not belong to Q and T.
Which one of the following statements is not correct?
(a)C belongs to P (b)D belongs to R
(c) A belongs to Q (d)B belongs to S
Ans. (d)
Directions (Q. N. 2–3 ) Read the following information
carefully to answer these questions.
Five friends namely Alia, Geeta, Vasu, Ramesh and
Nishant have very good characteristics and are being
considered for various awards. Geeta, Alia and Vasu are
sincere. Alia, Ramesh and Nishant very brave. Ramesh, Vasu
and Nishant are very truthful. Alia, Geeta and Nishant are
courteous.
2. Which of the following persons is neither brave nor
courteous?
(a)Vasu (b)Alia (c) Ramesh (d)Geeta
Ans. (a)
3. Which combination of friends is not 'brave' but 'sincere'?
(a)Nishant and vasu (b)Vasu and Ramesh
(c) Geeta and Vasu (d)Geeta and Alia
Ans. (c)
Directions : (Q. n. 4 – 7) Read the given information
carefully and answer the question that follow.
There are five friends named as A, B, C, D and E all
working in various shops as stationary shop. book store,
grocery shop, hardware shop, sports goods shop. Also all like
to play different games such as hockey, kabbadi, basketball,
tennis and football. Consider the following statements and
answer the questions.
l A does not like hockey
l B has a book store and likes to play football.
l C and D do not like tennis and C has grocery shop
whereas D has hardware shop.
176 Fundamentals of Reasoning

l E likes kabbadi and does not work in stationary


shop.
l The person who likes hockey does not work in
hardware shop.
4. Who has stationary shop?
(a)A (b)C (c) D (d)B
Ans. (a)
5. Which game is played by D?
(a)Kabbadi (b)Hockey
(c) Basketball (d)Tennis
Ans. (a)
6. Which game is played by grocery shopkeeper?
(a)Hockey (b)Tennis (c) Kabbadi (d)Football
Ans. (a)
7. Which of the following option is true according to the
given statements?
(a)A-book store-tennis (b)C-grocery-hockey
(c) D-sports goods shop (d)B-stationary-kabbadi
Ans. (b)
Directions : (Ex. 8 – 10) : Study the following
information carefully to answer the given questions-
A, B, C, D, F and G are travelling in three buses X, Y, Z
with at least two of them in any of these buses. Each of them
likes a different sweet viz. Petha, Rasmalai, Rasgulla, Kheer,
Sandesh, Chamcham and Jalebi, but not necessarily in the
same order.
l B is travelling in bus Y will E.
l E's favourite seat is Jalebi.
l Those who travel in Bus X do not like Petha and
Kheer.
l The one who likes Rasmalai travels only with G in
Bus Z.
l The one whose favourite sweet is Sandesh does not
travel in the same bus with either E or G.
Reasoning-Puzzles 177

l A does not travel in Bus Y.


l A likes Chamcham.
l D and F are travelling in the same bus.
l F does not like Rasgulla.
l The one whose favourtie sweet is Kheer does not
travel in bus Y.
8. In which bus three of them are travelling?
(a)Only Z (b)Only X
(c) Only Y (d)Either X or Y
Ans. (b)
9. What is F's favourite sweet?
(a)Chamcham (b)Rasmalai
(c) Sandesh (d)Jalebi
Ans. (c)
10. Which of the following combinations is correct?
(a)G-X-Kheer (b)E-Z- Jalebi
(c) A-Y-Chamcham (d)D-X-Rasgulla
Ans. (d)
qqq
24 REASONING-MISSING
CHARACTERS

INTRODUCTION
Inserting a missing character problems are generally
asked in various competitive exams, which mainly consist of a
figure divided into various regions. In between these regions,
there will be numbers, letters or it may be the combination of
letters and numbers following certain sequence.
Such number or letters are arranged inside the figure
according to a certain pattern (i.e., based on particular logic
and/or mathematical calculations) but in such arrangement
one character is missing which is denoted by question mark
(?). The candidate is required to find out the character that
can replace the question mark (?) satisfying the logic and
calculations
TYPE-1 NUMBER PUZZLES
In such type of problems, some numbers are arranged in
a certain pattern, which can follow the rule of addition,
subtraction, multiplication, division, squaring etc. Based on
these different operations we have to find the missing
character in the given problems.
Directions (Ex. No. 1-9) : Find the missing character
from the given alternatives.
1.
48 55

? 62

(a)73 (b)49 (c) 53 (d)69


Ans. (d)
Explanation : Moving clockwise,
48 + 7 = 55 and 55 + 7 = 62

(178)
Reasoning-Missing Characters 179

∴ ? = 62 + 7 = 69
Hence, option (d) is correct answer.
2. 33
29
?
23 26
24
(a)36 (b)38 (c) 35 (d)39
Ans. (b)
Explanation : Moving anticlockwise:
23 + 1 = 24
24 + 2 = 26
26 + 3 = 29
29 + 4 = 33
33 + 5 = 38
Hence, answer is (b)
3. 7
162 12

82 22
?

(a)44 (b)49 (c) 42 (d)46


Ans. (c)
Explanations : 7 × 2 − 2 = 12, 12 × 2 − 2 = 22,
22 × 2 − 2 = 42, 42 × 2 − 2 = 82
Hence, answer is (c)
4. 15 16 9
5 6 3 12 24 6 34 ? 11

23 28 23

(a)36 (b)40 (c) 45 (d)53


Ans. (d)
Explanation : As, (5 × 15) − (23 × 3) = 75 − 69 = 6
and (12 × 16) − (28 × 6) = 192 − 168 = 53
Similarly (34 × 9) − (23 × 11) = 306 − 253 = 53
Hence, answer is (d)
180 Fundamentals of Reasoning

5.
289 13

169 ?

(a)17 (b)11 (c) 15 (d)19


Ans. (a)
Explanation : The numbers have their in diagonally
opposite space.
6. 5 7 12 15 39 18

4 9 ?

(a)15 (b)17 (c) 19 (c) 21


Ans. (c)
Explanation : The number at the bottom is obtained as,
5+ 7 12 + 15 38 + 18
= 4 and = 9, similarly = 19. Therefore
3 3 3
missing number is 19.
7. 8 5 13
2 7 6 3 21 36 4 ? 101

32 45 91

(a)34 (b)20 (c) 43 (d)29


Ans. (a)
Explanation : The numbers are obtained as
as (32 ÷ 8) + (6 ÷ 2) = 4 + 3 = 7
and (45 ÷ 5) + (36 ÷ 3) = 9 + 12 = 21
Similarly (91 ÷ 13) + (108 ÷ 4 ) = 7 + 27 = 34
8. 2 8 7

5 3 6

29 13 ?

(a)90 (b)89 (c) 68 (d)85


Ans. (d)
Explanation : Number are obtained as, 2 2 + 5 2 = 29 and
8 2 + 3 2 = 73, therefore missing number is 7 2 + 6 2 = 85.
Reasoning-Missing Characters 181

9. 16 324 81

64 196 9 25 256 196

36 169 361

(a)115 (b)68 (c) 139 (d)61


Ans. (b)
Explanation :
As, ( 16 + 196 + 36 + 64 ) = 4 + 14 + 6 + 8 = 32
and ( 324 + 25 + 169 + 9 ) = 18 + 5 + 13 + 3 = 39
Similarly ( 81 + 576 + 361 + 256 ) = 9 + 24 + 19 + 16
= 68
10. 6 8 7
2 9 13
20 28 ?
4 7 9
8 4 11

(a)77 (b)40 (c) 117 (d)63


Ans. (b)
Explanation : As, 2 + 6 + 8 + 4 = 20
and 8 + 9 + 4 + 7 = 28
Similarly, 13 + 11 + 7 + 9 = 40
TYPE-2 LETTER PUZZLES
In this type of problems, the arrangement of the letters of
the English alphabet is done according to a certain pattern
that may involve addition/subtraction of the positional values
of letters. The candidate is required to find out the rule and
the missing character from the arrangement based on that
rule.
Directions (Ex. no. 11–13) : In each of the following
questions, which character when placed at the sign of
interrogation (?) shall complete the given pattern.
11. ? D
N H
J L

(a)R (b)S (c) T (d)P


Ans. (a)
182 Fundamentals of Reasoning

4 10 8
+6 +6
Explanation : D → J and H → N
12 18
+6
Similarly L → R . So missing character is R
12.
C K V

E N Z

G Q ?
(a)B (b)I (c) D (d)C
Ans. (c)
Explanation : Moving columnwise,
+2 +2
C  → E  → G
+3 +
and K  → N → Q
+4 +4
Similarly, V  → Z  → D . So missing character is
D.
13.
AZ BY CX

DW EV FU

? HS IR

(a)GT (b)GV (c) GO (d)GP


Ans. (a)
Explanation : In each cell, pair of opposite letters is
given.
+1
∴ F  → G
and opposite letter of G is T.
∴ GT will come in place of question mark.
14. I M
R Q
27 30

32 17
X ?
H L

(a)F (b)c (c) I (d)E


Ans. (d)
Reasoning-Missing Characters 183

Explanation : The positions of letters in the English


alphabet are added in this case.
As, 18 + 9 = 27, 17 + 13 = 30
R I Q M
and 24 + 8 = 32
X H

Similarly, 12 + 5 = 17
L E

∴ Missing letter is (e)


15.
P Z T
C L Q

M N ?
(a)J (b)C (c) S (d)O
Ans. (b)
Explanation : As, 16 − 3 = 13 and 26 − 12 = 14
P C M Z L C
Similarly, 20− 17 = 3
T Q C
∴ ? = C (letter at 3rd position in the English alphabet).
PRACTICE SET
Directions (1-10) : In each of the following questions,
which character when placed at the sign of interrogation (?)
shall complete the given pattern.
1.
12 2 20

24 4 88

11 1 ?

(a)23 (b)18 (c) 12 (d)14


Ans. (c)
2. 20 7 12
16 4 18
80 56 ?
5 14 4
4 8 6

(a)72 (b)40 (c) 108 (d)48


Ans. (a)
184 Fundamentals of Reasoning

3. 289 4 9 625 49 441

34 75 ?

(a)147 (b)109 (c) 126 (d)88


Ans. (a)
4. G P 23

K H 19

U ? 27

(a)H (b)B (c) I (d)F


Ans. (d)
5. 5
?
4
64 3
125
(a)27 (b)20 (c) 45 (d)61
Ans. (a)
6. Y

P M ? G D

(a)L (b)J (c) K (d)I


Ans. (b)
7. A D H

F I M

? N R

(a)GT (b)GV (c) GO (d)GP


Ans. (c)
8. F H
? J

R L
P N
Reasoning-Missing Characters 185

(a)V (b)T (c) U (d)X


Ans. (b)
9.
DE TR
9 38
LG XZ
19 ?

(a)2 (b)42 (c) 38 (d)50


Ans. (d)
10. 12 17 6
3 81 19 9 86 2 8 ? ?

2 4 13

(a)146 (b)139 (c) 134 (d)95


Ans. (c)
qqq
25
REASONING-NUMBER SERIES

INTRODUCTION
In such types of questions, a series of numbers is given
with one number/term missing from its place. The candidate
is required to recognize the pattern involved in the formation
of series and find the missing number accordingly.
Now, let us discuss some most common patterns involved
in the formation of series.
Directions (Ex. no. 1-10) : In the following series,
replace the questions mark (?) with suitable option.
1. What comes in place of question mark (?) in the following
number series?
7, 8, 10, 13, 17, 22
(a)23 (b)28 (c) 25 (d)29
Explanation : In the given series, the difference
between two consecutive numbers is in increasing order
i.e., 1, 2, 3, 4, 5 and 6, respectively.
7 8 10 13 17 22 28

+1 +2 +3 +4 +5 +6
∴ ? = 28
2. What comes in place of question mark (?) in the following
number series?
576, 288, 144, 72, 36, ?
(a)16 (b)18 (c) 15 (d)17.5
Ans. (b)
Explanation : In the given series, previous element is
divided by 2 to get the next element.
576 288 144 72 36 18

+2 +2 +2 +2 +2
(186)
Reasoning-Number Series 187

∴ ? = 18
3. 15, 17, 21, 29, 45, ?
(a)65 (b)69 (c) 61 (d)77
Ans. (d)
Explanation :
15 17 21 29 45 77

+2 +4 +8 +16 +32
∴ ? = 69
4. 27, 32, 30, 35, 33, ?
(a)51 (b)49 (c) 50 (d)46
Ans. (a)
Explanation : The pattern is as follows
27 32 30 36 33 38

+5 –2 +5 –2 +5
∴ ? = 38
5. What comes in place of question mark (?) in the given
number series?
3, 15, 5, 35, 7, 63, ?
(a)8 (b)10 (c) 9 (d)6
Ans. (c)
Explanation : In the given series, following pattern is
used.
3×5 5×7 7×9

3 15 5 35 7 63 9

+2 +2 +2

∴ ?=9
6. Replace the question mark (?) in the series given
below with the suitable option.
5, 6, 10, 19, 35, ?
(a)60 (b)45 (c) 48 (d)55
Ans. (a)
Explanation : In the given series, following pattern is
used.
188 Fundamentals of Reasoning

5 6 10 19 35 60

+12 +22 +32 +42 +52


∴ ?=9
7. 4, 8, 24, 96, ?
(a)132 (b)480 (c) 768 (d)288
Ans.
Explanation : The pattern is as follows
4 8 24 96 480

4×2 8×3 24 × 4 96 × 5
∴ ? = 360
8. 113, 103, 88, 68, ?
(a)58 (b)42 (c) 47 (d)43
Ans. (d)
Explanation : The pattern is as follows
113 103 88 68 43

–10 –15 –20 –25


∴ ? = 37
9. Suitable option.
7, 15, 32, 67, 138, ?
(a)237 (b)190 (c) 168 (d)281
Ans. (d)
Explanation : In the given series, following pattern is
used.
7 15 32 67 138 281

×2+1 ×2+2 ×2+3 ×2+4 ×2+5


∴ ? = 281
10. 6, 7, 15, 42, 106, ?
(a)21 (b)176 (c) 231 (d)229
Ans. (c)
Reasoning-Number Series 189

Explanation : In the given series, following pattern is


used.
PRACTICE SET
Directions (Q.n. 1–10) : In the following series, replace
the question mark (?) with the suitable option.
1. 26, 27, 29, 32, 36, 41, ?
(a)47 (b)45 (c) 49 (d)46
Ans. (a)
2. 13, 22, 33, 46, 61, ?
(a)74 (b)83 (c) 78 (d)75
Ans. (c)
3. 440, 220, 110, 55, 27.5, ?
(a)9 (b)13.75 (c) 20 (d)12.5
Ans. (b)
4. 5, 16, 51, 158, ?
(a)1452 (b)483 (c) 481 (d)1454
Ans. (c)
5. 17, 36, 74, 150, ?, 606.
(a)250 (b)303 (c) 300 (d)302
Ans. (d)
6. 2, 10, 30, 68, ?
(a)125 (b)120 (c) 130 (d)135
Ans. (c)
7. 1, 4, 9, 16, 25, 36, ?
(a)49 (b)60 (c) 48 (d)51
Ans. (a)
8. 101, 89, 74, 68, 59, ?
(a)51 (b)49 (c) 50 (d)46
Ans. (a)
9. 356, 352, 343, 327, ?
(a)278 (b)291 (c) 322 (d)302
Ans. (d)
10. 17, 17, 34, 102, 408, ?
(a)74 (b)83 (c) 78 (d)75
Ans. (b)
qqq
26 REASONING-RANKING
AND ORDER

INTRODUCTION
In this type of questions, the rank or position of a person
(s) from either of the two ends of a row/queue is given and it is
asked to determine things such as the total number of persons
in the group or the number of persons to the left/right (or
above/below) of a particular person etc.
The position or rank can be calculated with the help of
following formulae :
(i) Position (or rank) from the left end (or top) = (Total
number of persons/students
– Rank from the right end (or bottom) +1
(ii) Position (or rank) from the right end (or bottom) =
(Total number of persons/students
–Rank from the left end (or top) +1
(iii) In this type of questions, it is asked to calculate the
total number of persons when the rank of a person
from both the ends is given. Following formula is
helpful in the calculation of total number of
objects/persons in a queue.
Total number of persons/in a row or queue =
(Position (or rank) of a person from the left end (or
top or front) + Position (or rank) of the person from
the right end (or bottom or last) –1.
If positions of two persons from the two opposite end and
the total number of places between these positions is given,
then
Maximum number of persons the can be present in the
row = Sum of positions of both persons + Number of places in
the middle.

(190)
Reasoning-Banking and Order 191

1. In a class of 32 students rank of Harish is 7 th from top.


what will be his rank from the bottom?
(a)26 (b)28 (c) 23 (d)25
Ans. (a)
Explanation : Harish's rank from bottom
= 32 − 7 + 1 = 26
2. In a row of 35 girls, when Ankita was shifted to her left by
5 places, her place from the left end of the row became 8.
What is the position of Shahid from the right end of the
row, if Shahid was three places to the right of Ankita's
original position?
(a)22 (b)20 (c) 24 (d)16
Explanation : One shifting 5 places to the left, Ankita is
8 th from the left end of the row. Thus, Ankita's original
position was 13 th from the left end.
3. In a class of students, Sandeep is 18th from top and 25 th
from bottom in a class, then total number of students in
the class is
(a)42 (b)40 (c) 45 (d)48
Ans. (a)
Explanation : Total students in a class = (Rank of
Sandeep from top + Rank of Sandeep from bottom)
−1 = 18 + 25 − 1 = 43 − 1 = 42
4. In a queue of fifty persons, Karthik is 10th from the left
and Ajay is 25th. from the right end. Shilpa is present in
the middle of the two. What is the position of shilpa from
the left end?
(a)17 (b)19 (c) 18 (d)20
Ans. (c)
Explanation :
Ajay's position from left = 50 − 25 + 1 = 26 th
Karthik's position from left = 10 th
Clearly, there are 15 persons in between Ajay and
Karthik.
Shilpa lies in between these 15 persons.
192 Fundamentals of Reasoning

Hence, She will be at 8 places to the right of Karthik and


18th form the left end.
5. In a class of thirty students, Jay is 14th from the left and
Vijay is 20th from the right end. How many students are
there between Jay and Vijay ?
(a)2 (b)4
(c) 3 (d)Data inadequate
Ans. (a)
Explanation :
Vijay's position from left = 30 − 20 + 1 = 10 + 1 = 11 th
Jay's position from left = 14th
Clearly, there are 2 students in between Vijay and Jay
Hence, option (a) is the correct answer.
6. In a row of girls Anuskha is fourth either side. How many
girls are there in the row?
(a)5 (b)8 (c) 6 (d)7
Ans. (d)
Explanation : Total number of girls = (Rank of Anuskha
from left + Rank of Anuskha from right) –1
∴ total number of girls are 4 + 4 − 1 = 7
7. In a row of boys Ram is 4th from left end and Shyam is 8th
from left end. How many boys are there between Ram and
Shyam?
(a)4 (b)3 (c) 5 (d)2
Ans. (b)
Explanation : Clearly, Between 4th and 8th position
there are 5 6 and 7 position so the answer is 3 boys.
8. In a row of girls, Sita is 6th from left end and Gita is 9th
from right end. How many girls are there between Sita
and Gita?
(a)4 (b)3
(c) 5 (d)Data inadequate
Ans. (d)
Explanation : Data is inadequate as total number of
students are not given.
Reasoning-Banking and Order 193

9. In a row of children facing North, Ritesh is 12th from the


left end. Virat, who is 22nd from the right end is 4th to the
right of Ritesh. Total how many children are there in the
row?
(a)37 (b)34 (c) 36 (d)35
Ans. (a)
Explanation : Virat's position from left = (12 + 4 ) = 16th
from left Clearly, there are three students between
Ritesh and Virat.
∴ Total number of children = (12 + 3 + 22) = 37
10. In a class of 42 students, a boy is ranked 20th. When two
boys joined, his rank was dropped by one. What is his new
rank from the other end?
(a)25th (b)26th (c) 24th (d)28th
Ans. (c)
Explanation : Total number of boys after 2 new boys
joined = 44
Since, the rank of the boy dropped by 1, it became 19th
∴ His new rank from the end = 42 − 19 + 1 = 24.
PRACTICE SET
1. Sagar ranked 8th from the top and 36th from the bottom
in a class. How many students are there in the class?
(a)39 (b)40 (c) 43 (d)41
Ans. (c)
2. In a row of boys, Pankaj is 7th from the start and 11th
from the end. In another row of boys, Vikas is 10th from
the start and 12th from the end. How many boys are there
in both the rows together?
(a)32 (b)40
(c) 38 (d)Cannot be determined
Ans. (b)
3. In a row of 25 children, Surbhi is 14th from the right end.
Arun is 3rd to the left of Surbhi in the row. What is Arun's
position from the left end of the row?
(a)9th (b)7th (c) 8th (d)10th
Ans. (a)
194 Fundamentals of Reasoning

4. Dharmesh ranked 17th from top and 28th from bottom.


How many students are there in the class?
(a)40 (b)45 (c) 42 (d)44
Ans. (d)
5. Nikita is 14th from the right end in a row of 40 girls. What
is her position from the left end?
(a)25th (b)29th (c) 27th (d)25th
Ans. (c)
6. Jatin boys are standing in a row facing the school
building. Jatin is 12th from the left end. If he is shifted six
places to the right, what is his position from the right
end?
(a)16th (b)15th (c) 20th (d)18th
Ans. (b)
7. In a row of boys, Rohit is seventh from either side. How
many boys are there in the row?
(a)13 (b)18 (c) 16 (d)17
Ans. (a)
8. There are 27 boys in a horizontal row. Varun was shifted
by four places towards his right side and he occupies the
middle position in the row. What was his original position
from the left end of the row?
(a)15th (b)9th (c) 12th (d)10th
Ans. (d)
9. Neha is 13th from either end of a row of girls.
(a)25 (b)28 (c) 27 (d)26
Ans. (a)
10. If in a row, Shiv is 15th from left and Saurabh is 13th
from right and there are five persons in between Shiv and
Saurabh, then find the maximum number of persons in
the row.
(a)27 (b)33 (c) 30 (d)28
Ans. (b)
qqq
27
REASONING-LETTER SERIES

INTRODUCTION
In questions on series, a specified sequence/order of
letters is given where one of the term of the series is missing
either at the end of the series or in between the series.
The candidate is required to identify the pattern involved
in the formation of series and accordingly find the missing
term to complete the series. There is no set pattern and each
question may follow a different pattern or sequential
arrangement of letters or digits, which you have to detect
using your common sense and reasoning ability.
Types of Letter Series
There are three types of letter series that we will discuss.
They are-
l Letter Series for Repeating Letters
l Letter Series for Non-Repeating Letters
l Letter Series Having Group of Letters
Letter Series for Repeating Letters
This series generally consists of small letters of English
alphabet where the letters are arranged following a certain
pattern. However, some letters are missing from the series.
The candidate is required to identify the pattern and
accordingly find the proper sequence of these missing letters
from the given alternatives.
Following example will give you a better idea about the
type of questions-
Directions (Ex. no. 1–5) : Find the missing term in the
following series.

(195)
196 Fundamentals of Reasoning

1. _bba_b_b_ab
(a)aabba (b)abaab (c) abbab (d)ababb
Ans. (d)
Explanation : The series is as follows
abb / abb / abb / abb
∴ Required answer = ababb
2. _b_bca_a_ab_
(a)abbca (b)babac (c) accbc (d)bbabb
Ans. (c)
Explanation : The series is as follows
abc / bca / cab / abc
∴ Required answer = accbc
3. _sr_tr_srs_r_srst_
(a)tstttr (b)ttssrr (c) strtrs (d)ttssr
Ans. (a)
Explanation : Series pattern : tsr / str / tsr / str / tsr / str
∴ Required answer = tstttr
4. ab_c_c_a_ab_a_cc
(a)bbcca (b)aabcba (c) cbacbb (d)baccbb
Ans. (c)
Explanation : Series is as follows
abcc / bcaa / cabb / abcc
∴ Required answer = cbacbb
5. pqr_ _rs_rs_ _s_ q_
(a)sqppqpr (b)sqprrqr (c) spqppr (d)pqrrppq
Ans. (b)
Explanation :
Directions (Ex. no. 6–10) : What comes in place of
question mark (?) in the following letter series?
6. A, C, F, J, O, ?
(a)T (b)V (c) R (d)U
Ans. (d)
Explanation : Every next term is obtained by adding 2,
3, 4, 5, 6 respectively from its corresponding position.
Reasoning-Letter Series 197

1 3 6 10 15 21
A C F J O U

+2 +3 +4 +5 +6
7. Z, X, ?, N, F
(a)Q (b)R (c) O (d)T
Ans. (d)
Explanation :
25 23 21 18 15 11 7
Y W U R O K G

–2 –2 –3 –3 –4 –4
8. B, B, A, D, Z, F, Y, H, X, ?
(a)J (b)G (c) M (d)K
Ans. (a)
Explanation : This series has two parts. In first part,
every next term is obtained by subtracting 1 from the
corresponding position of its previous letter. While in
second part, every next term is obtained by adding 2 to
the corresponding position of its previous term (letter).
2 2 1 4 26 6 25 8 24 10
B B A D Z F Y H X J

–1 –1 –1 –1
+2 +2 +2 +2

9. R, A, M, A, N, N, A, M, A, ?
(a)Y (b)C (c) R (d)E
Ans. (c)
Explanation : The first part of the series (R, A, M, A, N)
is written in reverse order (N, A, M, A, R).
10. Y, W, U, R, O, K, ?
(a)F (b)H (c) E (d)G
Ans. (d)
Explanation : The pattern is as follows
26 24 20 14 6
Z X T N F

–2 –3 –6 –8
198 Fundamentals of Reasoning

Series Having Group of Letters


In such series, each element consists of group of letters
instead of a single letter.
11. Analyze the series and choose the answer that will come
in the place of question mark.
AK, FP, ?, PZ, UE,
(a)KU (b)JT (c) JU (d)KV
Ans. (a)
Explanation : The pattern is as follows :
+5 +5 +5 +5

A K F P K U P Z U E

+5 +5 +5 +5

12. What comes in place of question mark in the following


letter series?
ZXV, TRP, NLJ, ?
(a)HFD (b)HGE (c) HDF (d)IGF
Ans. (c)
Explanation : The pattern is as follows
26 20 14 8
–6 –6 –6
1st Letter : Z T N H
24 18 12 6
–6 –6 –6
2nd Letter : X R L F

22 16 10 4
–6 –6 –6
3nd Letter :V P J D

13. What comes in place of question mark in the following


letter series?
AC, FH, KM, PR, ?
(a)VW (b)UW (c) UX (d)TV
Ans. (b)
Explanation : The pattern is as follow
1 3 6 8 11 13 16 18 21 23
A C F H K M P R U W

+2 +3 +2 +3 +2 +3 +2 +3 +2
Reasoning-Letter Series 199

14. Choose the term which will continue the following series-
E 3C , G 5 F, I8I, K 12 L,?
(a)L17O (b)M16 M (c) N 18O (d)M17O
Ans. (d)
Explanation : Pattern of give letter-number series is as
follows:
Letter : E +2 +2 +2 +2
G I K M

Number : 3 2+ +3 +4 +5
5 12 12 17
Letter : C +3 +3 +3 +3
F I L O

15. ICV, 7FU, 12IT, ? 220R


(a)19LS (b)17LS (c) 15LS (d)13LS
Ans. (b)
Explanation : Pattern of given alpha-numeric series is
as follows
+5 +5 +5 +5
2 7 12 17 22
3 6 9 12 15
+3 +3 +3 +3
C F I L O

22 21 20 19 18
–1 –1 –1 –1
V U T S R

Directions (Q.n. 1–10): What comes in place of question


mark (?) in the series given below?
1. BDF, HJL, NPR, ?
(a)UVW (b)TVX (c) TUV (d)OQS
Ans. (b)
2. BA 100 ZY, DC 400 XW, FE 900 VU, ? JI 2500 RQ
(a)GH 1000 ST (b)HG 1300 TS 2500
(c) HG 1400 TS (d)HG 1600 TS
Ans. (d)
3. Q, N, K, H, E, ?, Y
(a)A (b)Z (c) C (d)B
Ans. (a)
4. aa_aabb_b_aa_aabb_bb
(a)bbbaa (b)bbbba (c) aabbb (d)babban
Ans. (d)
200 Fundamentals of Reasoning

5. A, R, C, S, E, T, G, ?, ?
(a)V,B (b)X,Z (c) U,I (d)X,Z
Ans. (c)
6. Z9A, X7D, ?, T3J, R1M
(a)G9V (b)S3H (c) VSG (d)W6F
Ans. (c)
7. pqr_ _ rs _ rs_ _ s_q_
(a)sqppqp (b)sqprrqr (c) spqpprr (d)pqrrpq
Ans. (a)
8. L, N,R,Z, ?
(a)E (b)P (c) J (d)O
Ans. (b)
9. NZ, OY, PX, ?, SU
(a)UR (b)RD (c) RU (d)RV
Ans. (d)
10. X24C3, V22E5, T20G7, ?
(a)R17l9 (b)R18l9 (c) R19l10 (d)R161l7
Ans. (b)
qqq
28 REASONING-SITUATION
AND REACTION

INTRODUCTION
In these questions, a situation related to the day-to-day
life is given along with four alternatives which gives various
probable reactions to these situations. The candidate is
required to find out the most appropriate reaction to these
situations. For solving these questions, first of all try to
analyse the demand of the situation. Depending upon the
need, try to analyse the pros and cons of each alternative and
then choose that alternative which solves the problem and
purpose without any negative effect.
You will get a better idea about the type of questions
asked, from the examples given below.
1. You find that one of your friends has stolen a book fro
your college library. you would
(a)Immediately go to principal sir and inform him about
it.
(b)Immediately go to library and inform sir about the
theft.
(c) Politely ask your friend to deposit the book in library
as this is college's property.
(d)Write his name on the notice board that he has stolen
the book from library.
Ans. (c)
Explanation : Now let's analyse the situation carefully.
The person is your friend. If you found that he has stolen
the book then talking directly to the principal may
hamper his image. Any such situation like talking to the
library sir would result in same. You must look for those
solutions in which the loss of image will be less but the
situation will be handled well.

(201)
202 Fundamentals of Reasoning

For this reason, talking to your friend would be the first


thing that you should do. If he will not listen to your
advice then you can take any extreme major step like
talking to the library sir, or principal etc.
2. While travelling in a train, you notice that a man fall off
the train from the coach behind yours coach. You would
(a)Jump off the train to assist the falling man
(b)Shout at the falling man asking him to get up quickly
and entrain
(c) Wait till the train stops at the next station and inform
the railway authorities there
(d)Pull the alarm chain, so that the train may stop and
the man may be helped.
Ans. (d)
Explanation : Clearly, the situation demands taking
quick action to provide help to the victim which in turn
which in turn requires that the train be stopped
immediately.
3. Your team lead in the office is absent today and there is
an urgent assignment that needs to be submitted by
today evening. You are more experienced than all other
team mates. You will:
(a)Wait for the team lead to arrive.
(b)Correspond with the client regarding the problem.
(c) Volunteer yourself and lead the team.
(d)Correspond with the boss regarding the problem.
Ans. (c)
Explanation : As you are enough experienced in your
team next to your team lead, leading your team in his
absence should be your reaction. You should not follow
the reactions given in option (b) or (d) as they can create
negative impression from both client side and boss.
4. Your are spending your holidays in a picnic spot and
suddenly you see a purse which have his owner's photo
and contain around 1000 rupees. You would
(a)Take 1000 rupees and put the purse where it is
Reasoning-Situation and Reaction 203

(b)Contact your nearest police station and deposit it


there.
(c) Take the purse and get away from there
(d)Make a research by uploading the photo of the owner
on the internet on different social media sites and try
to find that person.
Ans. (b)
Explanation : You are not a thief so you will not steal
money and there is no time in this world for us to make a
research about the person who has lost his purse. So the
best possible way is to deposit the purse with all items in
it at the nearest police station in that person.
5. While playing cover drive shot, one of your friends has
pointed out your mistakes and shared it with you. You
would
(a)Feel thankful. (b)Beat him with your bat
(c) You will cry out (d)You will be ashamed of it
Ans. (a)
Explanation : If anyone is pointing out our mistakes
and discussing it with us then we should be thankful
towards him.
6. While walking on a street, you see that two men are
bothering a girl. You would
(a)Stand in a corner and enjoy the proceedings
(b)Get out of there as it is none of your business.
(c) Inform the police to take necessary action
(d)Immediately reach to them and help the girl from
strangers.
Ans. (d)
Explanation : In this situation you must have to show
humanity. So the best option is d.
7. You are moving on your motorcycle with your friend and
have seen that two person on bike came and snatched
necklace from a lady. You would
(a)Stand and see what happens
(b)Talk with the lady and try to calm her down.
204 Fundamentals of Reasoning

(c) Ask other people to chase those thieves.


(d)Chase the snatchers to catch hold of them
Ans. (d)
Explanation : In this situation, you should chase the
boys to catch hold of them so, the best answer is (d)
8. When you reached the railway platform to catch the train
to Delhi, you find that the train is about to start and you
have not bought the ticket yet. The best thing for you is
to?
(a)Catch the train and inform the T.T.I at the next
stoppage about your inability to purchase the ticket.
(b)Rush to the train and perform your journey quietly.
(c) Don't enter the train without any ticket.
(d)Rush to he train and when the TT comes hide yourself
in the toilet.
Ans. (a)
Explanation : I know many of colleges students may
prefer option D but it is not correct if you will get caught
then fine will be imposed so the best option is enter the
train and inform the TT about your problem.
9. The thieves and burglars in your city are increasing in
great numbers and one night you are alone in your house.
Suddenly you hear a sound from the kitchen.
(a)You will start crying loudly.
(b)You will call either your neighbour who lives beside
your house or a good friend of yours.
(c) You will pray to god to get you out of this situation.
(d)You will run out of your home and reach to the nearby
police station
Ans. (b)
Explanation : The best way to react in this situation is
to call a friend in neighbourhood because if you will run to
nearby police station then there are chances that the
thief will come into your living room and steal valuables.
Reasoning-Situation and Reaction 205

10. You are unemployed and you have seen an advertisement


in a local newspaper who is offering jobs in Dubai by
taking charges of 100000 rupees.
(a)You will rush to that office and immediately deposit
100000 rupees.
(b)Call your unemployed friends and ask them to deposit
money there.
(c) Request the officials to give you a concession in the
amount of fees.
(d)First make a research about that company and take
appropriate steps.
Ans. (d)
Explanation : The correct option (d) because first we
have to be confirmed about the genuineness of the
company. We cannot give money to someone by just
looking at the advertisement.
PRACTICE SET
1. After a purchase, the shopkeeper returns ` 100 extra to
you. You will
(a)Offer ` 20 at a temple and pray for more such instance
(b)Hope he will give another ` 100 extra
(c) Return the extra money to the shopkeeper
(d)Treat your friends to a lunch
Ans. (c)
2. When you see a blind man trying to cross the road, you
(a)Go and help him
(b)Wait till he crosses the road
(c) Ask someone to help him
(d)Ignore and move on
Ans. (a)
3. Your bathroom tap is leaking and is a constant source of
irritating noise. You will
(a)Sleep with pillows upon your ears
(b)Try to put up a cork upon the mouth of the tap
(c) Put a bucket underneath
206 Fundamentals of Reasoning

(d)Call a plumber to repair the tap


Ans. (b)
4. Your friends like smoking and influence you to do the
same. You will
(a)Refuse to smoke
(b)Smoke but only in their presence
(c) Smoke only because your friends are smoking
(d)Refuse and lie to them that you have asthma
Ans. (a)
5. You are passing by a river and you know swimming.
Suddenly you hear the cry of drowning child. You would
(a)Wait to see, if some other person is there to help
(b)Dive into the river to save him
(c) Look for professional divers
(d)Console the childs parents
Ans. (b)
6. While travelling in your car, certain persons stop you on
the way asking you to take an injured child to the
hospital. You would
(a)Ask them to first call the police
(b)Get out of the car and ask some other person to help
them
(c) Ask them to leave your way and then drive away
(d)Immediately take the child to hospital
Ans. (d)
7. Your friend has lost his/her purse with your important
document in it. You would
(a)Feed angry but do not react as anyone can make
mistakes
(b)Blame him/her for being careless and stop talking to
him/her
(c) Feel angry and ask him/her to forget everything.
(d)Understand the situation and tell him/her that It's ok
and not to worry about it.
Ans. (a)
Reasoning-Situation and Reaction 207

8. Your classmate who is usually very energetic and happy


all the time looks very down and upset. You
(a)Carry on with your work
(b)Tell one of your friends to go and talk to her
(c) Go up to her and ask the reason
(d)Wait for her to come up and tell you the reason
Ans. (d)
9. Your classmate, who got you in a fix recently with a
teacher, has met with an ancient. You
(a)Feel that God taught him/her a lesson
(b)Carry on with life unaffected
(c) Tell others that this is the way one suffers
(d)Decide to visit him/her in the hospital
Ans. (d)
10. You are playing football in a park. When you kick the
ball, it strikes and breaks the window pane of a nearby
house. You would
(a)Stealthily get your ball back
(b)Apologize to the house owner and contribute to
replace the glass
(c) Demand your ball back from the house owner
(d)Say that it was no fault of yours
Ans. (b)
qqq
29 REASONING-STATEMENT
AND ARGUMENTS

INTRODUCTION
Statement and arguments are those types of problems
where your decision making power and analytical power are
being exercised. A given statement will have two possible
arguments. The candidate is required to decide which
argument is much stronger enough to support the main
statement. There may be a case where neither or both the
statements may be strong. Sometimes even any one of the
argument can be strong.
Question asked from this chapter are basically based on
following format
Statement……………
Arguments I. Yes, …………………
II. No. ………………….
Types of Arguments
There are generally two types of arguments.
l Strong argument
l Weak argument
Strong Argument : These are the types of arguments
that strongly support the given statement and give some
logical base.
For example
Statement : Should only articles of well-established
authors be published?
Arguments :
I. Yes, paper will be saved through this
II. No, new authors will not get any chance.

(208)
Reasoning-Statement and Arguments 209

Explanation : Here on comparing the two arguments,


we can see that argument II is much stronger because it
states the need of chance that new authors need to prove
themselves.
Weak Arguments : These arguments are either illogical
or do not have any strong point to support the given
statement. Let's take an example to understand it in a much
better way.
Statement : Should prestigious people who commit a
crime by mistake should be treated differently?
Arguments : I. Yes, because they did the crime
unintentionally.
II. No, everyone is equal before the law.
Explanation : Clearly if we will try to find the weak
statement among these two we will choose the 1st argument.
Because we know that crime is crime and everybody should be
treated according to their deeds, irrespective of their attire.
Example given below will give you a better idea about the
type of questions asked is exam.
Directions (Ex. 1–10) : Study the following instructions
carefully and then answer the questions that follow. In
making decisions about important questions, it is desirable to
be able to distinguish between 'strong' and 'weak' arguments
so far as they relate to the questions. 'Weak' arguments may
not be directly related to the question and may be of minor
importance or may be related to the trivial aspect of the
question. Each question below is followed by two arguments
numbered I and II. You have to decide which of the arguments
is a 'strong' argument and which is a 'weak' argument?
Give answer :
(a) If only Argument I is strong
(b) If only Argument II is strong
(c) If either I or II is strong
(d) If neither I nor II is strong
210 Fundamentals of Reasoning

(e) If both I and II are strong


1. Statement : Should young entrepreneurs be
encouraged?
Arguments : I. Yes, they will help in the industrial
development of the country.
II. Yes, they will increase the burden on the employment
market.
Ans. (e)
Explanation : Both the arguments refer to the practical
consequences of the action mentioned in the statement
and hence, are strong.
2. Statement : Should encouragement be given to the
horticulture industries in rural areas?
Arguments : I. Yes, creativity is there among rural
people.
II. Yes, this will help in eradicating problem of
unemployment to much extent in rural area.
Ans. (e)
Explanation : Both the arguments strong enough to
support the given statement logically.
3. Statement : Should rapists be hanged?
Arguments : I. Yes, they are sinners and must be
hanged.
II. No, they are human beings give them one more
chance.
Ans. (c)
Explanation : Both are true but both cannot be true at
the same time. Hence, either is strong.
4. Statement : Should public holidays be declared on
demise of important national leaders?
Arguments : I. No, such unscheduled holidays hamper
national progress.
II. Yes, people would like to pay their homage to the
departed soul.
Reasoning-Statement and Arguments 211

Ans. (a)
Explanation : Considering India's economy,
unscheduled holidays will hamper national progress.
Hence, Argument I is strong, Argument II is ambiguous
and hence, it is weak.
5. Statement : Should so much of money be spent on games
and sports?
Augments I. Yes, we are a rich nation and can easily
spare any amount of money for games and sports.
II. No, our teams are unable to put up a good show in
international competitions.
Ans. (d)
Explanation : None of the arguments is strong.
Spending money just because of being rich is not a valid
reason. Argument 11 is weak because it is absolutely
illogical that just because of poor performance in
international competitions, we should not finance
games? sports.
6. Statement : Will the newly elected CMC mayor fulfil her
promises?
Augments : I. Yes, she has to otherwise her existence as
mayor will be in danger.
II. No, candidates after getting power and positions
forget their promises.
Ans. (e)
Explanation : Both the arguments are true as
politicians have immense pressure to uphold their
position and after getting into it, they hardly fulfil their
promises.
7. Statement : Should the age for marriage of boys be
increased to more than 30 years?
Augments : I. No, it depends upon the person and his
family members.
II. Yes, it may check the population increase to a certain
level.
Ans. (d)
212 Fundamentals of Reasoning

Explanation : None of the arguments are strong


regarding the statement.
8. Statement : Should our government bring down the age
of retirement in public services?
Arguments : I. Yes, as this may create new vacancies for
unemployed and talented ones to serve their country.
II. No, government officers may have their own family
problems which may not be solved if retirement is taken
early.
Ans. (e)
Explanation : Here both the arguments are strong and
logical.
9. Statement : Should school education in poor states like
Bihar, Assam, Odisha be made free?
Arguments : I. Yes, as these are poor states, this is the
only way to improve literacy.
II. No, this might have adverse impact on the economy.
Ans. (b)
Explanation : Argument I is weak because making
education free may not lead to 100% literacy but II is
strong because it can put heavy financial burden.
10. Statement : Should all the factories in the cities be
shifted to the outskirts, far away from the main city?
Arguments : I. Yes, this is an essential Step for
controlling pollution in the city.
II. No, such a Step will lead to lot of inconvenience to the
employees of the factories and their families as well.
Ans. (a)
Explanation : Argument I is strong because pollution
control is highly desirable. Argument II is weak.
PRACTICE SET
1. Statement : Should there be complete ban on
manufacture of fire crackers in India?
Arguments : I. No, this will render thousands of workers
jobless.
Reasoning-Statement and Arguments 213

II. Yes, the fire cracker manufacturers use child labour to


a large extent.
Ans. (b)
2. Statement : Should strikes and dharnas in the field of
hospital be banned?
Arguments : I. Yes, it is hampering the hospital and
patients are suffering a lot.
II. No, the employees must get their dues in proper
manner.
Ans. (e)
3. Statement : Should the sale of all the toys made in China
be banned in India?
Arguments : I. No, Indian toys are of much better
quality and their sale will not be affected.
II. Yes, these are very cheap and hence, will put the local
toy manufactures out of business.
Ans. (c)
4. Statement : Should young entrepreneurs be
encouraged?
Arguments : I. No, this will render thousands of workers
jobless.
II. Yes, the fire cracker manufacturers use child labour to
a large extent.
Ans. (a)
5. Statement : Should the organisations like the UNO be
abolished?
Arguments : I. No, in the absence of these organisations,
there will b e a World War.
II. The cold war is going to an end, so there is no role of
such organisations.
Ans. (e)
6. Statement : Should the women be advised not to travel
alone at night in view of the increasing incidences of rape
and sexual abuse?
214 Fundamentals of Reasoning

Arguments : I. No, instead the government should take


measures to control such incidences.
II. Yes, it is difficult even for the police department to
control such cases.
Ans. (e)
7. Statement : Should smoking cigarettes and drinking
alcohol by the actors be completely banned in the movies
in India?
Arguments : I. Yes, this will significantly reduce the
trend of smoking cigarettes and drinking alcohol among
the youth in India.
II. No, the films portray the society and hence such scenes
should be an integral part of the movie, if the storyline
demands so.
Ans. (e)
8. Statement : Should India make efforts to harness solar
energy to fulfill its energy requirement?
Arguments : I. No, harnessing solar energy requires a
lot of capital, which India lacks in.
II. Yes, most of the energy sources used at present are
exhaustible.
Ans. (b)
9. Statement : Should sale of vital human organs be made
legal in India?
Arguments : I. Yes, this will bring an end to the illegal
trading of human organs.
II. No, this will lead to unhealthy practices.
Ans. (d)
10. Statement : Should road repair work in big cities be
carried out only late at night?
Arguments : I. Yes, the commuters will face a lot of
problems due to repair work during the day.
II. No, this way the work will never get completed.
Ans. (a)
qqq
30 REASONING-STATEMENT
AND ASSUMPTIONS

INTRODUCTION
Statement is an information or a fact related to any
general subject and Assumption is the hidden or unsaid part
of the statement. An assumption is said to be hidden because
it is generally assumed, supposed and taken for granted.
There are some special rules related to assumptions
which are as follows :
(i) Assumption should be a general statement.
(ii) The meaning of assumption should be embedded in
the statement.
(iii) There should be no repetition of statement in
assumption.
(iv) Statement and assumption should be logical to each
other.
(v) Assumption should not be a conclusion on the basis
of statement.
Directions (Ex. no. 1–10) : In each of the questions
below, is given a statement followed by two assumptions
numbered I and II. An assumption is something supposed or
taken for granted. You have to consider the statement and the
assumptions and decide which of the assumption (s) is/are
implicit in the statement.
Give answer :
(a) If only Assumption I is implicit
(b) If only Assumption II is implicit
(c) If either I or II is implicit
(d) If neither I or II is implicit
(e) if both I and II are implicit

(215)
216 Fundamentals of Reasoning

1. Statement : After doing this operation, condition of the


patient will improve.
Assumptions : I. Under the current condition, operation
of the patient can be done.
II. Operation cannot be done under the current of the
patient.
Ans. (a)
Explanation : By reading the statement it can be
understood that improvement in the health condition
will be there after the operation, which means that there
is a possibility of doing operation on patient's body under
the current situation.
2. Statement : Mobile subscriber can activate internet
services by calling to toll free number 4024.
Assumptions : I. The toll free number can b e misused.
II. Subscribers will be very happy with such services.
Ans. (b)
Explanation : Assumption I is not implicit as it does not
give any information about its mis use. Further, this step
will benefit many subscribers.
Hence II is implicit.
3. Statement : ICICI bank has recently highest increase in
gross non-performing assets.
Assumptions : I. Business of ICICI bank will be greatly
affected.
II. Borrowing from the banks will not be easy by its
customers.
Ans. (e)
Explanation : According to the given statement, as the
NPA has been increased, so it is obvious that the business
will be affected. As a result of which, it will not be easy on
the part of the bank to provide loans and advances to its
customers.
4. Statement : HDFC launched its co-branded credit card
in association with Andhra bank card.
Reasoning-Statement and Assumptions 217

Assumptions : I. Association with Andhra bank will


increase its market penetration.
II. There are no other trusted partners in banking.
Ans. (a)
Explanation : Assumption I is implicit, as we can
understand that being a big PSU bank, market
penetration will be easy but nothing can be said about the
trusted partner. Hence, only I is implicit.
5. Statement : TRAI made mandatory for telecom
operators to compensate for call from 1st Jan 2016.
Assumptions : I. There will not be any scam in future.
II. Telecomm subscriber's confidence and trust will
increase for TRAI.
Ans. (b)
Explanation : No guarantee can be given regarding not
happening of any scam in future. So, assumption II is not
practical.
6. Statement : Most people who stop smoking gain weight.
Assumptions : I. If one stops smoking, one will gain
weight.
II. If one does not stop smoking, one will not gain weight.
Ans. (d)
Explanation : Most of the people who stop smoking
having gained weight, that does not mean that if one
quits smoking, he/she will surely gain weight or
non-smokers cannot gain weight. So, neither Assumption
I nor II is implicit.
7. Statement : Longman dictionary is famous among all.
Assumptions : I. Apart from being compact and precise,
it covers all or maximum words.
II. College students are introvert.
Ans. (a)
Explanation : As Longman dictionary among all, so it is
quite natural that it will cover all words and could also be
218 Fundamentals of Reasoning

compact and precise but nothing can be judged about the


college students from the statement.
8. Statement : FIFA banned Nepal's Football Chief for 10
years.
Assumptions : I. After Nepal, India is FIFA's next
target.
II. India will cut its relation with Nepal in football.
Ans. (d)
Explanation : Here none of the assumptions follows
because both are not logical at all.
9. Statement : Detergent should be used to clean clothes.
Assumptions : I. Detergents form less lather.
II. Detergents help to remove grease and dirt.
Ans. (b)
Explanation : Assumption I is not implicit because
nothing is mentioned about the lather formation by the
detergent. Further, detergents should be used as they
clean clothes better and more easily. Hence Assumption
II is implicit.
10. Statement : To excel in IIT exam, one has to score more
in Mathematics.
Ans. (e)
Explanation : Definitely Mathematics would be mark
fetching. Otherwise, it would not have been
recommended and secondly, it also signifies that any
other subject apart from mathematics is difficult.
PRACTICE SET
Directions (Q.no. 1–10) : Each of these questions has a
statement followed by two assumptions numbered I and II.
An assumption is something supposed or taken for granted.
Consider the statement and the following assumptions.
Give answer :
(a) If Assumption I is implicit
(b) If Assumption II is implicit
Reasoning-Statement and Assumptions 219

(c) If either Assumption I or II is implicit.


(d) If neither Assumption I nor II is implicit
(e) if both I or II one implicit
1. Statement : Blue chip company X now enters to spices
product.
Assumptions : I. X's business policy is to enter to spice
market and earn money.
II. Other blue chip companies will also follow the
footsteps of X.
Ans. (a)
2. Statement : Everybody loves reading adventure stories.
Assumptions : I. The adventure stories are the only
reading materials.
II. Nobody loves reading any other material.
Ans. (d)
3. Statement : Imprisonment for 27 yr made Nelson
Mandela the President.
Assumptions : I. Travelling by air has become a status
symbol now.
II. Large number of people are able to afford air travel
now.
Ans. (d)
4. Statement : Odisha Railway station became the first
visually challenged friendly railway station.
Assumptions : I. Blind people can travel comfortably in
Odisha railway station.
II. Those who have eyes may face difficulty in Odisha.
Ans. (a)
5. Statement : A major retail store announced thirty
percent reduction on all food items during the weekend.
Assumptions : I. People may still prefer buying food
items from other stores.
II. A large number of customers may visit the retail store
and buy food items.
220 Fundamentals of Reasoning

Ans. (b)
6. Statement : Be humble even after gaining victory.
Assumptions : I. Many people are humble after gaining
victory.
II. Generally people are not humble.
Ans. (d)
7. Statement : It is faster to travel by air to Lucknow from
Pune.
Assumptions : I. Pune and Lucknow are connected by
air.
II. There are no other means of transport available to
Lucknow from Pune.
Ans. (a)
8. Statement : If it does not rain throughout this month
most farmers would be in trouble this year.
Assumptions : I. Timely rain is essential for farming.
II. Most of the farmers are generally dependent on rains.
Ans. (e)
9. Statement : It is surprising that India beat Australia
even after follow on. It happens 2nd time in the history of
test cricket.
Assumptions : I. Winning after follow on is possible.
II. India cannot repeat such victory again.
Ans. (a)
10. Statement : There has been a remarkable increase in
the air traffic in India during the past few years.
Assumptions : I. Travelling by air has become a status
symbol now.
II. Large number of people are able to afford air travel
now.
Ans. (b)
qqq
31 REASONING-STATEMENT
AND CONCLUSIONS

INTRODUCTION
Statement and conclusion is basically a logical reasoning
section. In this section, a statement will be given followed by a
set of conclusions. You need to choose the conclusion that
logically follows the statement the most. Sometimes the
conclusions can be directly understood by reading the
statement and sometimes the reader needs to analyse it to get
the indirect conclusion. Sometimes any one of the conclusion
may follow and sometimes all may follow. It may also happen
that either one of them follow or none of them follow. It
depends upon the reader's perception power to analyse the
statements to arrive at the right conclusion.
Following examples will give you a better understanding
about this type of questions.
Directions (Ex. 1–10) : In each of the following
questions, a statement is followed by two Conclusions I and
II.
Give answer :
(a) If only Conclusion I follows
(b) If only Conclusion II follows
(c) If either I or II follows
(d) If neither I nor II follows
(e) If both I and II follow
1. Statement : An apple a day, keeps the doctor away.
Conclusions : I. Apple is good for health.
II. Orange is better than apple.
Ans. (a)

(221)
222 Fundamentals of Reasoning

Explanation : Here the statement is saying about the


positive effect of eating an apple on our health. SO
definitely eating apple is good for our health. But nothing
in the statement has been told about the benefits of the
orange. Also we do not know about the comparison of
benefits between orange and apple. Hence here only first
conclusion is valid one. So our anser is option A.
Note :
l While solving this type of problems, we have to
assume whatever has been told in the statement is
true. For instance, in the above example if it would
have been told that "A cup of tea keeps the doctor
away", for us this would be the true statement.
l Do not assume anything outside the given
statement. That means while solving the above
example, we encountered a statement "Orange is
better than apple". May be it is true but for us it is
not. This is because nothing has been told about
orange in the given statement. So we cannot assume
it as true sentence.
2. Statement : Panel has decided to shortlist those
candidates having minimum 65% throughout their
career. Madhuri has secured 72% in 10th and 63% in
12th. She graduated with 75%.
Conclusions : I. Madhuri will be shortlisted.
II. Madhuri will not be shortlisted.
Explanation : Here we have to carefully arrive at the
conclusion. Madhuri has secured more than 65% in both
graduation and 10th. However, her 12th percentage is
not above 65%. So, she will not be shortlisted.
3. Statement : You work hard and consistently, you will
pass the exam.
Conclusions : I. Ravi is working hard and he is
consistent too. He will pass the exam.
Ans. (c)
Reasoning-Statement and Conclusions 223

Explanation : Here with a careful reading one can


understand that either of the two conclusions can happen
but not both. Either Ravi will work hard and will be
consistent, or else he will be not. So clearly our answer
will be either conclusion I or II follows.
4. Statement : Economy of India primarily depends upon
forest.
Conclusions : I. Saving of the tree should be done to
grow economy of India.
II only India needs to focus on saving the tree.
Ans. (a)
Explanation : Here the 1st conclusion is the indirect
explanation of the given statement. Hence conclusion I is
valid. Conclusion II has said nothing about the other
nation's condition. So; II conclusion is not valid.
Therefore; in this case, only conclusion I is valid. Or
answer is option A.
5. Statement : Only smart and intelligent people are
invited into the conference. Sahil has been invited to
attain the conference.
Conclusions : I. Sahil is smart.
II. Sahil is intelligent.
Ans. (e)
Explanation : Sahil has been called for the conference
means that he is smart and intelligent. Hence both the
conclusions are valid.
6. Statement : Pakistan is planning to test five nuclear
bombs in Baghdad.
Conclusions : I. People of Pakistan are supporting the
decision of the nation.
II. Other nations will oppose the decision made by
Pakistan.
Ans. (d)
Explanation : From the given statement, it cannot be
conferred clearly whether people of Pakistan and other
224 Fundamentals of Reasoning

nations will support the decision or not. Hence neither of


the conclusions is valid.
7. Statement : Quality comes with a price. India is
spending a lot for education.
Conclusions : I. Soon improvement in the quality of
education in India will happen.
II. Only funding can improve the quality of education.
Ans. (a)
Explanation : According to the given statement, to get
quality education, one needs to spend much and India is
spending much for quality education. Therefore, it is for
sure that the quality of education is going to improve in
near future. Hence conclusion I is valid. Conclusion II
however; is irrelevant related to statement.
8. Statement : Vehicles destroy the natural resources.
Conclusions : I. All natural resources are destroyed by
vehicles.
II. No vehicles, no environmental pollution.
Ans. (d)
Explanation : None of the conclusions follows the
statement. It is erroneous to assume that all natural
resources are destroyed by vehicles. Similarly, there are
other factors which pollute environment.
9. Statement : Fashion is a form of ugliness and becomes
necessary to change it every six months.
Conclusions : I. Fashion designers are incapable of
understanding the mind of the general public very well.
II. Public go for a change every time they see new
arrivals.
Ans. (b)
Explanation : First conclusion is not true because it is
the work of the fashion designers to judge the fashion
trends and needs of the people before designing.
However, people generally get attracted by new arrivals
every time. So only conclusions II is valid.
Reasoning-Statement and Conclusions 225

10. Statement : Morning walks are good for health.


Conclusions : I. All healthy people go for morning
walks.
II. Evening walks are harmful.
Ans. (d)
Explanation : In the statement it is said that morning
walks are good for health. But it does not mean that all
the healthy persons go for morning walks. Hence,
Conclusion I does not follow. Also, nothing is said about
the evening walks in statement.
Thus, Conclusion II also does not follow.
PRACTICE SET
1. Statement : It is not good to interfere in anyone's private
life.
Conclusions : I. Encroachment in anyone's personal
matter is bad.
II. Professional detail does not come under private life.
Ans. (a)
2. Statement : Maoists called a two day protest opposing
operation Green hunt.
Conclusions : I. Maoists are against this operation.
II. Maoists support this operation.
Ans. (a)
3. Statement : You work hard and consistently, you will
pass the exam.
Conclusions : I. Concentration powers is required to
pass the exam.
II. Minimum 10 minutes yoga is necessary to increase the
concentration power.
4. Statement : The panel asked 5 managers to resign from
job or else they will be facing rejection. Four of them have
resigned till this evening.
Conclusions : I. One manager will resign tomorrow.
II. The panel will terminate him tomorrow.
Ans. (c)
226 Fundamentals of Reasoning

5. Statements : Vasu is not the father of Girish. Girish is


the son of Mahesh. Mahesh has three sons.
Conclusions : I. Girish is the brother of Vasu.
II. Mahesh is the father of Girish.
Ans. (b)
6. Statement : Sun is the source of light.
Conclusions : I. Moon is not the source of light.
II. Light has only one source.
Ans. (d)
7. Statement : From the next academic year, students will
have the option of dropping Mathematics and Science for
their school leaving certificate examination.
Conclusions : I. Students, who are weak in Science and
Mathematics, will be benefitted.
II. Earlier students did not have the choice of continuing
their education without taking these subjects.
Ans. (e)
8. Statement : Unemployment is one of the main reason for
the poverty of the country.
Conclusions : I. To end poverty, it is required to create
employment opportunities.
II. All the people in the country are unemployed.
Ans. (a)
9. Statement : Fortune favours the brave.
Conclusions : I. Cowards die many times before their
death.
II. Risks are necessary for success.
Ans. (b)
10. Statement : Power consumption in every family has
been doubled during the last ten years.
Conclusions : I. Power rates have become cheaper.
II. There is a lot of development in the society.
Ans. (d)
qqq
32
REASONING-WORD FORMATION

INTRODUCTION
Word formation is a process of utilizing your intellectual
knowledge to form a meaningful word from a given series of
jumbled letters. You should have strong vocabulary
knowledge to solve these types of questions. Unless you are
not good at judging the write spelling of word, situation may
become difficult for you with a rise in complexity of the
questions. Examples given below will give you a better idea
about the type of questions asked.
1. In the following question, a group of letters is given which
are numbered 1, 2, 3, 4, 5 and 6. Below are given four
alternatives containing a combination of these numbers.
Select that combination of numbers so that letters
arranged accordingly, form a meaningful word.
EMLEPT
1 2 3 4 5 6

(a)612534 (b)623154 (c) 234561 (d)123456


Ans. (a)
Explanation : The word is ' TEMPLE'.
2. In the following question, a group of letters is given which
are numbered 1, 2, 3, 4, 5 and 6. Below are given four
alternatives containing a combination of these numbers.
Select that combination of numbers so that letters
arranged accordingly, form a meaningful word.
H N RC A B
1 2 3 4 5 6
(a)2,5,3,4,1,6 (2)4,1,5,6,2,3
(c) 6,3,2,4,5,1 (d)3,5,6,4,1,2
Ans. (c)
Explanation : The word is 'BRANCH'.
(227)
228 Fundamentals of Reasoning

3. In the following question, a group of letters is given which


are numbered 1, 2, 3, 4, 5 and 6. Below are given four
alternatives containing a combination of these numbers.
Select that combination of numbers so that letters
arranged accordingly, form a meaningful word.
RTAOU H
1 2 3 4 5 6
(a)2,3,6,4,1,5 (b)1,3,4,5,6,2
(c) 6,3,2,4,5,1 (d)3,5,2,6,4,1
Ans. (d)
Explanation : The word is 'AUTHOR'.
4. In the following question, a group of letters is given which
are numbered 1, 2, 3, 4, 5 and 6. Below are given four
alternatives containing a combination of these numbers.
Select that combination of numbers so that letters
arranged accordingly, form a meaningful word.
1 N L A SG
1 2 3 4 5 6
(a)2,4,3,6,1,5 (b)6,1,3,5,4,2
(c) 5,1,6,2,4,3 (d)3,4,6,1,2,5
Ans. (c)
Explanation : The word is 'SIGNAL'
5. In the following question, a group of letters is given which
are numbered 1, 2, 3, 4, 5 and 6. Below are given four
alternatives containing a combination of these numbers.
Select that combination of numbers so that letters
arranged accordingly, form a meaningful word.
RPEDI
1 2 3 4 5
(a)2,1,5,4,3 (b)1,3,2,5,4 (c) 4,3,2,1,5 (d)3,2,1,5,4
Ans. (a)
Explanation : The word is 'PRIDE'.
6. In the following question, a group of letters is given which
are numbered 1, 2, 3, 4, 5 and 6. Below are given four
alternatives containing a combination of these numbers.
Select that combination of numbers so that letters
arranged accordingly, form a meaningful word.
RMNBUE
Reasoning-Word Formation 229

(a)1,5,4,2,6,3 (b)4,6,3,2,1,5
(c) 3,5,2,4,6,1 (d)2,6,3,4,1,5
Ans. (c)
Explanation : The word is 'Number'
7. In the following question, a group of letters is given which
are numbered 1, 2, 3, 4, 5 and 6. Below are given four
alternatives containing a combination of these numbers.
Select that combination of numbers so that letters
arranged accordingly, form a meaningful word.
EO C D L I
1 2 3 4 56
(a)3,2,4,5,6,1 (b)4,2,3,6,5,1
(c) 4,2,3,1,6,5 (d)3,2,5,1.4,6
Ans. (b)
Explanation : The word is 'DOCILE'.
8. Rearrange the following jumbled letters "NKEAS" to get
a meaningful word that will indicate the name of an
animal. What is the first letter of the word so formed?
(a)N (b)K (c) A (d)S
Ans. (d)
Explanation : The correct arrangement will lead to the
word "Snake". Hence first letter is 's'.
9. The letters of the word NUPMKIP are in disorder. If they
are arranged in proper order, the name of a vegetable is
formed. What is the last letter of the word so formed?
(a)P (b)U (c) K (d)N
Ans. (d)
Explanation : The name of the vegetable is PUMPKIN.
The last Letter is N.
10. If by arranging the letters of the word NABMODINT, the
name of a game is formed, what are the first and last
letters of the word so formed?
(a)B,N (b)N,D (c) M,T (d)B,T
Ans. (a)
Explanation : The name of the game is BADMINTION.
The first and last letters are B and N respectively.
230 Fundamentals of Reasoning

PRACTICE SET
1. A chained structure of letters are numbered as 1,2,3,4,5
and 6. Four alternatives given below contain structures
of these numbers. Arrange the letters to form a
meaningful word by selecting proper structure of letters.
T E RG E R
1 2 3 4 5 6
(a)3,2,4,5,6,1 (b)1,2,3,4,5,6
(c) 6,5,4,3,2,1 (d)1,2,5,6,3,4
Ans. (c)
2. A chained structure of letters are numbered as 1,2,3,4,5
and 6. In the options, four possible arrangements of these
numbers are given. Choose the option so that it forms a
meaningful word.
K A TC E L
1 2 3 4 5 6
(a)4,2,3,1,5,6 (b)1,2,4,5,6,3
(c) 6,5,3,2,4,1 (d)3,2,4,1,6,5
Ans. (d)
3. A chained structure of letters are numbered as 1,2,3,4,5
and 6. In the options, four possible arrangements of these
numbers are given. Choose the option so that it forms a
meaningful word.
Y E EG D R
1 2 3 4 5 6
(a)4,6,2,3,5,1 (b)4,5,2,3,1,6
(c) 1,3,2,4,6,5 (d)1,2,3,4,5,6
Ans. (a)
4. A chained structure of letters are numbered as 1,2,3,4,5
and 6. In the options, four possible arrangements of these
numbers are given. Choose the option so that it forms a
meaningful word.
STE IPR
1 2 3 45 6
(a)6,5,4,3,2,1 (b)1,2,4,5,6,3
(c) 5,6,4,3,1,2 (d)4,5,6,3,2,1
Ans. (c)
5. A chained structure of letters are numbered as 1,2,3,4,5
and 6. In the options, four possible arrangements of these
Reasoning-Word Formation 231

numbers are given. Choose the option so that it forms a


meaningful word.
E E E D XC
1 2 3 4 5 6
(a)1,5,6,2,3,4 (b)1,6,5,2,4,3
(c) 1,2,3,5,6,4 (d)1,2,3,4,5,6
Ans. (a)
6. A chained structure of letters are numbered as 1,2,3,4,5,6
and 7. In the options, four possible arrangements of these
numbers are given. Choose the option so that it forms a
meaningful word.
JCO P T E R
1 2 3 4 5 6 7
(a)7,6,4,5,1,3,2 (b)4,7,3,1,6,2,5
(c) 2,4,6,1,7,5,3 (d)1,3,4,5,2,7,6.
Ans. (b)
7. A chained structure of letters are numbered as 1,2,3,4,5
and 6. In the options, four possible arrangements of these
numbers are given. Choose the option so that it forms a
meaningful word.
NGEFRI
1 2 3 4 5 6
(a)6,4,5,3,2,1 (b)1,3,2,4,6,5,
(c) 6,5,3,4,2,1 (d)4,5,6,1,2,3
Ans. (d)
8. If a meaningful word can be formed by rearranging the
letters USCACA, the first letter of the word so formed is
the answer. If no such word can be formed, the answer is
X.
(a)X (b)A (c) S (d)C
Ans. (d)
9. Choose one word which can be formed from the letters of
the given word.
RECOMMENDATION
(a)MEOICINE (b)REMINDER
(c) MEDIATE (d)COMMONICATE
Ans. (c)
232 Fundamentals of Reasoning

10. If with the third, fourth, fifth, seventh and tenth letters of
the word 'PERSONALITY', a meaningful word is formed,
then first letter of the word is the answer. If no word is
possible then X is the answer.
(a)R (b)T (c) O (d)X
Ans. (a)
qqq
33 REASONING-SEATING
ARRANGEMENT

INTRODUCTION
Questions based on 'Sitting Arrangement' involve
arranging the persons/objects according to the conditions
given in the question.
Different types of questions covered in this chapter are as
follows:
l Linear Arrangement
l Circular Arrangement
l Polygonal Arrangement
The candidate is required to understand all the
conditions given in the question as the given information may
be in jumbled form. The candidate has to segregate this
information and make the arrangement as per the given
conditions.
Arrangement may be along a straight line (row) or along
shapes like circle, triangle, rectangle etc.
Based on the various patterns of sitting arrangement,
they can be categorized as-
l Circular Seating Arrangement
l Linear Seating Arrangement
Circular seating arrangement
In this type of arrangement, objects/persons are placed
around a circle either facing the centre or facing the direction
opposite to centre. The left and right of each person/object in
both the cases can be understood with the help of following
diagrams.

(233)
234 Fundamentals of Reasoning
L = Left
L = Left L R L R = Right
R L R R = Right R
L
L A
A R
R L H B
H B
L R R L

Right Left G C
G C Left Right
L R
R L
R F D L
F D R
L
E
E L R
R L
L R R L

Facing the centre Facing the direction opposite to centre

1. Six persons are sitting in a circle facing the centre of the


circle. Akshay is between Yash and Manoj. Asha is
between Gaurav and Pankaj. Gaurav is to the immediate
left of Yash. Who is to the immediate right of Manoj ?
(a)Asha (b)Panjak (c) Gaurav (d)Akshay
Ans. (b)
Explanation :
Asha

Gaurav Pankaj

Yash Manoj

Akshay

Clearly, Pankaj is to the immediate right of manoj.


Directions (Ex. no. 2–6) : Study the given information
across the border. Major Bakshi is sitting between Major
Yadav and major Sodhi. Major Yadav is sitting immediate left
of Major Bakshi. Major Kumar is sitting second to the left of
major Yadav. Major Nanda is sitting between Major Kumar
and Major Sodhi.
2. What is the position of Major Batra?
(a)Major Batra is sitting between Major Kumar and
Major Yadav.
(b)Major Batra is second to the right of major Nanda.
(c) Major Batra is sitting to the immediate left of Major
Yadav
(d)All the above are true.
Ans. (d)
Reasoning-Seating Arrangement 235

Explanation : On the basis of given information, the


sitting arrangement of the majors will be as follows:
Major Nanda

Major Kumar Pankaj

Major Batra Manoj Bakshi

Yadav

All the options A , B and C satisfy our condition. Therefore


option D is correct.
3. Who amongst the following is sitting third to the left of
major Bakshi?
(a)Major Kumar (b)Major Batra
(c) Major Nanda (d)Major Sodhi
Ans. (a)
Explanation : From the above arrangement, it is clear
that major kumar is sitting third to the left of Major
Bakshi.
4. Who amongst the following are immediate neighbours of
major sodhi?
(a)Major Nana, Major Kumar
(b)Major Batra, Major Yadav
(c) Major Yadav, Major Bakshi
(d)Major Bakshi, Major Yadav
Ans. (d)
Explanation : Clearly Major Bakshi and Major Nanda
are immediate neighbours of Major Sodhi.
5. Who is sitting to the immediate right of Major Yadav?
(a)Major Kumar (b)MajorKalia
(c) Major Bakshi (d)Major Batra
Ans. (c)
Explanation : By observing the diagram, one can easily
conclude that Major Bakshi is sitting to the immediate
right of Major Yadav.
236 Fundamentals of Reasoning

6. What is Major Kumar position with respect to Major


Nanda?
(a)Second to the left. (b)Immediate Right.
(c) Fourth to the Right. (d)Immediate left
Ans. (b)
Explanation : Clearly, Major Nanda is to the immediate
right of Major Kumar.
7. Six persons are playing a card game. Himanshu is facing
Raghav who is to the left of Amit and to the right of
Prakhas. Amit is to the left of Dhiraj. Yogesh is to the left
of Prakhas. If Dhiraj exchanges his seat with Yogesh and
Prakhas exchanges with Raghav, who will be sitting to
the left of Raghau?
(a)Dhiraj (b)Raghav (c) Yogesh (d)Prakhas
Ans. (d)
Explanation : Arrangements according to the question
are as follows
Himanshu Himanshu

Yogesh Dhiraj Dhiraj Yogesh

Prakhas Amit Raghuav Amit

Raghav Prakhas
Original New
Arrangement Arrangement

8. 5 friends are sitting on a bench. A is to the left of B but on


the right of C.D. is to the right of B but on the left of E.
Who are at the extremes?
(a)B,D (b)C,E (c) A,D (d)A,B
Ans. (b)
Explanation : Arrangement according to the question is
as follows
Left Right
C A B D E

Clearly, C and E are at the extremes.


Linear Seating Arrangement
In a linear seating arrangement, persons are sitting in a
line or row facing north or south direction. Here also some
Reasoning-Seating Arrangement 237

conditions are given on the basis of which we have to get our


required answer.
9. There are six houses in a row. Mr Lal has Mr Bhasin and
Mr. Sachdeva as neighbours. Mr. Bhatia has Mr. Gupta
and Mr. Sharma as neighbours. Mr. Gupta's house is not
next to Mr. Bhasin or Mr. Sachedava and Mr. Sharama
does not live next to Mr. Sachedeva. Who are Mr.
Bhasin's next door neighbours?
(a)Mr. Sharma and Mr. Lal
(b)Mr. Lal and Mr. Bhasin
(c) Mr. Lal and Mr. Sachdeva
(d)Only Mr. Lal
Ans. (b)
Explanation : According to the question, the
arrangement is as follows
Mr. Sachedeva Mr. Lal Mr. Bhasin Mr. Sharma Mr. BHhaia Mr. Gupta

Directions Ex. (10 – 14) : Read the following


information and answer the questions given below
I. A,B,C,D,E,F,G and H are sitting in a row facing
North.
II. A is fourth to the right of E.
III. H is fourth to the left of D.
IV. C and F, who are not at the ends, are neighbour of B.
10. What is the position of F?
(a)Between G and H (b)Sixth to the right of D
(c) Next to the right of D (d)Next to the right of G.
Ans. (c)
Explanation : F is next to the right of E.
11. Who is/are the neighour/(s) of D?
(a)B and C (b)F alone
(c) C alone (c) Cannot be determined
Ans. (c)
Explanation : C alone is the neighbour of D
238 Fundamentals of Reasoning

12. Who are sitting at the ends?


(a)E and C (b)F and D (c) G and B (d)None
Ans. (d)
Explanation : E and D are sitting at th ends
13. Which of the following statements is not true?
(a)H is second to the right of F
(b)E is third to the left of A
(c) D is fourth to the right of H
(d)None of the above.
Ans. (d)
Explanation : D is fourth to the right of H.
14. How many persons are to the right of G?
(a)2 (b)3 (c) 4 (d)5
Ans. (d)
Explanation : Clearly five persons (i.e., H,A,B,C,D) are
to the right of G.
15. There are five books A,B,C,D and E. Book C lies above D.
Book E is below A, D is above A and B is below E. Which
book is at the top?
(a)C (b)B (c) D (d)A
Ans. (a)
Explanation : According to the given information, books
are arranged as follows
Top C

Bottom B

PRACTICE SET
Directions (Q.n. 1–5): Study the given information
carefully and answer the following question.
Eight men A,B,C,D,E,F,G and H are sitting around a
circular table facing the centre for having their lunch.
l G is not an immediate neighbour of C.
Reasoning-Seating Arrangement 239

l A is third to the right of C.


l E is second to the left of C.
l C is second to the left of B.
l F is second to the left of D.
l A is second to the left of F.
1. Who is sitting to the immediate right of C?
(a)G (b)F (c) D (d)H
Ans. (d)
2. What is the position of D with respect to H?
(a)Third to the left (b)Second to the left
(c) Immediate next (d)Fifth to the left
Ans. (b)
3. Which of the following statements is true?
(a)G is fourth to the right of H
(b)E is the neighbour F and C
(c) D and G are sitting opposite to each other
(d)B is to the immediate left of A.
Ans. (d)
4. Which of the following groups represents the immediate
neighbours of G?
(a)AB (b)EF (c) AF (d)CD
Ans. (c)
5. Among the eight men which two men are not neighbours
(a)CH (b)BA (c) GC (d)EF
Ans. (c)
6. What is the position of I?
(a)Sitting between E and D
(b)To the immediate left of G
(c) Sitting between A and H
(d)To the immediate right of B.
7. Who of the following pairs represents the immediate
neighbours of C?
(a)ID (b)DA (c) FD (d)AI
Ans. (b)
240 Fundamentals of Reasoning

8. Which girl is sitting exactly in the middle of the all girls?


(a)A (b)F (c) E (d)C
Ans. (c)
9. How many girls are seated between E and H?
(a)Three (b)Four (b)Five (d)Six
Ans. (c)
10. Who amongst the following sits exactly in the middle of
the girls who sit fifth from the right and the person who
sits seventh from the left?
(a)C (b)F (c) D (d)H
Ans. (b)
qqq
34
REASONING-DICE

INTRODUCTION
Dice are cubical shape objects containing
numbers/figures/symbols/embedded on their surfaces. Dice
problems are frequently asked in candidate selection process.
Six faces of a dice can be made by folding a piece of paper cut
in a predetermined shape having six squares as described in
the figure.
Let's have a piece of paper such as

which can be folded to form a cube in following manner.

Various faces of a dice have been described in the figures.

back face
top face
top
right face
side
Left Side Front
face
Face Face Left side
Face
Front
Face
right side
face

bottom
Face

bottom face back


face
And

Different types of questions covered in this chapter are as


follows :
l Finding Digits/Dots/Letters on the Opposite Face of
Any Particular Face

(241)
242 Fundamentals of Reasoning

l Questions Related to Unfolded Dice


CASE-1
When two positions of a single dice are given
1. In the following question, two positions of a single dice
are given. Find the face which is opposite to the face
having digit 2.
1 5
2 4 4 6

(a)3 (b)5 (c) 4 (d)6


Ans. (b)
Explanation : Since, common digit 4 is at different faces.
∴ Move digits clockwise.
1 2
4 4 1
2

Clearly, 2 and 5 are opposite to each other.


Note :
In this case, 1 and 6 are opposite and face having common
digit will be opposite to the face which is invisible.
∴ Digit 4 will be opposite to the invisible digit 3.
2. On the basis of the given two positions of single dice, find
the letter at the face opposite to the face having letter P.
A D
C C E
B

I II
(a)B (b)A (c) E (d)C
Ans. (a)
Explanation : Common letter = C (on different face)
A A
C C A
B

Clearly, B is opposite to D.
Reasoning-Dice 243

Case-2
When three positions of a single dice are given.
In this case, questions may be asked to find out opposite
face or in given question figure there will be some missing
character in any one view of dice and candidate has to
compare various letters, figures, or numbers as per given
question figures to find out answer letter, figure, or number.
3. In the following question, three positions of the same dice
are given. What will be the digit on the face opposite to
the face having digit 6?
4 2 3
1 2 3 5 6 4

I II III
(a)1 (b)3 (c) 4 (d)2
Ans. (d)
Explanation : From positions (I) and (III), common digit
= 3 (at different faces)
2 3
5 5 2
3

Clearly, 5 is opposite to 6.
4. From the given three positions of a single dice, find the
letter at the face opposite to the face having letter f.
b d e
a d a e d f

I II III
(a)b (b)a (c) e (d)f
Ans. (b)
Explanation : From positions II and III,
Common letters = d and e
Clearly, a is opposite to f.
5. From the given three position of a single dice, find the
digit at the face opposite to the face having digit 4.
244 Fundamentals of Reasoning

2 3 4
4 5 4 2 1 5

I II III
(a)1 (b)3 (c) 5 (d)6
Ans. (d)
Explanation : From all the three positions of the dice,
adjacent faces of 4 = 12, ,3 and 5.
It is clear that, the digit opposite to 4 = 5
Case-3
In some exams, candidate may be provided with four
position of dice, various questions may be asked related to the
given figure are,
l Bottom face or back face of any of the position of dice
given in the question figure.
l Choosing certain statement correct or incorrect
based on the question figure.
l To find out pair of opposite faces.
Directions (Ex. 6–7) : Four positions of a dice are given
below and answer the questions that follow.
B C F F
D B D E C D A
A

(i) (ii) (iii) (iv)

6. Which letter is on the bottom face of the figure (iii)?


(a)C (b)D (c) B (d)A
Ans. (c)
Explanation : From figure i and ii A, C, and D will lie
adjacent to B and from figure i , ii , and iii A,B,C, and F are
adjacent to D, therefore dice can be shown as:
B
A D C
F
E

Clearly, letter opposite to F is B.


Reasoning-Dice 245

7. Which statement is true ?


(a)A and C are opposite faces
(b)At the back face of figure iii is D
(c) At the back face of figure ii is A.
(c) Both (a) & (b)
Ans. (b)
Explanation : From the unfold figure it is clear that
option c is the answer.
Case-4
In this type of questions a figure of unfolded dice with
design/symbols/signs/figure etc., is given and candidates are
required to pick a dice (to be formed with the given unfolded
dice) out of the given options.
Before moving to the question, first learn about a
unfolded dice.
Unfolded Faces Opposite Faces

1 1
3 2 4 3 2 4
6 6 1 and 6 are opposite
2 and 5 are opposite
5 5
3 and 4 are opposite

3 1 3 1
2 2
6 6 1 and 6 are opposite
5 6 5 4 2 and 5 are opposite
3 and 4 are opposite

1 1
3 2 3 2
6 6 1 and 6 are opposite
5 4 5 4 2 and 5 are opposite
3 and 4 are opposite

1 1
2 2
6 3 6 3
5 5 1 and 6 are opposite
4 4 2 and 5 are opposite
3 and 4 are opposite

Some unfolded patterns of a dice are given below


246 Fundamentals of Reasoning

E C
B
F
A D

8. Which dice is similar to the question figure?


B F C F
D E C B D D A
A

(a) (b) (c) (d)

Ans. (d)
Explanation : According to unfolded dice,
E and F are opposite
B and D are opposite
C and A are opposite
Hence, option (d) is the correct answer.
9. Which dice similar to the question figure?
F
F D E F
A E
D E A B C B E
B E
C D
(a) (b) (c) (d)

Ans. (a)
Explanation : According to the question,
F

A E
B So, A is opposite to D
B is opposite to F
C D and C is opposite to E

Hence, only the cube in figure (b) is the correct answer.


10. Which dice is similar to the question figure?
Fig.
Ans. (c)
Explanation : According to the unfolded dice.
Reasoning-Dice 247

So, two blank faces of cube are opposite to two another


blank faces.
And • is opposite to
Hence, only the cube figure (a) is the correct answer
PRACTICE SET
1. From the given three positions of a single dice, find the
letter at the face opposite to the face having letter R.
P R U
T R U S T Q

I II III
(a)S (b)T (c) U (d)Q
Ans. (d)
2. From the given three positions of a single dice, find the
digit at the face opposite to the face having digit 3.
6 5 3
3 1 2 4 5 1

I II III
(a)4 (b)5 (c) 2 (d)6
Ans. (a)
3. A dice is thrown four times and its different positions are
shown below.
Which figure is opposite to
2

2
2

(i) (ii) (iii) (iv)

2
(a) (b) (c) (d)

Ans. (c)
4. From the given four positions of a single dice, find the
number opposite to face having number 5?
248 Fundamentals of Reasoning

1 3 3 2
5 4 6 5 2 1 6
4

(a) (b) (c) (d)


(a)3 (b)4 (c) 6 (d)1
Ans. (c)
5. From the given four positions of a single dice, find the
colour at the face opposite to the face having green colour.
r n ge
ite Silve Gree Oran
Wh

d
een

d
d

Re
Re
Re
Orange Violet
Gr

Orange Green

(a) (b) (c) (d)


(a)Red (b)Violet (c) Orange (d)Silver
Ans. (d)
6. Which figure will represent the question figure?
S A
P
C

R B

C C 5 5
P P R R A C R
B

(a) (b) (c) (d)

Ans. (b)
7. Which figure can be formed by complete folding of
question figure?

(a) (b) (c) (d)


Reasoning-Dice 249

Ans. (a)
8. From the given four positions of a single dice, find the
number of dots at the face opposite to the face having 3
dot.

(a) (b) (c) (d)


(a)3 (b)4 (c) 5 (d)6
Ans. (c)
9. A cube has six numbers marked 1,2,3,4,5 and 6 on its
faces. Three views of the cube are shown below
1 3 3
6 4 1 2 5 6

I II III
What possible numbers can exist on the two faces marked
A and B, respectively?
B
5
A

(a)4 and 3 (b)6 and 4 (c) 1 and 4 (b)3 and 1


Ans. (b)
10. Which figure can be formed by complete folding of
question figure?
1
6 2 6 6
6 2
2 3 6 2 4 5 1
3 5
4 5
(a) (b) (c) (d)
Ans. (d)
qqq
35 REASONING-THEME
DETECTION

INTRODUCTION
In theme detection, the problems are related to the
passages. From the passage, read the data carefully and
decide the primary point the creator is attempting to make.
What conclusion can be drawn from the contention.
Every passage is trailed by four statements. One
statement supports the creator's contention superior to the
others do. They are actually easy to solve as we do not have to
remember any grammer rule to solve them us they are
related with reading comprehension.
The solved examples given below will give you a better
idea about such type of problems.
Directions (Ex. n. 1–10) : Each of the following
questions contains a small paragraph followed by a question o
it. Read each paragraph carefully and answer the question
given below it.
1. Due to enormous profits involved in smuggling, hundreds
of people have attracted towards this anti-national
activity. Some of them became millionaires overnight.
India has been a heaven for smugglers who have been
carrying on their activities making huge profits as India
has vast coastline both on Eastern and Western coast.
From the passage, the statement that best supports the
passage is,
(a) Smuggling ought to be curbed
(b) Smuggling is increasing in our country owing to huge
profits it entails.
(c) Authorities are taking strict measures to curb
smuggling

(250)
Reasoning-Theme Detection 251

(d) Smuggling hampers the economic development of


nation.
Ans. (b)
2. People say that it is a bad habit to sleep in the afternoon
but a slight modification in the method can largely affect
your productivity. Instead of sleeping for long hours, you
can take rest for few minutes by closing your eyes.
Research says that a small nap of 10 minutes is powerful
enough to re-energize you.
From the passage, the statement that best supports the
theme of the passage is
(a) Try to sleep not more than 10 minutes in the afternoon.
(b) Do not sleep for longer hours in Asha
afternoon. Gaurav Pankaj
(c) Taking power nap can help in
increasing your productivity.
Yash Manoj
(d) None of the above
Ans. (c) Akshay

3. Yoga is very beneficial in eradicating life-threatening


diseases of the human body. Simultaneously, if not done
according to the procedure, it can have adverse effects on
the body too. Many people have complained about their
disease becoming more serious after performing yoga. It
was later found that the way they practiced was wrong.
From the passage, the statement that best supports the
theme of the passage is
(a) Yoga should be avoided
(b) Yoga should be practiced under the guidance of an
expert.
(c) Meditation is better than yoga.
(d) None of these
Ans. (b)
4. Swami Vivekananda has said that "If you are not talking
to yourself daily, them you are missing a great person." It
is true in the context that by talking to yourself daily you
can realize your strong and weak points. This will let you
keep a track of your personality.
From the passage, the statement that best supports the
theme of the passage is
252 Fundamentals of Reasoning

(a) Vivekananda has asked people to talk more.


(b) It is not good to keep silence with yourself
(c) Don't miss a great person in you.
(d) It is good to put a habit of talking to yourself daily.
Ans. (d)
5. The future of women in India is quite bright and let us
hope that they will justify their abilities by rising to the
occasion. Napolean was right when he declared that by
educating women we can educate the whole nation.
Because a country can never rise without the
contribution of 50% of their population.
From the passage, the statement that best supports the
theme of the passage is
(a) All women should be well educated.
(b) A nation can progress only when women are given
equal rights and opportunities as men.
(c) India is striving hard for the liberation of women.
(d) Women need to be imparted full freedom to prove their
worth and contribute to the progress of the nation.
Ans. (d)
6. The press should not be afraid of upholding and
supporting a just and righteous cause. It should not be
afraid of criticizing the government in healthy manner. It
should also give a balanced view of the things so that
people can be helped in the formation of a healthy public
opinion.
The passage best supports the statement that
(a) The freedom of press is essential for proper functioning
of democracy.
(b) Press has a great role to play in a democracy.
(c) All the information given by the press should be
well-articulated so as to gain a good opinion towards
the ruling party.
(d) The press is the only means to project to the masses the
polices of the government.
Ans. (a)
7. The basic principle to expand your business is that, the
company should be more service-oriented rather than
focusing only on the production. This requires active
Reasoning-Theme Detection 253

participation of the managers with the other people


rather than the products and the assemblies.
The passage well describes :
(a) Focus on service not on production
(b) Managers should be good at talking to people
(c) In the upcoming days, the most important factor will be
interpersonal skills.
(d) Managers should be good at talking to people.
Ans. (c)
8. There is a thin line that separates the two words-
"relationship" and "friendship". If one boy and girl are
closely associated with each other mentally and
emotionally, that does not mean that they are in
relationship. Good friends can also share this strong
emotional bond.
This passage is trying to convince the fact that:
(a) Friends can also share strong mental and emotional
attachment.
(b) Relationship is more complex than friendship
(c) The persons who are good friends are in relationship
with each other.
(d) The persons who are in relationships are good friends.
Ans. (a)
9. Through heavy industrial revolution, the businessman
has changed the whole gesture of people. They have
provided the people with world class luxury items and
have introduced them to the modern standard of living.
Past few centuries the standard of people has been
changed by the businessman.
The passage clearly talks about:
(a) In the past few years, business has increased rapidly
(b) Everybody should do business
(c) People need to thank the businessman
(d) When it comes to the growth of civilization and the
standards of people, the contribution of the
businessman is more.
Ans. (d)
254 Fundamentals of Reasoning

10. The prevention of accidents makes it necessary not only


that safety devices be used to guard exposed machinery
but also that mechanics be instructed in safety rules
which they must follow for their own protection, and that
lighting in the plant be adequate.
The passage best supports the statement that industrial
accidents:
(a) Usually result from inadequate lighting
(b) Can be eliminated with the help of safety rules
(c) Are always avoidable
(d) Cannot be entirely overcome.
Ans. (b)
PRACTICE SET
1. There are two types of brain tumour; one that grows very
fast and other one is just opposite. Most doctors suggest,
surgery as the primary option but unless the tumour is
removed completely, there is a fair chance of re-growing
of the tumour. Thanks to the advance technology, that
provides the option of complete removal of tumour
through radiation therapy. It is a painless procedure and
during treatment patient can continue his/her regular
duty.
(a) Radiation therapy is the primary option for treating
tumours
(b) Radiation therapy is a painless procedure
(c) Surgery is not the primary solution for every tumour.
(d) Not every doctor is upgraded with latest technology.
Ans. (c)
2. Gone are the days where you used to score good marks by
reading your books and subjects prescribed in your
course of syllabus. Now every board of education in
changing their style of education and exam format.
Nowadays, to get good marks, you have to read the out of
the regular course of syllabus and also take help of the
Internet if needed.
The passage well describes the that
(a) Internet is necessary nowadays.
Reasoning-Theme Detection 255

(b) You need to focus less on outside activities and more on


study.
(c) Knowledge of a particular subject in all aspects is
important.
(d) Study has become so important that all extracurricular
activities should be curbed.
Ans. (c)
3. Throughout the ages the businessman has helped build
civilization great cities, provided people with luxuries
and artists with patronage, and lift his fellow citizens to
understand the standard of living.
The passage best supports the statement that the
business-
(a) Lives luxurious & comfortable life
(b) Is capable of raising his standard of living
(c) Both (a) & (b)
(d) Has contributed to the growth of civilization.
Ans. (d)
4. Smartphones are making people crazy. They are wasting
their valuable time chatting with others, surfing on net,
and playing games but still there are many advantages of
having a smartphone. Top C

What is the theme of the passage? D


(a) Author is optimistic on using smartphones A
(b) Author is pessimistic on using smartphones E
(c) Author is neutral on using smartphones Bottom B
(d) None of the above
Ans. (a)
5. There is a shift in our economy from a manufacturing to a
service orientation. The increase in service sector will
require the managers to work more with people rather
than with objects and things from the assembly line.
The passage best supports the statement that:
(a) Manufacturing organizations ignore importance of
people.
(b) Interpersonal skills will become more important in the
future work place.
(c) Managers should have a balanced mind
256 Fundamentals of Reasoning

(d) Service organisations will not deal with objects and


things.
Ans. (d)
6. To forgive an injury is often considered to be a sign of
weakness, it is really a sign of strength. The man who
forgives an injury proves himself to be superior of the
man who wronged himself & puts the wrong-does to
shame.
The passage best supports the statement that:
(a) A person with calm and composed nature has depth of
thought.
(b) Mercy is noblest form of revenge.
(c) People end to forgive the things happened in the past.
(d) None of the above
Ans. (b)
7. Most of the child laborers are engaged in agriculture and
allied subject like livestock, foresting and fisheries. In
Urban areas, children work in dhabas, cleaners in trucks
and as domestic servant etc. They have long working
hours, bad and unhygienic working conditions and fewer
wages.
The passage best supports the statement that:
(a) Child labour is a social evil in every nation.
(b) Almost every country in the world has laws relating to
& aimed at preventing child labour.
(c) Poverty compels poor parents to make their children
employed as labourers.
(d) Rise in Urban child labour attributed to increased
migration for employment.
Ans. (a)
8. Terrorism is one of the bad debts of the world and the
people's are affected by this form of violence. Terrorism
means the policy of striking terror in the minds of the
people by violent methods to achieve some ends.
The passage best supports the statement that.
(a) High profile attacks in major cities have set the world
on edge.
(b) Over 31000 people from 86 countries have join ISIS of
since June 2014.
Reasoning-Theme Detection 257

(c) It is a law of the jungle to use muscular power & force to


get things done.
(d) Cyber attacks could be bigger threat to world than
terrorism.
Ans. (c)
9. As many as thirty sightings of king cobras were recorded
in residential areas across Kerala within a year & most of
them were caught from bathrooms & courtyards of
houses and roads.
(a) The grasslands in Kerala were under increasing threat
from tourism
(b) Large scale deforestation and disturbances caused by
poachers could be forcing king cobras to migrate
(c) The occurrences were strange as king cobras were
never known to trespass into human territory
(d) None of the above.
Ans. (b)
10. Writers are those free birds who have the liberty of
expressing their thoughts against the different sections
of society, including government. But nowadays, they are
reaming under the political pressure. Some do not even
fear to be bribed.
From the passage, the statement that best supports the
theme of the passage is
(a) Writers should not work under anyone's pressure
(b) Nowadays, everyone is reaming under political
pressure.
(c) Those writers who are accepting bribe should be jailed.
(d) Nowadays, everyone is reaming under political
pressure.
Ans. (a)
qqq
36 REASONING-CAUSE
AND EFFECT

INTRODUCTION
Cause is the logical or scientific reason of an event that
has occurred and Effects to this cause are the consequences of
that event.
Different types of question covered in this chapter are as
follows
l Statement and Direction Based Questions
l Direct Cause and Effect Based Questions
Different Types of Causes
Immediate Cause : It immediately precedes the effect.
In other words immediate causes are the most proximate in
time, to the effect.
Principal Causes : It is the most important reason
behind the effect.
Common Causes : Two effects given in two statements
may be caused by a third unmentioned event which may be
called the common cause of the given events.
Difference between Principal Cause and Immediate Cause
Principal and immediate cause sound similar but there is
a difference between principal and immediate cause. The
principal cause is the main and most important reason for the
effect, while immediate cause is the most proximate in time to
the effect.
Directions (Q. no. 1–30) : In this type of questions, two
statements are given and the student has to identify whether
they are independent causes or effects of independents causes
or a common cause etc., and accordingly have to select the
answer options.

(258)
Reasoning-Cause and Effect 259

Following examples will give a better understanding


about the types of questions asked.
Give answer :
(a) If Statement A is the cause and Statement B is its
effect.
(b) If Statement B is the cause and Statement A is its
effect.
(c) If both statements are independent causes.
(d) If both statements are effects of independent causes.
(e) If both statements are effects of some common cause.
1. A. There has been a curfew in the city.
B. 24 hours police patrolling is going on in the city.
Ans. (E)
Explanation : Clearly Both the statements are directing
towards a common reason. To simply our thought process
we can assume that for some common reason like
Hindu-Muslim riot must have been reason this type of
situation.
2. A. The prices of pulses have been increased considerably
during the summer.
B. There is tremendous increase in the temperature
during this summer there by damaging crops greatly.
Ans. (b)
Explanation : Clearly, damage to crops due to high
temperature may have resulted in a short supply of
pulses and an increase in their prices.
3. A. Worldwide recession has created uncertainty in job
market.
B. Many people are opting for change from private sector
to public sector.
Ans. (a)
Explanation : Many people are opting for change from
private sector to public sector because worldwide
recession has created uncertainly in the job market. It is
260 Fundamentals of Reasoning

considered that public sector jobs are more certain than


the private sector jobs, So, Statements A is the cause
while Statements B is the effect.
4. A. The Government has decided to increase the prices of
LPG cylinders with immediate effect.
B. The Government has decided to increase the prices of
LPG cylinders with immediate effect.
Ans. (e)
Explanation : It seems the price of Petroleum has
increased in general. Alternatively, subsidies may have
been reduced, again a cause common to both the
statements.
5. A. The Government has suspended several police officers
in the city.
B. Four persons carrying huge quantity of illegal alcohol
were arrested by police.
Ans. (d)
Explanation : A certainly cannot be the effect of B, if we
believe in an ethical world. A is the effect of bad policing
and B of good policing.
6. A. The expansion of few small engineering colleges is
restricted by the AICTE.
B. The bigger engineering colleges like IIT, NIT and state
government colleges are top in competition and small
private colleges are not in a position to withstand the
competition.
Ans. (b)
Explanation : The small colleges in education sector are
not competitive so AICTE put restrictions on them.
7. A. Most of the students enrolled themselves for the
educational tour scheduled for next month.
B. The college authority cancelled the educational tour
scheduled for next month.
Ans. (c)
Reasoning-Cause and Effect 261

Explanation : A happened so that students could see


more o the world. B happened so that the college may
attend to other important task.
8. A. In Odisha, people living in the low lying areas have
been evacuated to safer places in large number during
the last few days.
B. The Odisha government has rushed to the affected
areas to provide relief Supplies to the people.
Ans. (e)
Explanation : Both the statements are response of some
common reason i. e., cyclone.
9. A. Majority of the parents signed a joint petition and are
complaining against the open environment for students
in colleges.
B. The BPUT University has instructed all the colleges
under its jurisdiction to ban smoking inside the college
premises as well as in the hostels.
Ans. (a)
Explanation : As the guardians have complained
against open environment so it now necessary to take
appropriate steps so option follows.
10. A. For the past few months the prices of spices have not
changed in the domestic market.
B. For the past few months the prices of spices have gone
up substantially in the international market.
Ans. (c)
Explanation : Clearly, each statement is self-sufficient
in itself and stands independent to the other.
PRACTICE SET
Directions (Q. no. 1–10) : Below in each question are
given two Statements A and B. These statements may be
either independent causes or may be effects of independent
causes or of a common cause. One of these statements may be
the depicts the relationship between these two statements.
262 Fundamentals of Reasoning

Give answer :
(a) If Statement A is the cause and Statement B is its
effect.
(b) If Statement B is the cause and Statement A is its
effect.
(c) If both statements are independent causes.
(d) If both statements are effects of independent causes.
(e) If both statements are effects of some common
cause.
1. A. Today's life of IT professionals is hectic, fast,
demanding and result oriented so, it leads to a stressful
situation.
B. Whether it is IT professional or any other professional,
heart diseases are increasing day by day.
Ans. (e)
2. A. 64 students fall sick after consuming mid-day meal in
UP.
B. Lucknow's international airport apron is seen flooded
following monsoon rains in Indian state of UP.
Ans. (d)
3. A. There has been a curfew in the city.
B. Doctors advice to eat an apple every day.
Ans. (d)
4. A. Majority of aurobindonagar citizens have decided to
protest against the construction of a government office
instead of a park.
B. There are no parks in that locality.
Ans. (b)
5. A. Frequent robberies in jewellery shops were recorded in
distant suburbs of the city.
B. Shop owners in the city and suburbs demanded
improvement in security situation from the police
authorities.
Ans. (a)
Reasoning-Cause and Effect 263

6. A. Police resorted to lathi charge to disperse the unlawful


gathering of large number of people.
B. The citizen's forum called a general strike in protest
against the police atrocities.
Ans. (a)
7. A. The private medical colleges have increased the
tuition fees in the current year by 12% over the last year's
fee to meet the expenses.
B. The Government medical colleges have not increased
their fees inspite of price evaluation.
Ans. (c)
8. Statement A. Many people visits the religious places on
week days and weekends to pray to Mother Durga in
Navratras.
Statement B. Many religious people go on fasting
during Navratras to seek the blessings of Mother Durga.
Ans. (e)
9. A. The government has reduced the petrol prices by Rs.
2.50 after increasing the prices by Rs. 4
B. The tax on petrol imports has dropped marginally
during the week.
Ans. (b)
10. A. The prices of vegetables have increased substantially
during the last few days.
B. The prices of fruits have dropped substantially during
the last few days.
Ans. (d)
qqq
37
REASONING-CODING DECODING

INTRODUCTION
Coding-Decoding is process of transmitting an
information from one place to other using some suitable
codes, so that it might reach to other person safely.
Coding-decoding of an information is done with various rules
or patterns, so that only the right person can decipher it.
Type-1 Coding Based on Rearrangement of Letters
In this type of coding, the letters of the original word are
rearranged in a particular manner to obtain the code. Such
coding can be of following types
Following examples will give you a better idea about such
questions.
1. In a certain code language 'MADRAS' is written as
'DAMSAR', how can HOCKEY' be written in that code
language?
(a)CKYEOH (b)KEYCOH
(c) COHYEK (d)YEKCOH
Ans. (c)
Explanation :
As, M A D R A S

D M S R
A A

∴ COHYEK
Similarly, H O C K E Y

C H Y K
O E

(264)
Reasoning-Coding Decoding 265

2. If in a certain code language 'RIGHT' is written as


'GHRTI', then how will VENUS' be written in that code?
(a)NUVSE (b)NUSVE (c) NVUSE (d)NUEVS
Ans. (a)
Explanation : As, R I G H T → G H R T I
1 2 3 4 5 3 4 1 5 2

Similarly, V E N U S → N U V S E
1 2 3 4 5 3 4 1 5 2

∴ VE NUS = NUVSE
3. If 'MEAT' is written as 'TEAM', then DEAR is written as
(a)RDAE (b)RAED (c) READ (d)RDEA
Ans. (c)
Explanation : As, M E A T T E A M

Similarly, D E A R R E A D

4. If in a certain code language, 'BROWSER' is written as


'RESWORB', then how 'TEACHER' be coded in that same
language?
(a)REHCTEA (b)AHRCTEA
(c) REHCEAT (d)REHCAET
Ans. (d)
Explanation :
As, 1 B R7 Similarly, 1 T R7
2R E6 2E E6
3O S5 3A H5
4W W4 4C C4
5S O3 5H A3
6E R2 6E E2
7R B1 7R T1

∴ TEACHER ⇒ REHCAET
Type-2 Fictitious Language Coding
In this type of questions, some messages are provided in
the code language and some codes are assigned to each word
of the messages. The candidates are required to decipher the
code of each word by finding the common code for two words
266 Fundamentals of Reasoning

and this process is followed to decipher the code for each word
thereafter and hence the entire message is decoded.
Following example will give you a better idea about the
type of questions asked.
5. If in a certain code language 'fruits are sweet' is written
as 'pa ma la' and 'you are sweet' is written as 'pa ma ta',
then find the code for 'fruits.'
(a)pa (b)la (c) ma (d)ta
Ans. (b)
Explanation :
fruits are sweet pa ma la ...(i)

you are sweet pa ma la ...(ii)

From Eqs. (i) and (ii),


'are sweet' ⇒ pa ma
∴ fruits ⇒ la
6. In a certain language, 'sun shines brightly' is written as
'ba lo sul', 'houses are brightly lit' as 'kado ula ari ba' and
'light comes from sun' as 'dopi kup lo mo'. What are the
codewords for 'sun' and 'brightly?
(a)ba, lo (b)sul, lo (c) ba, sul (d)lo, ba
Ans. (d)
Explanation : Sun shines brightly → ba lo sul …(i)
houses are brightly lit → kado ula ari ba …(ii)
light comes from sun → dopi kup lo mo …(iii)
From Eqs. (i) and (ii), brightly → ba
From Eqs. (i) and (iii), sun → lo
Hence, sun → lo and brightly → ba
7. If in a certain code language 'where are you' is written as
'pit ka ta' and 'are they there' is written as 'sa da ka' and
'they may come' is written as 'da na ja', then how will
'there' be written in that language?
(a)sa (b)ka
(c) da (d)Data inadequate
Ans. (a)
Reasoning-Coding Decoding 267

Explanation : where are you → pit ka ta …(i)


are they there → sa da ka … (ii)
they may come → da na ja …(iii)
From Eqs. (i) and (ii),
are ⇒ ka
From Eqs. (ii) and (iii),
∴ they ⇒ sa
Type-3 Number Coding
In number coding, numerical code is assigned to the word
or the alphabetical letters and candidates are required to
analyse the codes as per the correlation between these
numbers and letters. This correlation is based on certain
pattern according to the position of letters in English
alphabet as per a set of given rules.
Following example will give you a better idea about the
type of question asked.
8. If 'GERMANY' is written as '7, 5, 18, 13, 1, 14, 25.' how
can 'FRANCE' be written in that code?
(a)8,2,14,5,13,6 (b)6,3,18,14,1,5
(c) 8,16,14,3,1,5 (d)6,18,1,14,3,5
Ans. (d)
Explanation : Letters are coded with their
corresponding positions.
As, G E R M A N Y

7 5 18 13 1 14 25
Similarly, F R A N C E

6 18 1 14 3 5

∴ FRANCE ⇒ 6,18,1,4 ,14 ,3,5


9. If P = 16 and TAP = 37, then CUP = ?
(a)36 (b)38 (c) 40 (d)39
Ans. (c)
Explanation : As, P = 16
268 Fundamentals of Reasoning

As, 20 1 16 Similarly, 3 21 16
G E R C U P

20 + 1 + 16 = 37 3 + 21 + 16 = 40

10. If DREAM' is coded as '78026' and 'CHILD' is coded as


'53417', how can 'LEADER' be coded?
(a)102087 (b)102708 (c) 102780 (d)102078
Ans. (b)
Explanation :
As, D R E A M

7 8 0 2 6

and C H I L D

5 3 4 1 7

Similarly, L E A D E R

1 0 2 7 0 8

∴ LEADER ⇒ 102708
PRACTICE SET
1. If in a certain code language 'HONESTY' is written as
'5132468' and 'POVERTY' is written as '7192068', then
how will 'HORSE' be written in that language?
(a)50124 (b)51024 (c) 51042 (d)52014
Ans. (b)
Directions (Q.n. 2 – 4) : Study the following information
carefully to answer the given questions.
'time and money' is written as 'ma jo ki'.
'manage time well' is written as 'p ru jo,
'earn more money' is written as 'zi ha ma' and
'earn well enough' is written as' si ru ha'.
2. What is the code for 'manage'?
(a)ha (b)ru (c) pa (d)jo
Ans. (c)
3. What is the code for 'earn'?
(a)ha (b)ru (c) ma (d)si
Reasoning-Coding Decoding 269

Ans. .(a)
4. Which of the following represents more time?
(a)ma ki (b)pa jo (c) jo zi (d)zi ki
Ans. (c)
5. In a certain language, 'EXECUTIVE' is coded as
'TCIEUXVEE', then how is 'MAUSOLEUM' coded in that
same language?
(a)AUEUOSEMM (b)LSEUOAUMM
(c) SLUEOAUMM (d)AUUCOSLMM
Ans. (b)
6. If in a certain code language 'PAPER' is written as
'16,1,16,5,18', then how will 'BRINJL' be written in that
language?
(a)2,18,9,14,10,21 (b)2,38,94,1,10,21
(c) 2,31,89,14,01,12 (d)2,18,9,14,10,12
Ans. (d)
7. In a certain code, 'KAVERI' is written as 'VAKIRE'. How
is 'MYSORE' written in that same code?
(a)SYMERO (b)SYMROE
(c) SYMEOR (d)EROSYM
Ans. (a)
8. If C = 3 and POLISH = 79, then POINTER = ?
(a)98 (b)96 (c) 97 (d)95
Ans. (c)
9. If in a certain code language 'monday is a holiday' is
written as 'sa da pa na' and they enjoy a holiday' is
written as 'da na ta ka', then how will 'monday' be written
in that language?
(a)pa (b)sa (c) da (d)sa
Ans. (d)
10. If T = 20 and TEAM = 39, then 'TREE = ?
(a)39 (b)48 (c) 36 (d)54
Ans. (b)
qqq
38
REASONING-ARITHMETICAL

INTRODUCTION
Arithmetical reasoning contains calculation with special
sense of reasoning. This reasoning chapter normally contains
all the chapters from quantitative aptitude. So, it is one of the
most interesting chapters in reasoning, because it contains
both aptitude and reasoning. Arithmetic reasoning may
contain the following chapters of aptitude-
l Time and work
l Time, speed, and distance
l Simple interest
l Compound interest
l Percentage
l Profit and loss
l Average
l Ratio and proportion
Let's explain a little bit of each type arithmetic reasoning
under this heading-
Time and work : Problems on time and work will be of
normal men work and men women work in 10 days, and after
this it is asked that 10 men will take how many days to do the
same work then at first we have to find that, 1 man can do the
job in how many days and then we can proceed further.
Time speed and distance : For problems regarding this
chapter, there is one formula which we can use in this context
i. e., distance = time × speed.
Simple interest : If P is taken as principal, R is taken as
rate of interest, T is taken as time, and I is taken as interest
then the relationship between them is
(270)
Reasoning-Arithmatical 271

P ×R×T
I=
100
Compound Interest : If P is principal, R is rate, amount
is A and time is n years then if interest is-
n
 R 
Compounded annually : A = P  1 + 
 100
n
 R 
Compounded half yearly : A = P  1 + 
 200
4n
 R 
Compounded quarterly : A = P  1 + 
 400
Percentage : The term 'per cent means one out of a
hundred. Percentages are used to describe parts of a whole,
the whole being made up of a hundred equal parts and is
denoted by %
Profit and loss : Profit = selling price – cost price and
Profit
Profit % = × 100.
C.P
Loss
Loss = cost – selling and Loss % = × 100
C.P
Average : The average is a measure of central point of a
set of numbers. It is an estimation of where the centre point or
weight of a set of number lies.
sum of sets of N numbers
Average =
N
The following examples will give you a better idea about
such questions.
1. Riyaz and Saqlain are two workers and they work for
GPR pumps and pipes. Riyaz is twice as good a workman
as Saqlain and together Riyaz and Saqlain finish a piece
of work in 20 days. In how many days will Riyaz alone
finish the work?
(a)90days (b)66 days (c) 30 days (d)29 days
Ans. (c)
272 Fundamentals of Reasoning

Explanation : If Riyaz takes x days to do a work then


Saqlain takes 2x days to do the same work.
1 / x + 1 / 2x = 1 / 20
3 / 2x = 1 / 20
x = 30
Hence, Riyaz alone can finish the work in 30 days.
2. How long will a 150m long train running at a speed of 60
km/hr take to cross a bridge of length 300m?
(a)45 sec (b)18 sec (c) 50 sec (d)27 sec
Ans. (d)
Explanation :
Total distance = 300 + 150 = 450m
 5  50 
Speed = 60km/hr =  60 ×  m/sec =   m/sec
 18  3
Distance = Speed × Time
450
Time = = 9 × 3 = 27 seconds
50 / 3
3. P and Q can finish a work in 18 & 15 days. Q starts the
work and leaves it after 10 days. The number of days in
which P alone can the remaining work is:
(a)6 days (b)10 days (c) 4 days (d)8 days
Ans. (a)
Explanation : Work done by Q in one day = 1 / 15
Work done by Q in 10 days = 10 / 15 = 2 / 3
Remaining work = 1 − 2 / 3 = 1 / 3
Number of days in which P can finish the remaining work
1/3
= = 6 days
1 / 18
4. A can do a piece of work in 4 hours, B and C together can
do it in 3 hours, while A and C together can do it in 2
hours. How long will B alone take to do it?
(a)18 hours (b)6 hours (c) 10 hours (d)12 hours
Ans. (d)
Reasoning-Arithmatical 273

1
Explanation : A'S 1 hour's work = ; ( B + C ) 's 1 hour's
4
1 1
work = ; ( A + C ) 's 1 hour's work =
3 2
 1 1 7
( A + B + C )' s 1 hour's work =  +  =
 4 3 12
7 1 1
B'S 1 hour's work = − =
12 12 12
∴ B alone will take 12 hours to do the work.
5. Average of 5 terms is 10. Average of first two terms is 7,
and last two terms is 13. What is the value of third term?
(a)9 (b)10 (c) 8 (d)7
Ans. (b)
Explanation : Sum of 5 terms = 10 × 5 = 50
Sum of first two terms = 2 × 7 = 14
Sum of last two terms = 13 × 2 = 26
Third term = 50 − 14 + 26 = 10
6. A bag contain Rs; 150 paisa and 25 paisa coins in the ratio
8:9:11. If the total money in the bag is Rs. 366. Find the
number of Rs. 25 paisa coins?
(a)245 (b)275 (c) 264 (d)120
Ans. (c)
Explanation : Let number of coins denomination be x .
Then 1 × 8x + 1 / 2 × 9x + 1 / 4 × 11x = 366
61x / 4 = 366
x = 24.
Hence, 25 paisa coins = 11x = 11 × 24 = 264 .
7. There is 60% increase in an amount in 6 years at simple
interest. What will be the compound interest of Rs. 12000
after 3 years at the same rate?
(a)Rs. 6240 (b)Rs. 3972
(c) Rs. 2754 (d)Rs. 3016
Ans. (b)
Explanation :
274 Fundamentals of Reasoning

Let P = Rs. 100, then S.I = Rs.60 and T = 6 years


 100 × 60
∴ R=  = 10% p.a.
 100 × 6 
Now, P = Rs. 12000, T = 3 years, R = 10% p.a.
3
 10 
∴ A = 12000 ×  1 + 
 100
= Rs. 15972
∴ CI = Rs. (15972 – 12000)
= Rs. 3972.
8. If 20% of a = b, then b% of 20 is the same as :
(a)5% of a (b)10% of a (c) 8% of a (d)4% of a
Ans. (d)
20 a a
Explanation : 20% of a = b ⇒ =b⇒b=
100 5
b a × 20 4a
∴ b% of 20 = × 20 = = = 4% of a
100 5 × 100 100
9. Rs. 41517 is distributed among A , B and C in the ratio of
3:7:11? What is B's share?
(a)Rs. 13839 (b)Rs. 5931
(c) Rs. 21747 (d)Rs. 19770
Ans. (a)
 7 
Explanation : B share = 41517 ×  
 3 + 7 + 11
7
= 41517 ×
21
= Rs.13839.
10. Richa purchased 20 dozens of toys at the rate of Rs. 375
per dozen. They sold each of them at the rate of Rs. 33.
What was his profit percentage?
(a)5.6 (b)4.4 (c) 6.2 (d)2.4
Ans. (a)
 375
Explanation : C.P of 1 toy = Rs.   = Rs.3125
.
 12 
Reasoning-Arithmatical 275

SP of 1 toy = Rs. 33
∴ Profit = SP – CP = Rs.175
.
 175
. 
∴ Profit % =  × 100 %
 3125
. 
= 56
. %
PRACTICE SET
1. The average age of 6 persons is 17.5 years. When one
person left the room, average age becomes 16 years. What
is age of the person who left?
(a)23 years (b)25 years (c) 30 years (d)33 years
Ans. (b)
2. P is thrice as good as Q and is therefore able to finish the
work in 60 days less than Q. If they work together, they
can finish the work in:
1
(a)26 days (b)18 days (c) 22 days (d)24 days
2
Ans. (c)
3. Total weight of A&B is 120 kg. If A weights 30 kg more
than B? What is ratio of B:A?
(a)5:3 (b)3:2 (c) 2:3 (d)3:5
Ans. (d)
4. The profit percentage earned by selling an article for Rs.
1920 is equal to the Loss percentage incurred by selling
the same article for Rs. 1280. At what price should the
article be sold to make 22% profit?
(a)Rs. 2196 (b)Rs. 2000
(c) Rs. 1952 (d)data inadequate
Ans. (c)
5. What is the difference between the compound interests
1
on Rs. 5000 for 1 years at 4% p.a compounded yearly
2
and half-yearly?
(a)Rs. 2.04 (b)Rs. 6.03 (c) Rs. 4.80 (d)Rs. 1.70
Ans. (a)
276 Fundamentals of Reasoning

6. An aeroplane covers a certain distance at a speed of 240


2
km/hr in 5hours. To cover the same distance in 1 hours,
3
it must travel at a speed of :
(a)720 km/hr (b)600 km/hr
(c) 480 km/hr (d)360 km/hr
Ans. (a)
7. A fruitseller had some mangoes. He sells 40% mangoes &
still has 420 mangoes. Originally he had :
(a)600 mangoes (b)672 mangoes
(c) 700 mangoes (d)588 mangoes
Ans. (c)
8. What percentage of number from 1 to 80 have 1 or 9 in the
unit's digit?
(a)8% (b)20% (c) 16% (d)24%
Ans. (b)
9. Two numbers are respectively 20% and 50% more than a
third number. The ratio of two numbers is:
(a)2:5 (b)6:7 (c) 3:7 (d)4:5
Ans. (d)
10. Find the length of the bridge which a train 130 metres
long and travelling at 45 km/hr can cross in 30 seconds.
(a)200m (b)245m (c) 250m (d)225m
Ans. (b)
qqq

You might also like